SlideShare a Scribd company logo
1 of 46
ĐẠI HOC THÁI NGUYÊN
TRƯ NG ĐẠI HOC KHOA HOC
Tải tài liệu tại sividoc.com
Viết đề tài giá sinh viên – ZALO:0973.287.149-TEAMLUANVAN.COM
VŨ DUY ĐẠT
M T SO DẠNG TOÁN
LIÊN QUAN ĐEN XÁC SUAT R I RẠC
VÀ ỨNG DỤNG
Chuyên ngành: PHƯƠNG PHÁP TOÁN SƠ CAP
Mã so: 8 46 01 13
LU N VĂN THẠC SĨ TOÁN HOC
Người hướng dan khoa hoc: GS.TSKH. Nguyen Văn M u
THÁI NGUYÊN - 2019
ĐẠI HOC THÁI NGUYÊN
TRƯ NG ĐẠI HOC KHOA HOC
Tải tài liệu tại sividoc.com
Viết đề tài giá sinh viên – ZALO:0973.287.149-TEAMLUANVAN.COM
VŨ DUY ĐẠT
M T SO DẠNG TOÁN
LIÊN QUAN ĐEN XÁC SUAT R I RẠC
VÀ ỨNG DỤNG
Chuyên ngành: PHƯƠNG PHÁP TOÁN SƠ CAP
Mã so: 8 46 01 13
LU N VĂN THẠC SĨ TOÁN HOC
Người hướng dan khoa hoc: GS.TSKH. Nguyen Văn M u
THÁI NGUYÊN - 2019
i
Viết đề tài giá sinh viên – ZALO:0973.287.149-TEAMLUANVAN.COM
Mnc lnc
L I CẢM ƠN ii
M ĐAU 1
Chương 1. M t so dạng toán liên quan đen xác suat r i rạc 2
1.1 Phép thả và bien co....................................................................................2
1.2 Xác suat của bien co...................................................................................3
1.2.1 Định nghĩa cő đien của xác suat....................................................3
1.2.2 Định nghĩa thong kê ve xác suat....................................................6
1.3 Định lý c®ng xác suat ............................................................................. 6
1.4 Định lý nhân xác suat............................................................................. 9
1.5 M®t so mở r®ng của định lý c®ng và định lý nhân xác suat ..................13
1.6 Bien ngau nhiên và kì vong ......................................................................20
1.6.1 Định nghĩa.....................................................................................20
1.6.2 Tính tuyen tính của kì vong.........................................................22
Chương 2. Ứng dnng phương pháp xác suat trong giải toán trung
hoc pho thông 24
2.1 Áp dụng xác suat và kì vong vào m®t so bài toán thi hoc sinh giỏi . 24
2.2 M®t so dạng toán thi Olympic liên quan...................................................34
KET LU N 40
TÀI LI U THAM KHẢO 41
ii
Viết đề tài giá sinh viên – ZALO:0973.287.149-TEAMLUANVAN.COM
L i cảm ơn
Lu n văn này được hoàn thành với sự hướng dan t n tình của GS.TSKH
Nguyen Văn M u - Đại hoc Khoa hoc tự nhiên - Đại hoc Quoc gia Hà N®i. Tự đáy
lòng mình, em xin tỏ lòng biet ơn sâu sac tới Thay đoi với sự quan tâm, chỉ bảo
t n tình của Thay. Em xin chân thành cảm ơn các thay cô trong Trường Đạị hoc
Khoa hoc - Đại hoc Thái Nguyên, đã giúp đơ em trong suot quá trình theo hoc.
Tôi cũng xin chân thành cảm ơn Ban Giám hi u, các đong nghi p Trường THPT
Đông Thành - Quảng Ninh và gia đình đã tạo đieu ki n cho tôi hoàn thành ke
hoạch hoc t p.
M c dù có nhieu co gang nhưng lu n văn khó tránh khỏi nhǎng thieu sót và
hạn che. Tác giả mong nh n được nhǎng ý kien đóng góp của các thay cô và các
bạn đoc đe lu n văn được hoàn thi n hơn.
Xin chân thành cảm ơn!
Thái Nguyên, tháng 5 năm 2019
Tác giả
Vũ Duy Đạt
1
Viết đề tài giá sinh viên – ZALO:0973.287.149-TEAMLUANVAN.COM
M ĐAU
Lu n văn nham cung cap các dạng toán xác suat rời rạc và các bài toán áng
dụng phương pháp xác suat trong giải toán trung hoc phő thông. Chuyên đe nam
trong chương trình phục vụ cải cách giáo dục và boi dương hoc sinh giỏi ở các lớp
chuyên toán phục vụ kỳ thi hoc sinh giỏi quoc gia, Olympic khu vực và quoc te.
Trong chương trình cải cách giáo dục hi n nay, các van đe liên quan đen xác
suat và thong kê đã được phê duy t đe giảng dạy chính thác trong các trường
trung hoc phő thông. nhieu nước trên the giới, trong các kì thi hoc sinh giỏi
toán các cap, Olympic Toán khu vực và quoc te có nhieu đe toán liên quan tới
lý thuyet và phương pháp xác suat, thong kê. Nhǎng dạng toán này thường được
xem là thu®c loại khó vì phan kien thác nâng cao ve chuyên đe này hi n nay không
nam trong chương trình chính thong của sách giáo khoa hi n hành b c trung hoc
phő thông.
Đe đáp áng nhu cau boi dương giáo viên và boi dương hoc sinh giỏi ve chuyên
đe xác suat và phương pháp xác suat, tôi chon đe tài lu n văn ”M®t so dạng toán
liên quan đen xác suat rời rạc và áng dụng”. Trong đó, khảo sát m®t so lớp bài
toán tà các đe thi hoc sinh giỏi Quoc gia và Olympic các nước nhǎng năm gan
đây ve chuyên đe này.
Cau trúc lu n văn gom 2 chương:
Chương 1. M®t so dạng toán liên quan đen xác suat rời rạc.
Chương 2. Úng dụng phương pháp xác suat trong giải toán trung hoc phő thông.
Thái Nguyên, tháng 5 năm 2019
Tác giả
2
Viết đề tài giá sinh viên – ZALO:0973.287.149-TEAMLUANVAN.COM
Chương 1. M t so dạng toán liên
quan đen xác suat r i rạc
Trong chương này trình bày cơ sở lý thuyet cùng các bài toán áp dụng nâng
cao ve phép tính xác suat trong đại so, áp dụng nhǎng quy tac tőng quát như quy
tac c®ng, quy tac nhân, công thác Bernoulli cho phép thả l p, công thác xác suat
đay đủ, công thác Bayes vào các ví dụ thực te. Nghiên cáu tính toán đại lượng
đ c trưng bien ngau nhiên rời rạc như kỳ vong, phương sai... trong thong kê. Tài
li u tham khảo chính trong phan này là các cuon [4], [5], [6], [7].
1.1 Phép thfi và bien co
Định nghĩa 1.1. Vi c thực hi n m®t nhóm các đieu ki n cơ bản đe quan sát m®t
hi n tượng nào đó có xảy ra hay không được goi là thực hi n m®t phép thả.
Ví dn 1.1. Thực hi n tung m®t đong xu bon lan là m®t phép thả nham quan sát
trong bon lan tung đong xu ta nh n được m t ngảa hay m t sap. Khi đó tat cả
các ket quả có the xảy ra sau 4 lan tung là 24 = 16. Khả năng xảy ra m®t trong
các ket quả là
1
.
16
Định nghĩa 1.2 (xem [4]). Không gian mau Ω của phép thả T là t p hợp tat cả
các ket quả của m®t phép thả T sao cho các ket quả có khả năng xảy ra như nhau.
T p A ⊂ Ω thì A được goi là bien co ngau nhiên. Khi A = Ω thì A được goi là
bien co chac chan (chac chan xảy ra). Khi A = ∅ thì A được goi là bien co không
(không xảy ra).
3
Viết đề tài giá sinh viên – ZALO:0973.287.149-TEAMLUANVAN.COM
1.2 Xác suat của bien co
Trở lại với ví dụ ở trên ta xét bien co A là "ket quả bon lan tung có ít nhat
ba lan xuat hi n m t ngảa". Ket quả của Ω làm A xuat hi n là: NNNN, NSNN,
NNSN, NNNS, SNNN. Khi đó năm ket quả này được goi là ket quả có lợi cho bien
co A, khả năng xảy ra bien co A là
5
.
16
Định nghĩa 1.3 (xem [4]). Xác suat của m®t bien co là m®t so đ c trưng cho
khả năng xảy ra bien co đó khi thực hi n m®t phép thả.
1.2.1 Định nghĩa co đien của xác suat
Định nghĩa 1.4. Xét phép thả T với không gian mau Ω là hǎu hạn. Bien co
A ⊂ Ω, khi đó tỉ so
P(A) =
|A|
|Ω|
được goi là xác suat của bien co A.
Nói m®t cách khác P là m®t hàm so xác định trên t p tat cả các t p con của
Ω, mà t p giá trị của P là [0,1] vì |A| ≤ |Ω| với moi A ⊂ Ω. Ta có m®t so tính chat
của xác suat như sau:
1) 0 ≤ P (A) ≤ 1, ∀ A⊂ Ω.
2) P(Ω) = 1.
3) P(∅) = 0.
Bài toán 1.1. [xem [5]]. Gieo đong thời hai đong xu cân đoi đong chat tìm xác
suat đe có bien co:
A: " Xuat hi n hai m¾t sap".
B: "M®t m¾t sap, m®t m¾t ngủa".
C: "Ít nhat m®t m¾t sap".
Lài giai.
Phép thả T là tung hai đong xu cân đoi đong chat.
Không gian mau: Ω = {SS, NN, NS, NN }, |Ω| = 4
A = {SS}, |A| = 1
B = {SN, NS}, |B| = 2
4
Viết đề tài giá sinh viên – ZALO:0973.287.149-TEAMLUANVAN.COM
6
C = {SS, SN, NS}, |C| = 3
nên P(A) =
1
= 0, 25; P(B) =
2
= 0, 5; P(C) =
3
= 0, 75.
4 4 4
Bài toán 1.2. [xem [7] trang 447]. Hi n nay có rat nhieu giải xő so trao giải
thướng lớn cho nhũng người chon đúng m®t b® sáu so trong so n so nguyên dương
đau tiên, trong đó n thường nam trong khoảng tù 30 đen 60. Tìm xác suat mà m®t
người chon đúng sáu so trong so 40 so?
Lài giai.
Chỉ có m®t b® đạt giải thưởng lớn. Tőng so cách đe chon sáu so trong so 40 là
40!
40 =
34!6!
= 3838380.
Do đó, khả năng chien thang là
1
3838380
≈ 0 . 00000026.
Bài toán 1.3. M®t h®p có a quả cau trang, b quả cau đen, lay ngȁu nhiên lan
lượt hai quả cau. Tìm xác suat đe bien co sau xảy ra
a) A : "Quả cau thú nhat là trang".
b) B : "Quả cau thú hai là trang biet quả cau thú nhat là trang".
Lài giai.
a) Ta có so cách lay lan lượt hai quả bóng là: (a + b)(a + b − 1) nên
|Ω| = (a + b)(a + b − 1).
So cách lay quả bóng đau tiên là trang, quả thá hai là tùy ý là a.(a + b − 1) nên
|A| = a(a + b − 1). V y
P(A) =
a(a + b − 1)
=
a
.
(a + b)(a + b − 1) a + b
b) Sau khi lan đau lay quả trang, so cách đe lan thá hai lay được quả trang là
(a − 1) nên |B| = a − 1. So cách đe lay được m®t quả tà a + b − 1 quả là a + b − 1,
tác |Ω| = a + b − 1 nên
P (B) =
a − 1
a + b − 1
Bài toán 1.4. Lay ngȁu nhiên ra 8 con bài tù b® tú lơ khơ 52 con. Tìm xác suat
của bien co sau
A : "Lay được 5 con màu đó".
B : "Lay được m®t con cơ, hai con rô, ba con bích".
C
5
Viết đề tài giá sinh viên – ZALO:0973.287.149-TEAMLUANVAN.COM
52 52
39
C : "Lay được m®t con át, hai con J, ba con 9, hai con 2".
D : "Lay được ba con cùng m®t chat đã chon trước".
Lài giai.
Đe lay 8 con tà 52 con tú có C8
(cách) nên |Ω| = C8
Ta can lay 5 con đỏ, 3 con đen, nên
3 5
|A| = C26C26 = 171028000.
Ta can lay 1 con cơ, 2 con rô, 3 con bích, 2 con tép, nên
5 3 3 2
|B| = C13.C26.C13.C13 = 22620312.
Ta can lay 1 con át, hai con J, ba con 9, hai con 2, nên
1 2 3 2
|C| = C4 .C4 .C4 .C4 = 576.
Ta can lay ba con cùng m®t chat và năm con thu®c ba chat khác nên
Ta có:
|D| = 13 .C5 = 286575757.
P(A) =
P (B) =
171028000
752538150
22620312
752538150
576
= 0, 227268;
= 0, 03006;
P (C) =
P(D) =
752538150
286575757
752538150
= 0, 000007654;
= 0, 2188148.
Bài toán 1.5. Có n người khách ra khói nhà mà không lay mũ của mình. Chủ
nhà không biet rõ chủ của các chiec mũ là ai nên gủi trả ho m®t cách ngȁu nhiên.
Tìm xác suat đe
a) Cả n người không nh¾n đúng mũ của mình.
b) Cả n người được trả đúng mũ.
c) Có k người (1 ≤ k ≤ n − 1) được trả đúng mũ.
Lài giai.
Ta có:|Ω| = n!
a) Goi bien co A : "Cả n người không nh n đúng mũ"
3
6
Viết đề tài giá sinh viên – ZALO:0973.287.149-TEAMLUANVAN.COM
| |
Σ
|C| = C D
;
n−k
n
≈
Khi đó:|A| = Dn
, nên P (A) =
Dn
n!
b) Goi bien co B : "Cả n người được trả đúng mũ"
Khi đó: B = 1, nên P (B) =
1
.
n!
c) Goi bien co C : "Có k người được trả đúng mũ"
Đe có k người được trả đúng mũ thì có đúng n − k người không được trả đúng
mũ, nên
n−1
k
n
k=1
n
Σ
−1
CkDn−k
P (C) = k=1 .
n!
1.2.2 Định nghĩa thong kê ve xác suat
Định nghĩa 1.5. Tan suat xuat hi n bien co trong n phép thả là t so giǎa so
phép thả trong đó bien co xuat hi n và tőng so phép thả được thực hi n.
Ta ký hi u so phép thả là n, so lan xuat hi n bien co A là k. Tan suat xuat hi n
bien co A là f(A) thì:
k
f(A) =
n
Định nghĩa 1.6. Xác suat xuat hi n bien co A trong m®t phép thả là m®t so p
không đői mà tan suat f xuat hi n bien co đó trong n phép thả sě dao đ®ng xung
quanh p, khi so phép thả của n tăng lên vô hạn thì P(A) ≈ f(A).
Bài toán 1.6. Có the xem xác suat sinh con trai là bao nhiêu khi theo dõi 88200
tré sơ sinh ớ m®t vùng có 45000 con trai.
Lài giai.
Ta có P(A) f(A) =
4500
88200
= 0, 51.
Tác xác suat sinh con trai xap xỉ 0,51. Hay tỉ l sinh con trai và con gái xap xỉ
là 51 nam, 50 nǎ.
1.3 Định lý c ng xác suat
Theo định nghĩa của bien co thì bien co là t p con của không gian mau Ω nên
trên các bien co cũng có các phép toán t p hợp, trên cơ sở đó ta cũng xây dựng
7
Viết đề tài giá sinh viên – ZALO:0973.287.149-TEAMLUANVAN.COM
m®t so công thác tính xác suat khác.
Định nghĩa 1.7. Bien co C là tőng của hai bien co A và B, kí hi u là A + B, neu
C chỉ xảy ra khi có ít nhat m®t trong hai bien co xảy ra.
Nói cách khác, ta viet C = A + B ⇔ C = A ∪ B.
Mở r®ng
n
C = A1 + A2 + ... + An ⇔ C =
i=
∪
1
Ai
Định nghĩa 1.8. Hai bien co A và B goi là xung khac với nhau neu chúng không
the đong thời xảy ra trong cùng m®t phép thả.
Ve m t t p hợp A và B xung khac tương đương A ∩ B = ∅.
Định nghĩa 1.9. Nhóm n bien co A1, A2, ..., An được goi là xung khac tàng đôi
m®t neu bat kì hai bien co này cùng xung khac nhau. Tác là
Ai ∩ Aj = ∅, ∀ i, j = 1, n.
Định lý 1.1 (Đ nh lý c®ng xác suat). Cho A và B là hai bien co xung khac, khi
đó
P(A + B) = P(A) + P (B).
ChGng minh. Ta có A + B = A ∪ B, giả sả |A| = m1, |B| = m2, tat cả các ket quả
của không gian mau là n.
Do |A ∪ B| = |A| + |B| − |A ∩ B|, mà |A ∩ B| = ∅ nên
|A ∪ B| = |A| + |B| = m1 + m2.
Suy ra:
P(A ∪ B) = (m1 + m2)/n;
P (A) + P (B) = m1/n + m2/n = (m1 + m2)/n.
V y ta có P(A + B) = P(A) + P(B).
H quả 1.1. Cho A1, A2, ..., An là nhóm các bien co đôi m®t xung khac, ta có:
P(A1 + A2 + ... + An) = P(A1) + P(A2) + ... + P (An).
Bài toán 1.7. Xác suat đe m®t xạ thủ ban trúng bia điem 10 là 0,1; trúng bia
điem 9 là 0,2; trúng bia điem 8 là 0,25 và ít hơn điem 8 là 0,45. Xạ thủ ay ban
8
Viết đề tài giá sinh viên – ZALO:0973.287.149-TEAMLUANVAN.COM
Σ
n−m ≤ n−m
−
− k
m®t viên đạn. Tìm xác suat đe xạ thủ được ít nhat 9 điem.
Lài giai.
Goi A1 là bien co "Xạ thủ ban trúng điem 10".
Goi A2 là bien co "Xạ thủ ban trúng điem 9".
Goi A là bien co "Xạ thủ ban được ít nhat 9 điem".
V y A = A1 + A2.
Vì A1, A2 xung khac nên ta có:
P(A1 + A2) = P(A1) + P(A2) = 0, 1 + 0, 2 = 0, 3.
Định nghĩa 1.10. Nhóm bien co A1, A2, ..., An được goi là m®t nhóm đay đủ các
bien co neu A1 + A2 + ... + An = Ω và Ai ∩ Aj = ∅, ∀ i, j = 1, n.
H quả 1.2. Neu A1, A2, ..., An tạo thành nhóm đay đủ các bien co thì
n
P
i=1
(Ai)
!
=
i=1
P(Ai) = 1
Định nghĩa 1.11. Hai bien co A và B được goi là đoi l p neu chúng tạo thành
m®t nhóm đay đủ các bien co, tác là A ∪ B = Ω và A ∩ B = ∅. Khi đó ta kí hi u
bien co đoi (đoi l p) của bien co A là A.
H quả 1.3. Neu A và A là hai bien co đoi l p, thì ta có P(A) = 1 − P(A).
Bài toán 1.8. Trong hòm có n sản phȁm trong đó có m chính phȁm m ≤ n. Lay
ngȁu nhiên k sản phȁm, tìm xác suat đe trong k sản phȁm có ít nhat m®t chính
phȁm (k ≤ n).
Lài giai.
Goi A là bien co "k sản phȁm lay ít nhat m®t chính phȁm", thì bien co đoi của A
là "k sản phȁm lay ra đeu là phe phȁm".
Do đó P (A) = 1 − P (A).
Mà |A| = Ck
C k
, k n m, nên P (A) = 1
n
Bài toán 1.9. Trong hòm có 10 chi tiet trong đó có 2 chi tiet hóng. Tìm xác suat
khi lay ra ngȁu nhiên 6 chi tiet thì không có quá m®t chi tiet hóng.
Lài giai.
Goi A là bien co "Trong 6 chi tiet lay ra không có chi tiet hỏng".
n
C
Σ
9
Viết đề tài giá sinh viên – ZALO:0973.287.149-TEAMLUANVAN.COM
C
Mà P (A0) = 8 =
C
; P(A1) = 2 8
i=1
Goi A1 là bien co "Trong 6 chi tiet lay ra có m®t chi tiet hỏng".
Goi A0 là bien co "Trong 6 chi tiet lay ra không có quá m®t chi tiet hỏng".
Khi đó A = A0 + A1, A0, A1 là hai bien co xung khac.
P(A) = P(A0 + A1) = P(A0) + P(A1).
C6 2
6
10
C 1C5 8
6 =
10
nên P (A) = 2/15 + 8/15 = 2/3.
1.4 Định lý nhân xác suat
Bây giờ ta xét trường hợp m®t bien co C là giao của hai bien co A, B.
Định nghĩa 1.12. Bien co C được goi là tích của hai bien co A và B, neu C xảy
ra khi và chỉ khi A và B cùng xảy ra.
Ve m t t p hợp: C = A.B ⇔ C = A ∩ B
Định nghĩa 1.13. Bien co A được goi là tích của n bien co A1, A2, ..., An neu A
Q
n
Định nghĩa 1.14. Hai bien co A, B được goi là đ®c l p với nhau neu vi c xảy ra
của bien co này không làm thay đői xác suat xảy ra bien co kia và ngược lại.
Hai bien co A, B không đ®c l p thì hai bien co đó goi là phụ thu®c nhau.
Ví dn 1.2. Trong bình có 3 quả cau trang, 2 quả cau đen. Lay ngau nhiên m®t
quả cau. Goi A là bien co "Lay được quả cau trang" thì P (A) = 3/5 . Quả cau
được bỏ lại vào bình và tiep tục lay ra m®t quả cau. Goi B là bien co "Lan thá
hai lay được quả cau trang" thì P (B) = 3/5, P (B) không phụ thu®c vào A nên A, B
đ®c l p. Tuy nhiên neu ta lay quả cau trang và không bỏ lại vào bình thì ta có
P(B) = 1/2. Khi đó A, B là phụ thu®c nhau.
Chú ý: Neu A và B là hai bien co đ®c l p thì A và B, A và B , A và B cũng đ®c
l p với nhau.
Định nghĩa 1.15. Các bien co A1, A2, ..., An goi là đ®c l p tàng đôi neu Ai, Aj đ®c
l p, i /= j, i, j = 1, n.
Định nghĩa 1.16. Các bien co A1, A2, ..., An goi là đ®c l p toàn phan với nhau
neu moi bien co đ®c l p với m®t tő hợp bat kì của các bien co còn lại.
xảy ra khi và chỉ khi tat cả n bien co A1, A2, ..., An xảy ra. Kí hi u: A = Ai.
5 15
10
Viết đề tài giá sinh viên – ZALO:0973.287.149-TEAMLUANVAN.COM
Q Q
Định lý 1.2 (Đ nh lý nhân xác suat). Cho A, B là hai bien co đ®c l p thì
P(A.B) = P(A).P (B).
Nh n xét 1.1. P(A.B) = P(A).P(B) ⇔ A và B đ®c l p.
H quả 1.4. Neu A, B đ®c l p thì
P(A) = P(A.B)/P(B), P(B) > 0.
H quả 1.5. Neu A1, A2, ..., An đ®c l p toàn phan thì P
n
Ai
i=1
n
=
i=1
P(Ai).
Bài toán 1.10. Trong m®t cu®c thi đau có A và B tham gia. Khả năng lot vào
chung ket của A là 90%, của B là 70%. A, B không cùng m®t bảng đau. Tìm xác
suat của các bien co
D: "Cả hai lot vào chung ket".
E: "Có ít nhat m®t người lot vào chung ket".
F: "Chí có A lot vào chung ket".
Lài giai.
Goi A là bien co "Người A lot vào chung ket".
Goi B là bien co "Người B lot vào chung ket".
Khi đó, de thay A, B là hai bien co đ®c l p, và D = A.B;
E = A.B + A.B + AB;
F = A.B.
Theo bài P (A) = 0, 9, P (B) = 0.7 nên
P(D) = P(A.B) = P(A)P(B) = 0, 9.0, 7 = 0, 63;
P(E) = P (A.B) + P(A.B) + P (A.B)
= 0, 7.0, 9 + 0, 7.0, 1 + 0, 9.0, 3 = 0, 97;
P(F ) = P(A.B) = P(A).P(B) = 0, 9.0, 3 = 0, 27.
Bây giờ ta xét trường hợp hai bien co A và B phụ thu®c nhau. Trước het ta xét
khái ni m xác suat có đieu ki n.
Định nghĩa 1.17. Xác suat của bien co A được tính với đieu ki n bien co B đã
xảy ra, goi là xác suat có đieu ki n của A và kí hi u là P(A/B)
11
Viết đề tài giá sinh viên – ZALO:0973.287.149-TEAMLUANVAN.COM
Định lý 1.3 (Xác suat có đieu ki n). Cho A, B là hai bien co phụ thu®c, khi đó:
P(A.B) = P (A).P (A/B) = P (B).P (B/A).
ChGng minh. Giả sả A, B là hai bien co của cùng không gian mau Ω và |Ω| = n,
|A| = m1.|B| = m2, |A.B| = k.
Khi đó P(A.B) = k/n, P(B/A) = k/m1, P(A/B) = k/m2. V y:
P (A.B) = k/n = (k/m1).(m1/n) = P (A)P (B/A).
P (A.B) = k/n = (k/m2).(m2/n) = P (B)P (A/B).
H quả 1.6. Neu P(B) > 0 thì P(A/B) = P(AB)/P(B).
H quả 1.7. Neu A1, A2, ..., An là n bien co phụ thu®c nhau thì
P (A1A2...An) = P (A1)P (A2/A1)P (A3/A1A2)...P (An/A1A2...An−1).
H quả 1.8. Hai bien co A và B đ®c l p khi và chỉ khi
P(A/B) = P (A), P (B/A) = P (B).
Bài toán 1.11. M®t cơ quan có ba xe ô tô, khả năng xảy ra sự co tương úng với
mői xe là 5%, 20%, 10%.Tìm khả năng xảy ra các tình huong sau:
- Cả ba ô tô b sự co.
- Có ít nhat m®t xe hoạt đ®ng tot.
- Có đúng m®t xe hoạt đ®ng tot.
- Cả ba xe hoạt đ®ng tot.
- Có không quá hai xe hoạt đ®ng tot.
Lài giai. Goi Ai là bien co "Xe thá i bị sự co", i = 1, 2, 3.
Ba bien co này không xung khac nhưng đ®c l p.
A là bien co "Cả ba ô tô cùng bị sự co".
B là bien co "Có ít nhat m®t xe hoạt đ®ng tot".
C là bien co "Có đúng m®t xe hoạt đ®ng tot".
D là bien co "Cả ba xe cùng hoạt đ®ng không tot".
E là bien co "Có không quá hai xe hoạt đ®ng không tot".
Khi đó theo giả thiet:
P (A1) = 0.05, nên P (A1) = 0, 95;
P (A2) = 0, 2, nên P (A2) = 0, 8;
12
Viết đề tài giá sinh viên – ZALO:0973.287.149-TEAMLUANVAN.COM
P (A3) = 0, 1, nên P (A3) = 0, 9.
Ta có the bieu dien A = A1A2A3 nên P (A) = P (A1A2A3) và
P (A1A2A3) = P (A1)P (A2)P (A3) = 0, 05.0, 2.0, 1 = 0, 001.
B = A1 + A2 + A3, xét B = A1A2A3 = A nên
P(B) = 1 − P (B) = 1 − P(A) = 1 − 0, 001 = 0, 999.
C = A1A2A3 + A1A2A3 + A1A2A3 nên
P (C) = 0, 05.0, 8.0, 1 + 0, 05.0, 2.0, 9 + 0, 95.0, 2.0, 1 = 0, 032.
D = A1A2A3 nên
P(D) = P(A1A2A3)
= P (A1).P (A2).P (A3)
= 0, 95.0, 8.0, 9 = 0, 684.
E: "Không quá hai xe bị sự co" là bien co "Có ít nhat m®t xe hoạt đ®ng tot",
tác E = B, nên P (E) = 0, 999.
Bài toán 1.12. Xét phép thủ tung m®t đong xu bon lan. Giả sủ ta không thay ket
quả nhưng ta biet có ít nhat hai m¾t sap. Tính xác suat tung được cả bon m¾t sap.
Lài giai.
Goi A là bien co "Tung được bon m t sap".
Goi B là bien co "Có ít nhat hai m t sap".
Do A ∩ B = A nên P (A/B) = P(A)/P(B).
Mà P (A) =
1 1 4 6 1 11
= ; P(B) = + + = .
24 16 16 16 16 16
Khi đó P (A) =
1
.
11
Bài toán 1.13. Cho P = P1P2...Pn là m®t hoán v ngȁu nhiên của n so tự nhiên
đau tiên. Goi A là bien co "P1 > P2", B là bien co "P2 > P3". Hói A và B có đ®c
l¾p không?
Lài giai.
Ta có với hai so P1, P2 cháa hai khả năng xảy ra P1 > P2 ho c P2 > P1 nên
P(A) = 1/2, tương tự P(B) = 1/2 và A ∩ B là bien co "P1 > P2 > P3",
P(A ∩ B) = 1/6.
13
Viết đề tài giá sinh viên – ZALO:0973.287.149-TEAMLUANVAN.COM
Vì với ba so P1, P2, P3 có 6 hoán vị, chỉ có m®t hoán vị thỏa mãn P1 > P2 > P3.
Khi đó:
P (A/B) = P (A ∩ B)/P (B) = (1/6)/(1/2) = 1/3
V y A, B không đ®c l p.
P (A).
M®t cách đơn giản hơn ta có ngay P (A ∩ B) = 1/6 =
/ P (A)P (B), nên A, B không
đ®c l p.
Bài toán 1.14. M®t người săn thó trong rùng. Khả năng anh ban trúng thó trong
mői lan ban tí l ngh ch với khoảng cách. Anh ta ban lan đau với khoảng cách 20m
với xác suat ban trúng là 50 %. Neu trượt anh ta ban tiep viên thú 2 ớ khoảng cách
30 m, neu trượt anh ta ban tiep viên thú 3 ớ khoảng cách 50 m. Tìm xác suat đe
người thợ săn ban được thó trong lan đi săn này.
Lài giai.
Goi Ti là bien co "Thợ săn ban trúng thỏ lan thá i”, i = 1, 2, 3, ba bien co này
không đ®c l p.
Theo bài ta có P (T1) = k/20 = 0, 5, Suy ra k = 10. Do đó:
P(T2/T1) = 10/30 = 1/3;
P (T 3/T2) = 10/50 = 1/5;
Goi T là bien co thợ săn ban trúng thỏ trong cu®c đi săn này
T = T 1 + T1.T 2 + T1.T2.T 3.
Khi đó:
P(T) = P(T1 + T1.T2 + T1.T2.T3)
= P(T1) + P(T1.T2 ) + P(T1.T2.T3)
= 0, 5 + P(T1).P(T2/T1) + P(T1).P(T2/T1).P(T3/T1T2)
= 0, 5 + (1 - 0, 5)(1/3) + (1 - 0, 5)(1 -1/3).0, 2 =22/30= 0, 733.
1.5 M t so m r ng của định lý c ng và định lý nhân xác suat
hai phan trước ta đã tìm hieu định lý c®ng xác suat với đieu ki n các bien co
xung khac, định lý nhân xác suat với đieu ki n các bien co đ®c l p. phan này
chúng ta sě xét m®t so công thác mở r®ng của hai định lý trên.
14
Viết đề tài giá sinh viên – ZALO:0973.287.149-TEAMLUANVAN.COM
Σ n
Σ
Σ Q
Σ Y
Y
Σ Q
Σ
Trước het trong định lý c®ng xác suat, neu A, B không xung khac ta có ket
quả sau:
Định lý 1.4. Cho A, B là hai bien co của cùng không gian mau Ω, khi đó
P(A + B) = P (A) + P(B) − P(A.B).
ChGng minh. Giả sả |Ω| = n, |A| = m1, |B| = m2, |A ∩ B| = k, k ≥ 0.
Do A, B không có đieu ki n xung khac. Khi đó ta có:
|A ∪ B| = |A| + |B| − |A ∩ B| = m1 + m2 − k.
Nên P (A ∪ B) = P (A + B) = (m1 + m2 − k)/n = m1/n + m2/n − k/n
= P(A) + P(B) − P(A.B).
Tương tự, áp dụng công thác tính so phan tả t p hợp ta cháng minh được:
H quả 1.9. Cho A1, A2, ..., An là các bien co của không gian mau Ω ta có
n
P
i=1
Ai
!
=
i=1
P (Ai) −
i<
Σ
j<1
P (Ai)P (Aj)+
+
i<j<1
P (AiAjAk) + ... + (−1)n−1P (A1A2...An)
Định lý 1.5. Cho A1, A2, ..., An là các bien co đ®c l p toàn phan với nhau, khi đó
ChGng minh.
n
n
P
i=1
n
Ai
!
= 1 −
n
P(Ai).
i=1
Goi A =
i=1
Ai thì A =
i=1
Ai do đó
Σ
n
!
Y
n
!
Do các Ai đ®c l p toàn phan nên Ai cũng đ®c l p toàn phan, nên ta có:
n
P
i=1
Ai
!
=
i=1
P(Ai).
Suy ra P
n
Ai
i=1
= 1 −
n
P (Ai)
i=1
Neu P (A1) = P (A2) = ... = P (An) = p thì ta có
n
P
i=1
Ai
!
= 1 − (1 − p)n
n
i=1
Ai
i=1
Ai
n
P(A) = P = 1 − P (A) = 1 − P
Σ
Y
15
Viết đề tài giá sinh viên – ZALO:0973.287.149-TEAMLUANVAN.COM
— −
Σ
− −
Σ
−
Σ
Bài toán 1.15. Xác suat đe đ®ng cơ thú nhat b trúng đạn là 0,2, đ®ng cơ thú hai
b trúng đạn là 0,3, xác suat phi công b trúng đạn là 0,1. Tìm xác suat đe máy
bay rơi, biet rang máy bay rơi khi cả hai đ®ng cơ đeu b trúng đạn ho¾c phi công
b trúng đạn.
Lài giai.
Goi Ai là bien co " đ®ng cơ thá i bị trúng đạn", i = 1, 2.
A3 là bien co "phi công bị trúng đạn".
A là bien co "máy bay rơi".
Khi đó A = A1A2 + A3, suy ra P (A) = P (A1A2 + A3). Do A1, A2, A3 không xung
khac nên
P (A) = P (A1A2) + P (A3) − P (A1A2A3).
M t khác A1, A2, A3 đ®c l p toàn phan nên ta có
P(A) = P(A1A2) + P(A3) − P(A1)P(A2)P(A3)= 0, 154.
Bài toán 1.16. M®t người viet n lá thư, bó ngȁu nhiên vào n phong bì có đe sȁn
đ a chí. Tìm xác suat có ít nhat m®t lá thư đúng đ a chí.
Lài giai.
Goi A là bien co "có ít nhat m®t lá thư đúng địa chỉ". Khi đó, A là bien co "tat
cả các lá thư không đúng địa chỉ".
So cách đe bỏ n lá thư vào n phong bì là n!.
So cách tat cả lá thư không đúng địa chỉ là:
n
Dn = (−1)
i−1 n!
, nên P
i! A =
Dn
. Suy ra:
n!
i=1
P(A) = 1 P(A) = 1
Dn
n!
n
= 1 ( 1)i−1
1
i!
i=1
n
= ( 1)i
1
i!
i=1
Bài toán 1.17. Phải tung m®t con súc sac bao nhiêu lan đe với xác suat không
nhó hơn 0,5 ta có hy vong rang có ít nhat m®t lan được m¾t sáu cham.
Lài giai.
Giả sả ta tung con súc sac n lan.
Goi Ai là bien co "tung lan i được m t sáu cham", i = 1, n .
Goi A là bien co "trong n lan có ít nhat m®t lan tung được m t sáu cham". V y
16
Viết đề tài giá sinh viên – ZALO:0973.287.149-TEAMLUANVAN.COM
Σ
Σ
Σ
Q
n
A = Ai , các bien co Ai là không xung khac và đ®c l p toàn phan với nhau nên
i=1 n
ta có: P(A) = 1 -
Ai .
i=1
Vì P(A1) = P(A2) = ... = P(An) = 1/6, nên
V y ta có P(A) = 1 -
P(A1) = P(A2) = ... = P(An) = 5/6
5 n
.
6
Theo giả thiet P (A) ≤ 0, 5 ⇔ 1 −
V y phải tung ít nhat bon lan.
5 n
≤ 0, 5 ⇔ n ≥ 3, 7, n ∈ N∗
.
Định lý 1.6 (Công thúc xác suat đay đủ). Cho H1, H2, ..., Hn là nhóm các bien co
đay đủ. A là m®t bien co của cùng m®t không gian mau Ω. Khi đó ta có
n
P(A) = P (Hi)P (A/Hi).
i=1
ChGng minh. Do H1, H2, ..., Hn là m®t nhóm các bien co đay đủ, nên
A = A ∩ Ω = A.H1 + A.H2 + ... + A.Hn.
Vì Hi ∩ Hj = ∅, ∀ i /= j nên A.Hi ∩A.Hj = ∅, ∀i /= j, i, j = 1, n . Do v y
n
P(A) = P (Hi)P (A/Hi).
i=1
Bài toán 1.18. Có ba h®p giong nhau, h®p thú nhat có 10 sản phȁm trong đó có
6 chính phȁm, h®p thú hai có 15 sản phȁm trong đó có 10 chính phȁm, h®p thú ba
có 20 sản phȁm trong đó có 15 chính phȁm. Lay ngȁu nhiên m®t h®p, tù h®p đó
lay ngȁu nhiên ra m®t sản phȁm. Tính xác suat đe lay được chính phȁm.
Lài giai.
Goi A là bien co "lay được chính phȁm".
Hi là bien co "chính phȁm lay ra thu®c h®p i".
Khi đó, A xảy ra đong thời với H1 ho c H2 ho c H3.
Xác suat đe lay được các h®p H1, H2, H3 là:
P(H1) = P(H2) = P(H3) = 1/3
6
17
Viết đề tài giá sinh viên – ZALO:0973.287.149-TEAMLUANVAN.COM
Σ
Σ
và P (A/H1) = 6/10, P (A/H2) = 10/15, P (A/H3) = 15/20.
Do đó:
P(A) = P (H1)P (A/H1) + P (H2)P (A/H2) + P (H3)P(A/H3)
1 6
. +
3 10
1 10
. +
3 15
1 15
. =
3 20
124 31
= .
180 45
Định lý 1.7. (Công thúc Bayes). Cho H1, H2, ..., Hn là m®t nhóm đay đủ các bien
co. A là m®t bien co của cùng không gian mau Ω. Khi đó
P(H /A) =
P (Hi) P (A/Hi)
.
i n
P (Hi) P (A/Hi)
i=1
ChGng minh. Theo định nghĩa xác suat có đieu ki n và công thác xác suat đay
đủ, ta có:
P (A.Hi) = P (A).P (Hi/A) = P (Hi).P (A/Hi), i = 1, n . Tà đó:
P(H /A) =
P (Hi) P (A/Hi)
=
P (Hi) P (A/Hi)
i P(A) n
P (Hi) P (A/Hi)
i=1
Bài toán 1.19. Dây chuyen lap ráp nh¾n được các chi tiet do hai máy sản xuat.
Trung bình máy thú nhat cung cap 60% chi tiet, máy thú hai cung cap 40% chi
tiet. Khoảng 90% chi tiet do máy m®t sản xuat là đạt chuȁn, 85% chi tiet do máy
thú hai sản xuat là đạt chuȁn. Lay ngȁu nhiên tù dây chuyen m®t sản phȁm, thay
sản phȁm đạt chuȁn. Tìm xác suat đe sản phȁm đó do máy thú nhat sản xuat.
Lài giai.
Goi A là bien co "chi tiet lay tà dây chuyen đạt chuȁn".
Hi là bien co "chi tiet do máy thá i sản xuat", i = 1, 2...
A xảy ra đong thời với H1, H2. Do đó xác suat can tìm là:
P(H1/A) =
P (H1) .P (A/H1)
P (H1) .P (A/H1) + P (H2) .P (A/H2)
Theo bài P(H1) = 0, 6; P (H2) = 0, 4; P (A/H1) = 0, 9; P (A/H2) = 0, 85 nên
0, 6.0, 9
P(H1/A) =
0, 6.0, 9 + 0, 4.0, 85
= 0, 614.
Bài toán 1.20. Trước khi đưa sản phȁm ra th trường người ta phóng van ngȁu
nhiên 200 khách hàng ve sản phȁm. Có 34 người trả lời "sẽ mua", 96 người trả lời
18
Viết đề tài giá sinh viên – ZALO:0973.287.149-TEAMLUANVAN.COM
n n
"có the sẽ mua", 70 người trả lời "không mua". Kinh nghi m cho thay ty l khách
hàng mua hàng tương úng với cách trả lời trên là 40%, 20%, 10%.
a) Tìm xác suat khách hàng mua sản phȁm.
b) Trong so khách hàng thực sự mua sản phȁm có bao nhiêu phan trăm trả lời
sẽ mua.
Lài giai.
Goi A là bien co "khách hàng mua sản phȁm".
H1 là bien co "người đó trả lời là sě mua".
H2 là bien co "người đó trả lời là có the sě mua".
H3 là bien co "người đó trả lời là không mua".
a) Theo công thác xác suat đay đủ
P(A) = P (H1)P (A/H1) + P (H2)P (A/H2) + P (H3)P(A/H3)
= (34/200).0, 4 + (96/200).0, 2 + (70/200).0, 01 = 0, 1675.
V y so người mua sản phȁm là 16,75
b) Theo công thác Bayes
P(H /A) =
P (H1) P (A/H1)
=
0, 17.0, 4
= 0, 40579.
1 P (A) 0, 1675
V y trong so khách hàng mua sản phȁm có xap xỉ 40% trả lời sě mua.
Trong hoạt đ®ng thực te, có rat nhieu trường hợp m®t phép thả được l p đi l p
lại. M®t dãy các phép thả được goi là đ®c l p với nhau neu xác suat xảy ra m®t
bien co nào đó trong tàng phép thả không phụ thu®c vào m®t bien co đó có xảy
ra ở các phép thả khác hay không. Chȁng hạn trong vi c sản xuat sản phȁm hay
tung nhieu lan các đong xu, con súc sac tạo nên m®t dãy các phép thả đ®c l p.
M®t dãy các phép thả đ®c l p thỏa mãn đieu ki n ket quả xảy ra chỉ có hai
trường hợp ho c bien co A xảy ra với xác suat p; ho c bien co A không xảy ra với
xác suat 1 − p = q, được goi là dãy phép thả Bernoulli.
Định lý 1.8. Trong m®t dãy n phép thả Bernoulli liên quan đen bien co A. Xác
suat đe bien co A xuat hi n đúng m lan là Pn(m) và
Pn(m) = Cmpmqn−m = Cmpm(1 − p)n−m
19
Viết đề tài giá sinh viên – ZALO:0973.287.149-TEAMLUANVAN.COM
n
Σ
C
100
10
n n
m
ChGng minh. Goi Ai là bien co "xảy ra bien co A ở lan thá i", i = 1, n. Khi đó
Ai là bien co "không xảy ra bien co A ở lan thá i".
B là bien co "trong n phép thả bien co A xảy ra đúng m lan". Khi đó bien co A
xảy ra m lan , A xảy ra n − m lan.
B = A1A2...AmAm+1...An + ... + A1 A2...An−mAn−m+1...An.
So cách chon m lan trong n lan xảy ra bien co A là Cm . Với moi cách chon m
lan đe bien co A xảy ra có n − m lan A xảy ra. Nên
Pn(m) = P (B) = pmqn−m + ... + pmqn−m = Cmpmqn−m = Cmpm(1 − p)n−m
Bài toán 1.21. M®t bài thi trac nghi m có 100 câu hói. Mői câu có bon phương
án trả lời, trong đó chí có m®t phương án đúng. Mői câu đúng được m®t điem còn
trả lời sai thì không được điem. M®t hoc sinh lười hoc đã chon ngȁu nhiên các
phương án trả lời. Tìm xác suat đe hoc sinh này
a) Đạt tù 50 điem trớ lên.
b) Đạt tù 49 điem trớ xuong.
Lài giai.
Goi A là bien co "chon câu trả lời đúng", thì ở moi lan chon P(A) = 1/4, nên
P (A) = 3/4.
a) Khi đó
P (m ≥ 50) = P100(50 ≤ m ≤ 100)
100
= 100(0, 25)m(0, 75)100−m.
m=50
b) P (m < 50) = P100(0 ≤ m ≤ 49) =
Σ
49
Cm (0, 25)m (0,75)
100−m
.
m=0
Bài toán 1.22. Theo ket quả đieu tra b nh lao, ty l người mac b nh lao ớ vùng
X là 0,001. Tìm xác suat đe khi khám cho 10 người.
a) Có hai người b lao.
b) Có ít nhat m®t người b lao.
Lài giai.
Goi A là bien co "g p người bị lao" thì P (A) = 0, 001, P (A) = 0, 999
P10(2) = C2 (0, 0001)2(0, 999)8
20
Viết đề tài giá sinh viên – ZALO:0973.287.149-TEAMLUANVAN.COM
Σ
P10(m ≥ 1) = 1 − P10(0) = 1 − (0, 999)10.
1.6 Bien ngȁu nhiên và kì vong
1.6.1 Định nghĩa
Định nghĩa 1.18. Cho Ω là t p hợp ket quả các phép thả nào đó. Bien ngau
nhiên X là hàm so được định nghĩa trên Ω.
Ví dn 1.3. Cho Ω là t¾p tat cả các đo th n đính, ta đ nh nghĩa bien ngȁu nhiên
X cho tương úng bới mői G ∈ Ω với X(G) là so cạnh của G. Ho¾c ta có the đ nh
nghĩa bien ngȁu nhiên Y cho tương úng với mői G ∈ Ω với Y (G) là thành phan
liên thông của G.
Định nghĩa 1.19. Cho X, Y là hai bien ngau nhiên trên Ω. Khi đó ta định nghĩa
tőng hai bien ngau nhiên X + Y cho tương áng với moi u ∈ Ω với
(X + Y )(u) = X(u) + Y (u)
và tích hai bien ngau nhiên X.Y cho tương áng moi u ∈ Ω với
X.Y (u) = X(u)Y (u).
Thông so đ c trưng quan trong nhat của bien ngau nhiên đó là kì vong của
bien ngau nhiên.
Định nghĩa 1.20. Cho X : Ω → R là m®t bien ngau nhiên sao cho t p giá trị của
X là S = {X(u), u ∈ Ω} hǎu hạn. Tác là X nh n hǎu hạn giá trị.
Khi đó so
E(X) = i.p(X = i)
i∈S
được goi là giá trị kỳ vong hay kì vong của X trên Ω.
Trong đó P (X = i) là xác suat của bien co X(u) = i.
P(X = i) =
|{u ∈ Ω, X (u) = i}|
|Ω|
Chú ý: Ta dùng kí hi u EΩ(X) đe kí hi u cho kì vong của bien ngau nhiên X được
định nghĩa trên không gian Ω. Ta có the viet là E(X) neu ta không can nhan mạnh
Ω. Kì vong chính là giá trị trung bình của các giá trị i.
21
Viết đề tài giá sinh viên – ZALO:0973.287.149-TEAMLUANVAN.COM
Σ
dq
i=1 i=1
Bài toán 1.23. Gieo đong thời hai con súc sac cân đoi đong chat. Goi X là tőng
so cham xuat hi n trên hai con súc sac. Tính E(X).
Lài giai.
Ta có Ω = {(i, j)}, 1 ≤ i, j ≤ 6. Bien ngau nhiên X : Ω → S cho tương áng u = (i, j),
với X(u) = i + j, nên S = {2..., 11, 12}. Ta l p được bảng
X 2 3 4 5 6 7 8 9 10 11 12
P(X)
1
36
2
36
3
36
4
36
5
36
1
36
5
36
4
36
3
36
2
36
1
36
Khi đó E(X) =
Σ
12
i.P(X = i)
i=2
= 2.1/36+ 3.2/36+ 4.3/36+ 5.4/36+ 6.5/36+ ... + 11.2/36+ 12.1/36= 7.
Bài toán 1.24. Xác suat đe m®t máy sản suat ra phe phȁm bang p. Máy sẽ được
sủa chũa ngay khi làm ra phe phȁm. Tìm so sản phȁm trung bình được sản suat
giũa hai lan sủa chũa.
Lài giai.
Goi X là so sản phȁm được sản xuat ra giǎa hai lan sảa chǎa. Khi đó X nh n giá
trị thu®c t p N∗ do xác suat sản xuat ra phe phȁm là p nên xác suat sản xuat ra
chính phȁm là q = 1 − p. Ta có the l p bảng
X 1 2 3 ... n ...
P p q.p q2.p ... qn−1.p ...
So sản phȁm trung bình được sản xuat ra chính là E(X). Ta có
∞ ∞
E(X) =
Σ
n.qn−1p = p
Σ
n.qn−1.
Do 0 < q < 1 nên theo công thác của cap so nhân lùi vô hạn
E(X) = p
Σ
∞
n.qn−1p = p.
d 1
d
= p. 1
= p
1
= p
1
=
1
dq 1 − q (1 − q)2 p2 p
Nh n xét 1.2. Ta có the mở r®ng t p S hǎu hạn thành t p S đem được neu tőng
∞ i.p(X = i) có giới hạn hǎu hạn.
i=1
1 − q
i=1
22
Viết đề tài giá sinh viên – ZALO:0973.287.149-TEAMLUANVAN.COM
i i
3
i=2
1.6.2 Tính tuyen tính của kì vong
Định nghĩa 1.21. Cho c là m®t hang so ta định nghĩa bien ngau nhiên cX cho
tương áng với moi x với so cX(u) = c(X(u)) với moi u ∈ Ω.
Với định nghĩa tőng, tích của hai bien ngau nhiên, tích của m®t so với bien
ngau nhiên chúng ta cháng minh được.
Định lý 1.9. Cho c là m®t so thực, X, Y là hai bien ngau nhiên trên không gian
Ω. Khi đó
E(X + Y ) = E(X) + E(Y ); E(c.X) = cE(X).
Nói m®t cách khác kì vong có tính chat tuyen tính.
Bài toán 1.25. Cho p = p1p2...pn là m®t hoán v của t¾p {1, 2, ..., n}, n ≥ 2. Khi
đó trung bình mői hoán v của 1, 2, ..., n sẽ có (n − 2)/3 so pi thóa mãn pi > pi − 1
và pi > pi+1 với i = 2, n − 1 .
Lài giai.
Goi Ω là t p tat cả các hoán vị của t p 1, 2, 3, ..., n. Trên Ω ta định nghĩa n − 2 bien
ngau nhiên Yi, i =2, n − 1 cho tương áng với p ∈ Ω.
Yi(p) = 1 neu p(i) thỏa mãn pi > pi − 1 và pi > pi + 1;
Y (p) = 0 neu pi không thỏa mãn đieu ki n trên.
Ta có với moi b® ba so pi−1, pi, pi+1 xác suat đe pi nhỏ nhat là 1/3, xác suat đe so
pi không nhỏ nhat là 2/3. Do đó
E(Yi) = 1/3.1 + 23.0 = 1/3
n
Σ
−1
so các pi thỏa mãn pi < pi−1 và pi > pi+1. Ta có
n−1 n−1
E(Y ) = E(
Σ
Y ) =
Σ
E(Y ) = (n − 2)E(Y ) =
n − 2
.
Bài toán 1.26. Cho p = p1p2...pn là m®t hoán v của t¾p {1, 2, ..., n}, i được goi
là điem co đ nh neu pi = i. Hói trung bình mői hoán v có bao nhiêu điem co đ nh.
Lài giai. Goi Ω là t p tat cả các hoán vị của 1, 2, ..., n.
Xét n bien ngau nhiên Xi trên Ω, cho tương áng moi p = p1, p2, ..., pn ∈ Ω với
Xi(p) = 1 neu pi = i, Xi(p) = 0 neu pi /= i.
i=2
i=2
Xét bien ngau nhiên Y = Yi . Khi đó với moi p ∈ Ω cho tương áng với Y (p) là
1
23
Viết đề tài giá sinh viên – ZALO:0973.287.149-TEAMLUANVAN.COM
Σ
Khi đó xác suat đe pi = i là 1/n, xác suat đe pi /= i là (n − 1)/n. Nên
E(X ) = 1/n +
n − 1
.0 = 1/n
Xét bien ngau nhiên X =
co định của p. Khi đó
i
n
i=1
n
n
Xi, cho tương áng p ∈ Ω với X(p) là so các điem
E(X) =
Σ
E(Xi) = nE(X1
1
) = n.
n
= 1.
i=1
V y trung bình moi hoán vị có m®t điem co định.
24
Viết đề tài giá sinh viên – ZALO:0973.287.149-TEAMLUANVAN.COM
Chương 2. Ứng dnng phương pháp
xác suat trong giải toán trung hoc
pho thông
Trong chương này trình bày vi c áp dụng tính chat đ c trưng xác suat, kỳ vong
đe giải toán đại so như cháng minh sự ton tại, cháng minh đȁng thác, bat đȁng
thác, tính toán tő hợp...Tài li u tham khảo chính trong phan này là các cuon
[1],[3],[9],[10].
2.1 Áp dnng xác suat và kì vong vào m t so bài toán thi hoc
sinh giỏi
Bài toán 2.1 (IMO 1970). Trên m¾t phȁng cho 100 điem, trong đó không có 3
điem nào thȁng hàng. Xét tat cả các tam giác có đính tại các điem đã cho. Chúng
minh rang không quá 70%các tam giác này là tam giác nhon.
Lài giai. Trước het ta cháng minh:"với 5 điem bat kì trên m t phȁng" trong đó
không có ba điem nào thȁng hàng thì có ít nhat ba tam giác với đỉnh lay tà năm
đỉnh trên không phải là tam giác nhon.
Th t v y ta xét 3 trường hợp
Trường hợp 1: Cho tam giác ABC và hai điem D, E nam trong ∆ABC, khi đó
có ít nhat hai trong ba tam giác ∆ADB, ∆ADC, ∆DBC tù tại D, ít nhat hai trong
ba tam giác ∆AEB, ∆AEC, ∆EBC tù tại E, do đó có ít nhat bon tam giác không
nhon trong tőng so các tam giác được tạo nên tà 5 điem A, B, C, D, E.
Trường hợp 2: Cho tá giác ABCD loi. Điem E nam trong tá giác ABCD. Trong
25
Viết đề tài giá sinh viên – ZALO:0973.287.149-TEAMLUANVAN.COM
100
97 100 97
n
100 97 100
Σ Σ
tá giác ABCD có ít nhat m®t đỉnh trong bon đỉnh A, B, C, D không nhon. Giả
sả là góc A thì có m®t tam giác không nhon là ABC. E nam trong tam giác ABC
ho c ∆BCD, khi đó nó sě tạo ra hai tam giác tù.
Trường hợp 3: Với ngũ giác loi ABCDE có ít nhat hai góc không nhon, giả sả là
hai góc A, B. Khi đó ∆ABC, ∆ABE là hai tam giác không nhon và tá giác BCDE
có ít nhat m®t góc không nhon, ta lại có thêm m®t tam giác không nhon.
Bây giờ ta thay so cách chon 5 điem trong 100 điem là C5 . Với moi cách lay
5 điem đó có ít nhat 3 tam giác không nhon. Nhưng moi đỉnh của tam giác được
đem C2 lan, do đó so tam giác không nhon ít nhat là 3.C5 C2 (với các đỉnh được
lay tà 100 điem trên). Khi đó xác suat đe có m®t tam giác nhon nhieu nhat là
1 − 3.C5 /(C2 C3 ) = 0, 7.
Tác ta có không quá 70% tam giác nhon.
Bài toán 2.2 (MOP test 2007). Trong bảng n×n mői m®t trong so các so 1, 2,
..., n xuat hi n đúng n lan. Chúng minh rang ton tại ít nhat m®t hàng ho¾c m®t
c®t với ít nhat
√
n so phân bi t.
Lài giai.
Ta sě đi cháng minh với m®t hàng hay m®t c®t bat kì, trung bình so phân tả khác
nhau là
√
n.
Th t v y so cách chon m®t hàng ho c m®t c®t trong bảng là 2n. Goi X là so
phan tả khác nhau trên m®t hàng ho c c®t đã chon Xi ∈ {0, 1}, Xi = 1 neu i có
m t trong hàng ho c c®t đã chon. Xi = 0 neu i không có m t trong hàng ho c c®t
đã chon.
n n
Khi đó X = Xi mà E(Xi) = Xi.p(Xi = 1) = p(Xi = 1).
i=1 i=1
M t khác so i xuat hi n trên ít hàng hay ít c®t nhat khi so i xuat hi n trong m®t
ô vuông nhỏ kích thước
√
n.
√
n nên:
2
√
n 1
E(Xi = 1) ≥
Do tính tuyen tính của kì vong:
2n
= √
n
E(X) =
Σ
i=1
1
E(Xi) ≥ n.E(Xi) = n.√
n
=
√
n.
26
Viết đề tài giá sinh viên – ZALO:0973.287.149-TEAMLUANVAN.COM
1 2 n
1 2 n
Mà E(X) là giá trị trung bình của so các phan tả khác nhau trên m®t hàng ho c
m®t c®t nên ton tại hàng ho c c®t sao cho so phan tả khác nhau lớn hơn
√
n, tác
ít nhat có
√
n phan tả khác nhau.
Bài toán 2.3 (Bulgaria MO 1984). Cho xi, yi (i = 1, 2, . . . , n) là 2n so thực dương
sao cho xi + yi = 1. Chúng minh rang
(1 − x1x2 . . . xn)m + (1 − ym)(1 − ym) . . . (1 − ym) ≥ 1
với moi so nguyên dương m và n.
Nh n xét 2.3. Có nhieu lời giải cho bài toán này, trong đó có cách giải thuan
túy đại so. Tuy nhiên, các cách giải đó đeu khá phác tạp. Lời giải dưới đây của
Pierre Bornsztein đăng trên website mathlinks.ro th t đep đě và thanh thoát.
Lài giai. Xét thí nghi m xác suat sau đây:
Xét c(1), c(2), . . . , c(n) là các đong xu sao cho với moi i, xác suat đe c(i) ra m t
ngảa là xi. Ta tung các xu này m®t cách đ®c l p m lan. Khi đó (1 − x1 . . . xn)m là
xác suat p(A) của bien co: "với moi m®t trong m lan tung, có ít nhat m®t đong xu
ra m t ngảa ".
Chú ý rang A = B ∪ C, trong đó B là "ton tại m®t đong xu ra m t ngảa ở moi
m®t trong m lan tung ", và C là "có ít nhat m®t đong xu ra m t ngảa ở moi lan
tung, nhưng đong xu này không giong nhau qua moi lan tung".
Hơn nǎa B ∩ C = ∅, do đó p(A) = p(B) + p(C).
M t khác, ta có (1 − ym)(1 − ym) . . . (1 − ym) là xác suat của bien co áng với moi
m®t đong xu ít nhat m®t lan không ra m t ngảa trong m lan tung bang p(B̄),
trong đó B̄ là bien co đoi l p với bien co B.
Như v y ve trái của bat đȁng thác đã cho là
p(A) + p(B̄) = p(B) + p(B̄) + p(C) = 1 + p(C) ≥ 1.
Chú ý rang đȁng thác xảy ra khi và chỉ khi n = 1.
Trong bài toán trên, bieu thác ve trái và đ c bi t là đieu ki n xi + yi = 1 gợi ý
ta đen ý tưởng sả dụng xác suat.
Xét bài toán sau đây tuy t nhiên không có ”dau hi u” của xác suat:
27
Viết đề tài giá sinh viên – ZALO:0973.287.149-TEAMLUANVAN.COM
Bài toán 2.4 (Putnam 2000). Cho aj, bj, cj, 1 ≤ j ≤ N là các so nguyên.
Giả sủ rang với mői j, trong ba so aj, bj, cj có ít nhat m®t so lé. Chúng minh
rang ton tại các so nguyên r, s, t sao cho raj
của j, 1 ≤ j ≤ N.
+ sbj + tcj
là lé với ít nhat
4N
7
giá tr
Lài giai. Cách 1. (Manjul Bhargava, Kiran Kedlaya và Lenny).
Xét 7 b® ba (a, b, c) với a, b, c ∈ {0, 1}, không phải tat cả đeu bang 0. Vì rang
rj, sj, tj không phải tat cả đeu chȁn, nên 4 trong các tőng arj + bsj + ctj với
a, b, c ∈ {0, 1} là chȁn và 4 là lẻ. Tat nhiên là tőng với a = b = c = 0 là chȁn,
do đó ít nhat 4 trong 7 tőng với a, b, c không đong thời bang 0 có tőng lẻ. Nói cách
khác, có ít nhat 4N trong các b® (a, b, c, j) cho tőng lẻ. Theo nguyên lý Dirichlet,
ton tại m®t b® (a, b, c) với ít nhat
4N
7
tőng là lẻ.
Cách 2. Bài toán trên đây được sả dụng lại tại kỳ thi Olympic Toán Singapore
năm 2012 (SMO 2012) và trên Mathlinks giới thi u lời giải xác suat thú vị sau.
Ta xét tat cả theo modul 2, vì trong bài ta chỉ quan tâm đen tính chȁn lẻ.
Ta có 7 cách chon cho b® (r, s, t) với r, s, t không đong thời bang 0; với moi b®
(a, b, c), có đúng 4 trong 7 b® sao cho ra + sb + tc ≡ 1.
Suy ra, với (ai, bi, ci) đã cho neu ta chon ngau nhiên (r, s, t) /= (0, 0, 0) thì giá trị
kỳ vong của so các bieu thác lẻ là
4N
. Nhưng neu đây là so trung bình thì phải
7
có ít nhat m®t b® (r, s, t) có so này lớn hơn hay bang
4N
.
7
Nh n xét 2.4. Lời giải trên đây đã sả dụng m®t nguyên lý rat đơn giản (giả thiet
Karramata): Neu trung bình của m®t so so là A thì sě có ít nhat m®t trong các so
đó ≥ A và ít nhat m®t trong các so đó ≤ A. Neu áp dụng vào xác suat thì giá trị
trung bình sě tương áng với giá trị kỳ vong. Đây là m®t nguyên lý rat hǎu hi u
mà ta sě còn nhac tới ở nhǎng phan sau.
Bài toán 2.5 (APMO 1998). Cho F là t¾p hợp tat cả các b® (A1, A2, . . . , An) trong
đó mői Ai, i = 1, 2, . . . , n là t¾p con của {1, 2, . . . , 1998}. Giả sủ |A| ký hi u so phan
tủ của t¾p hợp A, hãy tìm
(A1,A2
Σ
,...,An)∈F
|A1 ∪ A2 ∪ · · · ∪ An|.
Lài giai. Chú ý rang t p hợp {1, 2, . . . , 1998} có 21998 t p con vì ta có the chon
28
Viết đề tài giá sinh viên – ZALO:0973.287.149-TEAMLUANVAN.COM
Σ
C
v
C
799
399
x
hay không chon m®t phan tả vào t p con. Vì the có tat cả 21998n so hạng trong
tőng trên.
Bây giờ ta tính giá trị trung bình của moi so hạng. Với moi i = 1, 2, . . . , 1998. i
là phan tả của A1 ∪ A2 ∪ · · · ∪ An neu và chỉ neu nó là phan tả của ít nhat m®t
trong các A1, A2, . . . ,An.
Xác suat của bien co này là 1 − 2−n. Do đó, giá trị trung bình của moi so hạng
trong tőng là 1998(1 − 2−n), và như the đáp so là 21998n.1998(1 − 2−n).
Bő đe sau đây là chìa khóa cho nhieu bài toán giải bang phương pháp xác suat.
Bo đe 2.1. Cho X là bien ngau nhiên. Khi đó ton tại điem nào đó của không
gian xác suat mà X ≥ E[X], và ton tại điem bào đó của không gian xác suat mà
X ≤ E[X].
Bài toán 2.6 (Iran TST 2008). Giả sủ rang 799 đ®i bóng chuyen tham gia vào
m®t giải đau mà trong đó hai đ®i bat kỳ đau với nhau đúng m®t lan. Chúng minh
rang ton tại hai nhóm A và B rời nhau, mői nhóm có 7 đ®i sao cho mői đ®i bóng
của nhóm A đeu thua các đ®ng bóng của nhóm B.
Lài giai.
Xét giải đau như m®t đo thị có hướng đay đủ. Ta xét A là m®t t p ngau nhiên có
7 phan tả. Goi X là so đ®i thang tat cả các đ®i của A. Goi d(v−) là b c vào của
v, ta có
E(X) =
7
d(v−)
7 .
799
Nhưng
Σ
v d(v−) = C2 , nghĩa là b c trong trung bình của m®t đỉnh đúng bang
399. Theo tính loi của hàm C7, ta có
E(X) ≥
799C7
7
799
≈ 800.
1 7
≈ 6.25,
nhưng đieu này là đủ vì X là so nguyên. Chon 7 đ®i bóng của B tà nhóm đ®i
thang tat cả các đ®i của A ta có đieu can tìm.
Bài toán 2.7 (Russia MO 1996). Trong vi n Duma quoc gia có 1600 đại bieu,
l¾p thành 1600 tieu ban, mői tieu ban có 80 người. Chúng minh rang ta có the tìm
được hai tieu ban có ít nhat 4 thành viên chung.
2
C
29
Viết đề tài giá sinh viên – ZALO:0973.287.149-TEAMLUANVAN.COM
ni
.
Σ
E(X) ≥ n
C
= 800
Lài giai.
Chon ngau nhiên m®t c p tieu ban (tác là lay m®t cách ngau nhiên m®t c p trong
2
16000 c p). Goi X là so người có trong cả hai tieu ban được chon. Chú ý rang
X = X1 + · · · + X1600, trong đó moi Xi là bien ngau nhiên {0, 1} chỉ ra rang người
thá i có m t trong cả hai tieu ban hay không. Theo tính tuyen tính của kỳ vong,
ta có
E[X] = E[X1] + · · · + E[X1600].
Ta thay các E[Xi] có the tính de dàng.
Goi ni là so tieu ban mà người thá i thu®c vào tieu ban đó. Khi đó
E[Xi] = P[người thá i được chon vào cả hai tieu ban] =
C2
2
16000
Thông tin mà ta biet ve {ni} là tőng của chúng i ni = 16000.80.
Đieu này gợi cho ta sả dụng tính loi đe đánh giá E[X] thông qua giá trị trung bình
của {ni}, được ký hi u là n và bang n = (16000.80)/1600 = 800. Ta có
1600C2
2
16000
1600C2
2
16000
= 1600
800.799
16000.15999
= 3.995.
Theo Bő đe 2.1, ta biet rang sě có m®t ket quả nào đó cho ta X ≥ 3.995. Vì X
luôn là so nguyên, ket quả này thực sự phải có X ≥ 4.
Nói riêng, ta ket lu n rang có m®t c p hai tieu ban có ≥ 4 thành viên chung.
Bài toán 2.8 (MOP Test 2008). Giả sủ a, b, c là các so thực dương sao cho với
moi n nguyên
[an] + [bn] = [cn].
Chúng minh rang ít nhat m®t trong ba so a, b, c nguyên.
Nh n xét 2.5. Bạn có the sả dụng ket quả lý thuyet so quen thu®c sau đây: neu
x là so vô t thì phan phân của các b®i so của x phân bo đeu trên đoạn [0, 1].
Nói riêng, neu ta chon n m®t cách ngau nhiên trong {1, 2, . . . , N} thì E[xn] → 1/2
khi N → ∞.
Lài giai.
Giả sả rang không có so nào trong a, b, c là so nguyên. Chia hai ve cho n và cho n
dan đen vô cùng, ta được a + b = c.
C
C
C
30
Viết đề tài giá sinh viên – ZALO:0973.287.149-TEAMLUANVAN.COM
{ }
2
2100
2100
2q 2 2q 4 2
Tà đó, suy ra
{an} + {bn} = {cn}. (1)
Neu x vô t thì {xn} phân bo đeu trên đoạn [0, 1]. Nói riêng, neu ta chon n m®t
cách ngau nhiên trong {1, 2, . . . , N} thì E[{xn}] → 1/2 khi N → ∞.
M t khác, neu x là so hǎu t có dạng toi giản là p/q thì xn có kỳ vong tien
đen
q − 1
=
1
−
1
. Như v y nó nam trong khoảng
h1
,
1
.
Ket lu n: với so không nguyên x, E[{xn}] → t, trong đó t ∈
h1
,
1
i
.
Lay kỳ vong hai ve của (1), và cho n dan đen vô cùng, ta thay rang cách duy
nhat đe có đȁng thác là E[{an}] và E[{bn}] phải tien đen
1
, và
4
1
E[{cn}] →
2
.
Nhưng cách duy nhat đe có kỳ vong
1
4
là khi a, b hǎu t , còn cách duy nhat đe
có kỳ vong
1
là c vô t . Nhưng do a + b = c nên ta không the có hai so hǎu t c®ng
2
lại ra so vô t , mâu thuan.
Phương pháp xác suat có áng dụng hi u quả trong vi c cháng minh sự ton tại
của m®t cau trúc.
Ta biet rang trong 6 người bat kỳ ton tại 3 người đôi m®t quen nhau ho c ba
người đôi m®t không quen nhau. Khi 6 được thay bở 5 thì đieu này không còn
đúng nǎa và ta có the cháng tỏ đieu này bang cách chỉ ra phản ví dụ.
Khi các con so là lớn, vi c xây dựng phản ví dụ trở nên khó khăn. Trong nhǎng
trường hợp như the phương pháp xác suat tỏ ra hǎu dụng.
Bài toán 2.9. Chúng minh rang giũa 2100 người, không nhat thiet phải có 200
người đôi m®t quen nhau ho¾c 200 người đôi m®t không quen nhau.
Lài giai.
Ta sě cho m®t c p 2 người bat kỳ quen nhau ho c không quen nhau bang cách
tung m®t đong xu đoi xáng. Trong m®t nhóm 200 người, xác suat đe ho đôi m®t
quen nhau ho c đôi m®t không quen nhau là 2 × 2−C200
= 2−19899.
Vì có C200 cách chon ra 200 người, xác suat ton tại 200 người đôi m®t quen
nhau ho c đôi m®t không quen nhau nhieu nhat bang
(2100)
200
2101
C200
× 2−19899
<
200!
× 2−19899
=
200!
< 1.
Tà đây suy ra xác suat không ton tại 200 người đôi m®t quen nhau ho c đôi
m®t không quen nhau lớn hơn 0, tà đó suy ra đieu phải cháng minh.
4 2
31
Viết đề tài giá sinh viên – ZALO:0973.287.149-TEAMLUANVAN.COM
100 99
Ta thay ở đây m®t phương pháp tőng quát đe xây dựng ví dụ ngau nhiên: neu
xác suat của ton tại ví dụ ta can là dương thì ton tại ví dụ đó.
Bài toán 2.10. Trong mői ô của bảng 100×100, ta viet m®t trong các so nguyên
1, 2, . . . , 5000. Hơn nũa, mői m®t so nguyên xuat hi n trong bảng đúng 2 lan.
Chúng minh rang ta có the chon được 100 ô của bảng thóa mãn 3 đieu ki n sau:
(1) Mői m®t hàng được chon đúng m®t ô.
(2) Mői m®t c®t được chon đúng m®t ô.
(3) Các so trong các ô được chon đôi m®t khác nhau.
Lài giai.
Chon hoán vị ngau nhiên (a1, a2, . . . , a100) của {1, 2, . . . , 100} và chon ô thá ai trong
hàng thá i.
Cách chon như v y thỏa mãn (1) và (2).
Với moi j = 1, . . . , 5000, xác suat đe chon 2 ô có cùng so j là 0 neu hai ô này cùng
hàng ho c cùng c®t và là
1
×
1
trong trường hợp ngược lại.
Do đó xác suat đe cách chon này thỏa mãn (3) ít nhat là
1
1 − 5000 ×
100 × 999
> 0
và ta có đieu phải cháng minh.
Nh n xét 2.6. Ta có the de dàng chuyên hai lời giải xác suat nêu ở hai bài toán
trên sang lời giải chỉ sả dụng thuan túy phép đem (bang cách tính so các ket quả
thu n lợi thay vì tính xác suat), mà thực chat sě hoàn toàn giong. Nhưng lời giải
xác suat ngan gon hơn và tự nhiên hơn.
Nh n xét 2.7. M®t tính chat mang tính đ c trưng của xác suat là đȁng thác
P (A1) + P (A2) + · · · + P (An) = 1,
neu {A1, A2, . . . , An} là m®t phân hoạch của không gian xác suat Ω. Tính chat này
có the dùng đe cháng minh nhieu đȁng thác tő hợp bang phương pháp xác suat.
Bài toán 2.11. Cho p, q là các so thực dương sao cho p + q = 1. Chúng minh rang
p + pq + pq2 + pq3 + · · · = 1. (∗)
32
Viết đề tài giá sinh viên – ZALO:0973.287.149-TEAMLUANVAN.COM
P
Lài giai. Xét thí nghi m tung đong xu với xác suat ra m t ngảa là p và m t
xap là q. Ta thực hi n thí nghi m cho đen khi ra được m t ngảa. Goi X là so lan
tung, khi đó P(X = n) = pqn−1. Ve trái của đȁng thác (*) bang
P(X = 1) + P(X = 2) + · · · + P(X = n) + . . .
và dĩ nhiên là bang 1.
Bài toán 2.12 (IMO Shortlist 2006). Cho S là t¾p hũu hạn các điem trên m¾t
phȁng sao cho không có ba điem nào thȁng hàng. Với mői m®t đa giác loi P với
các điem thu®c S, goi a(P ) là so các điem của P và b(P ) là so các điem của S nam
ngoài P. Chúng minh rang với moi so thực x, ta có
Σ
xa(P )(1 − x)b(P ) = 1,
trong đó tőng được tính theo tat cả các đa giác loi có đính thu®c S.
Chú ý quan trong: đoạn thȁng, m®t điem và t p rong được coi là đa giác loi với
2, 1 và 0 đỉnh tương áng.
Lài giai.
Ta tô màu m®t cách ngau nhiên các điem bang màu đen và màu trang, trong
đó các điem được tô màu đen với xác suat x. Với moi đa giác loi P, goi EP là bien
co tat cả các đỉnh nam trên chu vi của P có màu đen và tat cả các đỉnh nam ngoài
P có màu trang. Các bien co này đôi m®t xung khac nhau, như the ve trái là xác
suat của sự ki n có m®t EP nào đó đúng. Nhưng đây là sự ki n chac chan xảy ra:
ta chỉ can xét bao loi của tat cả các điem màu đen!
Đe tính xác suat của m®t bien co theo định nghĩa cő đien ta thường phải giải
quyet hai bài toán tő hợp: tính so các ket quả thu n lợi và tính so các ket quả có
the. Thông thường, bài toán sau đơn giản hơn bài toán trước. Chính đieu này tạo
ra m®t tình huong áng dụng thú vị của xác suat: Neu ta tính được so ket quả có
the và tính được xác suat thì sě tính được so ket quả thu n lợi.
Bài toán 2.13. Trong so cách chon ra 3 đính tù 8 đính của hình l¾p phương đơn
v , có bao nhiêu cách chon thóa mãn đieu ki n: 3 đính được chon là đính của m®t
tam giác đeu.
Bài toán này không phải là khó, nhưng cũng khá roi. Ta giải bài này bang cách
tính xác xuat ba đỉnh được chon ngau nhiên tạo thành 3 đỉnh của 1 tam giác đeu.
33
Viết đề tài giá sinh viên – ZALO:0973.287.149-TEAMLUANVAN.COM
8
Ta lan lượt chon các đỉnh. Đỉnh đau tiên có the là 1 đỉnh bat kỳ. Với đỉnh
thá hai, khi đỉnh thá nhat đã được chon thì ta chỉ có the chon 1 trong 3 đỉnh có
khoảng cách
√
2 đen đỉnh đau. Xác suat thành công là 3/7. lượt cuoi, xác suat
thành công là 2/6. Như v y xác suat đe 3 đỉnh được chon là 3 đỉnh của 1 tam giác
đeu sě là 1/7. Vì so cách chon 3 đỉnh tà 8 đỉnh là C3 nên so cách chon thỏa mãn
đieu ki n 3 đỉnh được chon là đỉnh của m®t tam giác đeu sě bang
1
C3
=
8.7.6 = 8.
7 8 7.1.2.3
Các ví dụ trên đây cho thay phương pháp xác suat đôi khi mạnh hơn các phương
pháp truyen thong. Ta ket thúc phan này bang ví dụ sau, sả dụng m®t tính chat
mang tính hien nhiên của xác suat, đó là xác suat của m®t bien co luôn nam giǎa
0 và 1.
Bài toán 2.14. Trong m®t kỳ thi có n môn thi, trong đó có đe tieng Pháp và đe
tieng Anh. Thí sinh có the thi bao nhiêu môn tùy ý, nhưng thí sinh chí có the chon
m®t trong hai ngôn ngũ cho mői môn thi. Với hai môn thi bat kỳ, ton tại m®t thí
sinh thi hai môn này bang các ngôn ngũ khác nhau. Neu mői m®t môn có nhieu
nhat 10 thí sinh dự thi, hãy tìm giá tr lớn nhat có the của n.
Lài giai.
Đáp so là 1024. Ví dụ sau đây cho thay n = 1024 là có the. Giả sả có 10 thí sinh
(đánh so tà 1 đen 10) tham dự tat cả 1024 môn thi (đánh so tà 0 đen 1023). Với
thí sinh i, môn thi thá j sě được thi bang tieng Pháp neu chǎ so thá i tính tà
bên phải sang trong bieu dien nhị phân của j là 0 và sě thi bang tieng Anh trong
trường hợp ngược lại. Bang cách này de dàng kiem tra được đieu ki n được thỏa
mãn. (Ket quả cũng như ví dụ có the thu được không may khó khăn neu ta thay
10 bang 1 so nhỏ hơn và quan sát quy lu t.)
Đe cháng minh rang 1024 là so lớn nhat, ta gán ngau nhiên cho các thí sinh là
”người Pháp” ho c ”người Anh). Goi Ej là bien co ”moi thí sinh thi môn j đeu thi
bang đe đúng với quoc tịch mình được gán”.
Vì ‘với hai môn thi bat kỳ, ton tại m®t thí sinh thi hai môn này bang hai ngôn
ngǎ khác nhau’, nên không có hai Ej nào có the xảy ra đong thời. Tà đây suy ra:
P(ít nhat m®t trong các Ej xảy ra) = P(E1) + · · · + P(En) ≥ n/1024.
34
Viết đề tài giá sinh viên – ZALO:0973.287.149-TEAMLUANVAN.COM
≥
n
n
i
n
Σ
Σ
2
+ C2 −
≥
Nhưng vì xác suat của m®t bien co bat kỳ không vượt quá 1 nên tà đây ta suy
ra n ≤ 1024 , đó là đieu phải cháng minh.
2.2 M t so dạng toán thi Olympic liên quan
Bài 2.1 (IMO 1998). Trong m®t cu®c thi, có m thí sinh và n giám khảo, trong đó
n ≥ 3 là so nguyên lé. Mői m®t giám khảo sẽ đánh giá thí sinh đ¾u ho¾c rớt. Giả
sủ k là so sao cho với mői c¾p hai giám khảo đánh giá của ho trùng nhau ớ nhieu
nhat k thí sinh. Chúng minh rang:
k n − 1
m 2n
Lài giai.
Do n lẻ, đ t n = 2r + 1, tà n giám khảo có C2 c p giám khảo, mà 2 giám khảo có
ý kien trùng nhau nhieu nhat ở k thí sinh nên tőng so ý kien trùng nhau của các
giám khảo toi đa là C2.k.
M t khác thí sinh thá i được xi giám khảo đong ý, n−xi giám khảo không đong
ý. Khi đó so ý kien trùng nhau của giám khảo là:
xi n−xi
=
x2 + (n − xi)2 − n
2
r2 + (n − r)2 − n
2
(n 1)2
.
4
Nên tőng so ý kien trùng nhau ít nhat là
m(n − 1)2
4
, đieu đó dan đen:
k.C2 ≥
m(n − 1)2
4
nên
k n − 1
.
m 2n
Bài 2.2 (IMO 1987). Goi pn(k) là so các hoán v của t¾p {1, 2, ..., n} , n ≥ 1, có
đúng k điem bat đ®ng. Chúng minh rang:
n
k.pn(k) = n!
k=0
Lài giai. Đȁng thác can cháng minh tương đương với
n
k.pn(k)
= 1
n!
k=0
Goi Ω là t p tat cả các hoán vị của {1, 2, ..., n}, tác |Ω| = n!
P = p1p2...pn là m®t hoán vị của 1, 2, ..., n, X là so điem bat đ®ng của p ∈ Ω.
C ≥ =
35
Viết đề tài giá sinh viên – ZALO:0973.287.149-TEAMLUANVAN.COM
n
Σ
k=0
Ta có: E(X) = 1.
M t khác ta có xác suat đe P có k điem bat đ®ng là:
Ck.Dn−k
n!
Khi đó E(X) theo định nghĩa được tính:
n
=
pn(k)
.
n!
E(X) = k.
pn(k)
= 1
n!
k=0
Bài 2.3 (IMO Shortlist, 1999). Cho A là m®t t p gom N th ng dư modulo N 2.
Cháng minh rang ton tại t p B gom N th ng dư modulo N2 sao cho ít nhat m®t
nảa th ng dư modulo N2 có the viet dưới dạng a + b, a ∈ A, b ∈ B.
Bài 2.4 (IMO 1998). Trong m®t cu®c thi, có m thí sinh và n giám khảo, trong đó
n ≥ 3 là so nguyên lẻ. Moi m®t giám khảo sě đánh giá thí sinh đ u ho c rớt. Giả
sả k là so sao cho với moi c p hai giám khảo đánh giá của ho trùng nhau ở nhieu
nhat k thí sinh. Cháng minh rang
k
m
n − 1
2n
Bài 2.5 (IMO 1971). Cháng minh rang với moi so nguyên dương m, ton tại t p
hǎu hạn S các điem trên m t phȁng với tính chat sau: Với moi điem A trong S, có
đúng m điem của S có khoảng cách 1 đen A.
Bài 2.6 (China MO 1986). Cho z1, z2, . . . , zn là các so phác. Cháng minh rang ton
tại t p con S ⊆ {1, . . . , n} sao cho
.Σ . 1 Σ
.j∈S
zj
.
≥
π j=1
|zj|.
Bài 2.7 (IMO Shortlist 1987). Cháng minh rang ta có the tô màu các phan tả
của t p hợp {1, 2, . . . , 1987} bởi 4 màu sao cho moi cap so c®ng 10 phan tả của t p
hợp này đeu không đơn sac.
Bài 2.8 (Zarankiewicz). Cháng minh rang ton tại m®t cách chia t p hợp các so
nguyên dương thành hai t p con sao cho moi t p con đeu không cháa cap so c®ng
với vô so phan tả và không cháa ba so nguyên liên tiep.
Bài 2.9 (IMO 1987). Goi pn(k) là so các hoán vị của t p {1, . . . , n}, n ≥ 1, có đúng
Σ
n
≥
n
k điem bat đ®ng. Cháng minh rang kpn(k) = n!
36
Viết đề tài giá sinh viên – ZALO:0973.287.149-TEAMLUANVAN.COM
P
Bài 2.10 (Russia MO 1996). Trong vi n Duma quoc gia có 1600 đại bieu, l p
thành 16000 tieu ban, moi tieu ban có 80 người. Cháng minh rang ta có the tìm
được hai tieu ban có ít nhat 4 thành viên chung.
Bài 2.11 (IMO 1998). Trong m®t cu®c thi, có a thí sinh và b giám khảo, trong
đó b ≥ 3 là so nguyên lẻ. Moi m®t giám khảo sě đánh giá thí sinh ”đ u” ho c ”rớt”.
Giả sả k là so sao cho với moi c p hai giám khảo, đánh giá của ho trùng ở nhieu
nhat k thí sinh. Cháng minh rang k/a ≥ (b − 1)/(2b).
Bài 2.12 (APMO 1998). Cho F là t p hợp tat cả các b® (A1, A2, . . . , An) trong đó
moi Ai, i = 1, 2, . . . , n là t p con của {1, 2, . . . , 1998}. Giả sả |A| ký hi u so phan tả
của t p hợp A, hãy tìm
(A1,A2
Σ
,...,An)∈F
|A1 ∪ A2 ∪ · · · ∪ An|.
Bài 2.13 (USA TST 2001). Với t p hợp S, ký hi u | S | là so phan tả của S. Cho
A là t p hợp các so nguyên dương với |A| = 2001. Cháng minh rang ton tại t p B
sao cho
(i) B ⊆ A;
(ii) |B| ≥ 668;
(iii) với moi u, v ∈ B (không nhat thiet phân bi t), u + v ∈
/ B.
Bài 2.14 (Bay Area Math Olympiad 2004). Cho n so thực không đong thời bang
0 có tőng bang 0. Cháng minh rang ton tại m®t cách đánh so các so này là
a1, a2, . . . , an sao cho a1a2 + a2a3 + · · · + an−1an + ana1 < 0.
Bài 2.15 (IMO Shortlist 2006). Cho S là t p hǎu hạn các điem trên m t phȁng
sao cho không có ba điem nào thȁng hàng. Với moi m®t đa giác loi P với các điem
thu®c S, goi a(P ) là so các điem của P và b(P ) là so các điem của S nam ngoài P.
Cháng minh rang với moi so thực x, ta có
Σ
xa(P )(1 − x)b(P ) = 1,
trong đó tőng được tính theo tat cả các đa giác loi có đỉnh thu®c S.
Bài 2.16 (MOP Test 2007). Trong bảng n×n moi m®t trong các so 1, 2, . . . , n
xuat hi n đúng n lan. Cháng minh rang ton tại ít nhat m®t hàng ho c m®t c®t
với ít nhat so phân bi t.
37
Viết đề tài giá sinh viên – ZALO:0973.287.149-TEAMLUANVAN.COM
Bài 2.17 (Iran TST 2008). Giả sả rang 799 đ®i bóng chuyen tham gia vào m®t
giải đau mà trong đó hai đ®i bat kỳ đau với nhau đúng m®t lan. Cháng minh rang
ton tại hai nhóm A và B rời nhau, moi nhóm có 7 đ®i sao cho moi đ®i bóng của
nhóm A đeu thua các đ®ng bóng của nhóm B.
Bài 2.18 (MOP Test 2008). Giả sả a, b, c là các so thực dương sao cho với moi n
nguyên [an] + [bn] = [cn]. Cháng minh rang ít nhat m®t trong ba so a, b, c nguyên.
Bài 2.19. Cho X là bien ngau nhiên. Cháng minh rang ton tại điem của không
gian xác suat mà X ≥ E[X], và ton tại điem nào đó của không gian xác suat mà
X ≤ E[X].
Bài 2.20. Trong moi ô của bảng 100×100, ta viet m®t trong các so nguyên 1, 2,
. . . , 5000. Hơn nǎa, moi m®t so nguyên xuat hi n trong bảng đúng 2 lan. Cháng
minh rang ta có the chon được 100 ô của bảng thỏa mãn 3 đieu ki n sau:
(1) Moi m®t hàng được chon đúng m®t ô.
(2) Moi m®t c®t được chon đúng m®t ô.
(3) Các so trong các ô được chon đôi m®t khác nhau.
Bài 2.21. Trong so cách chon ra 3 đỉnh tà 8 đỉnh của hình l p phương đơn vị, có
bao nhiêu cách chon thỏa mãn đieu ki n: 3 đỉnh được chon là đỉnh của m®t tam
giác đeu.
Bài 2.22. Trong m®t kỳ thi có n môn thi, trong đó có đe tieng Pháp và đe tieng
Anh. Thí sinh có the thi bao nhiêu môn tùy ý, nhưng thí sinh chỉ có the chon m®t
trong hai ngôn ngǎ cho moi môn thi. Với hai môn thi bat kỳ, ton tại m®t thí sinh
thi hai môn này bang các ngôn ngǎ khác nhau. Neu moi m®t môn có nhieu nhat
10 thí sinh dự thi, hãy tìm giá trị lớn nhat có the của n.
Bài 2.23. Trong m®t giải cờ vua có 40 kỳ thủ. Có tőng c®ng 80 ván đau, và hai
kỳ thủ bat kỳ đau với nhau nhieu nhat m®t lan. Với m®t so nguyên n, cháng minh
rang ton tại n kỳ thủ chưa he đau với nhau (Tat nhiên là so n càng lớn càng tot.)
Bài 2.24. Cho p và q là các so không âm có tőng bang 1. Cháng minh rang
(1 − pm)n + (1 − qn)m ≥ 1.
38
Viết đề tài giá sinh viên – ZALO:0973.287.149-TEAMLUANVAN.COM
Σ
n
|
3
Σ
C ≡ 1 (mod p ).
p 2
Σ
Σ
x C ≡ 0 (mod p ).k
Σ
kC 2
Σ
(−1) C
k
p p+j
k
k k
j=0
Bài 2.25 (Hungarian MO 2001). Cho m và n là các so nguyên. Cháng minh rang
m là m®t ước của n
m−1
k=0
(−1) Ck.
Bài 2.26 (Chinese MO 1998). Xác định tat cả các so nguyên dương n ≥ 3 sao cho
22000 chia het cho 1 + C1 + C2 + C3.
n n n
Bài 2.27 (G p gơ Toán hoc 2014). Cho p ≥ 5 là so nguyên to và m, k ∈ Z+
. Cháng
minh rang
pk+2
Cp−1
mpk−1 — 1.
Bài 2.28 (Putnam 1996). Cho so nguyên to p ≥ 5 và k =
,2p
,
. Cháng minh rang
k
i 2
p
i=0
Bài 2.29 (Putnam 1991). Cho p là so nguyên to lẻ. Cháng minh rang
p
Σ
CjCj ≡ 2 + 1 (mod p ).
Bài 2.30 (Thi chon đ®i tuyen VMO 2012, tỉnh Ngh An). Cho p > 3 là so nguyên
to và t p hợp M = {1; 2; . . . ; p}. Với moi so nguyên k thỏa mãn 1 ≤ k ≤ p ta đ t
Ek = {A ⊂ M : |A| = k}, và xk =
A∈Ek
(min A + max A). Cháng minh rang
p−1
k 3
p
k=1
Bài 2.31 (VMO 2017). Cháng minh rang
1008
a) 2017 ≡ 0 (mod 2017 ).
k=1
504
b) 2017 ≡ 3 22016 − 1 (mod 20172).
k=1
Bài 2.32. Cho ā là b® (a1, a2, . . . , an) các so thực dương. Đ t P0 = 1,
Pk = Pk(ā), Er = Er(ā).
Cháng minh rang Pk−1Pk+1 ≤ P2 (k = 1, 2, . . . , n − 1) .
(Neu các ai đeu dương và không đong thời bang nhau thì ta có dau bat đȁng thác
thực sự).
k
39
Viết đề tài giá sinh viên – ZALO:0973.287.149-TEAMLUANVAN.COM
n 2
2 3 n
Bài 2.33. Cho các so ai > 0 (i ∈ {1, 2, 3, . . . , n}) và không đong thời bang nhau.
Cháng minh bat đȁng thác
1 1 1
P1 > P 2
P 3
> · · · > P 3
.
Bài 2.34. Cháng minh rang neu n và k là hai so tự nhiên thỏa mãn
0 ≤ k ≤ n thì
Bài 2.35. Cháng minh rang
n
2n+k
.Cn
2n−k
≤ (C2n) .
C2 + 2C3 + · · · + (n − 1)Cn > (n − 2)2n−1.
n n n
C1 + 2C2 + 3C3 + · · · + nCn
Bài 2.36. Cháng minh rang
với n ∈ N, n > 3.
n n n n
n
< n!,
C
40
Viết đề tài giá sinh viên – ZALO:0973.287.149-TEAMLUANVAN.COM
KET LU N
Lu n văn đã trình bày được nhǎng van đe sau:
1. Trình bày các bài toán cơ bản ve xác suat, khái ni m xác suat, tính chat,
quy tac tính, xác suat có đieu ki n, công thác Bernoulli, công thác Bayes ve xác
suat đay đủ. Các đại lượng đ c trưng của xác suat như kỳ vong, phương sai, bien
ngau nhiên rời rạc.
2. Trình bày các dạng toán thường g p ve xác suat và các áp dụng liên quan.
3. Trình bày được các bài toán đại so có the áng dụng xác suat đe giải.
4. Trình bày được các bài toán thi olympic có n®i dung xác suat.
Do thời gian thực hi n không nhieu và khả năng còn hạn che nên lu n văn mới
chỉ đưa ra được m®t so dạng toán ve xác suat rời rạc và áng dụng của nó trong
các bài toán thi hoc sinh giỏi.
Em xin chân thành cảm ơn!
41
Viết đề tài giá sinh viên – ZALO:0973.287.149-TEAMLUANVAN.COM
Tài li u tham khảo
A Tieng Vi t
[1] Nguyen Văn M u (2017), Tő hợp và các dạng toán liên quan, NXB Đại hoc
Quoc gia Hà N®i.
[2] Tạp chí TH&TT (2007), Các bài thi Olympic Toán trung hoc phő thông Vi t
Nam (1990-2006), NXB Giáo dục.
[3] Tran Nam Dũng, ”Phương pháp xác suat ”, bài viet đăng trên trang Thông
tin toán hoc, H®i Toán Hoc Vi t Nam tháng 12 năm 2012 t p 16 so 4, tháng
3 năm 2013 t p 17 so 1, tháng 6 năm 2013 t p 17 so 2.
[4] Đào Hǎu Ho (1996), ”Xác suat thong kê”, NXB Đại hoc Quoc gia Hà N®i,
trang 3 – 49.
[5] Đào Hǎu Ho (2011), ”Hướng dȁn giải bài toán xác suat thong kê”, NXB Đại
hoc Quoc gia Hà N®i, trang 3 – 52.
[6] Nguyen Văn Cao, Tran Thái Ninh (2006), "Bài t¾p xác suat và thong kê toán",
Đại hoc Kinh te Quoc dân, trang 5 – 74.
B Tieng Anh
[7] Kenneth H. Rosen (2012), Discrete Mathematics and its Applications, Seventh
Edition, Mc Graw Hill, pages 447, 449, 450.
[8] Paulo Ney de Sausa, Jorge- Nume Silva (1998), Berkeley Problems in
Mathematics, Springer.
[9] Titu Andreescu, Zuming Feng (2002), 102 combinatorial problems from the
training of the USA IMO team.
42
Viết đề tài giá sinh viên – ZALO:0973.287.149-TEAMLUANVAN.COM
[10] Dusan Djukic, Vladimir Jankovíc, Ivan Matíc, Nikola Petrovíc, ”The IMO
compendium”, Spinger, pages 338, 661, 667.

More Related Content

Similar to M t so dạng toán Liên quan đen xác suat r i rạc và ứng dụng.docx

Similar to M t so dạng toán Liên quan đen xác suat r i rạc và ứng dụng.docx (20)

Bài Toán Cực Trị Với Điều Kiện Ràng Buộc Bất Đẳng Thức, Hệ Bất Đẳng Thức.docx
Bài Toán Cực Trị Với Điều Kiện Ràng Buộc Bất Đẳng Thức, Hệ Bất Đẳng Thức.docxBài Toán Cực Trị Với Điều Kiện Ràng Buộc Bất Đẳng Thức, Hệ Bất Đẳng Thức.docx
Bài Toán Cực Trị Với Điều Kiện Ràng Buộc Bất Đẳng Thức, Hệ Bất Đẳng Thức.docx
 
Về hệ số nhị thức, hệ số đa thức Và một số bài toán liên quan.doc
Về hệ số nhị thức, hệ số đa thức Và một số bài toán liên quan.docVề hệ số nhị thức, hệ số đa thức Và một số bài toán liên quan.doc
Về hệ số nhị thức, hệ số đa thức Và một số bài toán liên quan.doc
 
Bat Phương Trình Hàm Sinh B I Các Đại Lư Ng Trung Bình B C Tùy Ý Và Các Dạng ...
Bat Phương Trình Hàm Sinh B I Các Đại Lư Ng Trung Bình B C Tùy Ý Và Các Dạng ...Bat Phương Trình Hàm Sinh B I Các Đại Lư Ng Trung Bình B C Tùy Ý Và Các Dạng ...
Bat Phương Trình Hàm Sinh B I Các Đại Lư Ng Trung Bình B C Tùy Ý Và Các Dạng ...
 
Bat đang thức trong so hoc và m t so Dạng toán liên quan.docx
Bat đang thức trong so hoc và m t so Dạng toán liên quan.docxBat đang thức trong so hoc và m t so Dạng toán liên quan.docx
Bat đang thức trong so hoc và m t so Dạng toán liên quan.docx
 
Phương Pháp Phương Trình Đại Số Chứng Minh Các Hệ Thức Lượng Giác.docx
Phương Pháp Phương Trình Đại Số Chứng Minh Các Hệ Thức Lượng Giác.docxPhương Pháp Phương Trình Đại Số Chứng Minh Các Hệ Thức Lượng Giác.docx
Phương Pháp Phương Trình Đại Số Chứng Minh Các Hệ Thức Lượng Giác.docx
 
Dãy Diatomic Của Stern.docx
Dãy Diatomic Của Stern.docxDãy Diatomic Của Stern.docx
Dãy Diatomic Của Stern.docx
 
Bat Đang Thức Và Bài Toán Cực Tr± Trong L P Các Đa Thức Và Phân Thức H So Ngu...
Bat Đang Thức Và Bài Toán Cực Tr± Trong L P Các Đa Thức Và Phân Thức H So Ngu...Bat Đang Thức Và Bài Toán Cực Tr± Trong L P Các Đa Thức Và Phân Thức H So Ngu...
Bat Đang Thức Và Bài Toán Cực Tr± Trong L P Các Đa Thức Và Phân Thức H So Ngu...
 
V N Dụng Chuői Đieu Hòa Vào Giải M T So Bài Toán Dành Cho Hoc Sinh Giỏi.docx
V N Dụng Chuői Đieu Hòa Vào Giải M T So Bài Toán Dành Cho Hoc Sinh Giỏi.docxV N Dụng Chuői Đieu Hòa Vào Giải M T So Bài Toán Dành Cho Hoc Sinh Giỏi.docx
V N Dụng Chuői Đieu Hòa Vào Giải M T So Bài Toán Dành Cho Hoc Sinh Giỏi.docx
 
Các ước so của so Mersenne.docx
Các ước so của so Mersenne.docxCác ước so của so Mersenne.docx
Các ước so của so Mersenne.docx
 
Hàm Đơn Đi U, Tựa Đơn Đi U Và M T So Ứng Dụng Của Phép Đơn Đi U Hóa Hàm So.docx
Hàm Đơn Đi U, Tựa Đơn Đi U Và M T So Ứng Dụng Của Phép Đơn Đi U Hóa Hàm So.docxHàm Đơn Đi U, Tựa Đơn Đi U Và M T So Ứng Dụng Của Phép Đơn Đi U Hóa Hàm So.docx
Hàm Đơn Đi U, Tựa Đơn Đi U Và M T So Ứng Dụng Của Phép Đơn Đi U Hóa Hàm So.docx
 
Tích Ngoài Của Véc Tơ Và Ứng Dụng.docx
Tích Ngoài Của Véc Tơ Và Ứng Dụng.docxTích Ngoài Của Véc Tơ Và Ứng Dụng.docx
Tích Ngoài Của Véc Tơ Và Ứng Dụng.docx
 
Đa Thức Trong Các Bài Toán Thi Học Sinh Giỏi.docx
Đa Thức Trong Các Bài Toán Thi Học Sinh Giỏi.docxĐa Thức Trong Các Bài Toán Thi Học Sinh Giỏi.docx
Đa Thức Trong Các Bài Toán Thi Học Sinh Giỏi.docx
 
Ứng Dụng Của Cap Và Chỉ So Cho So Nguyên Theo Modulo.docx
Ứng Dụng Của Cap Và Chỉ So Cho So Nguyên Theo Modulo.docxỨng Dụng Của Cap Và Chỉ So Cho So Nguyên Theo Modulo.docx
Ứng Dụng Của Cap Và Chỉ So Cho So Nguyên Theo Modulo.docx
 
Tổng quát về tích phân
Tổng quát về tích phân Tổng quát về tích phân
Tổng quát về tích phân
 
M T So Dạng Toán Ve Dãy So Sinh B I Các Hàm So Sơ Cap.docx
M T So Dạng Toán Ve Dãy So Sinh B I Các Hàm So Sơ Cap.docxM T So Dạng Toán Ve Dãy So Sinh B I Các Hàm So Sơ Cap.docx
M T So Dạng Toán Ve Dãy So Sinh B I Các Hàm So Sơ Cap.docx
 
Phương pháp chặn
Phương pháp chặnPhương pháp chặn
Phương pháp chặn
 
Về Đa Thức Bất Khả Quy Trên Trường Hữu Hạn.docx
Về Đa Thức Bất Khả Quy Trên Trường Hữu Hạn.docxVề Đa Thức Bất Khả Quy Trên Trường Hữu Hạn.docx
Về Đa Thức Bất Khả Quy Trên Trường Hữu Hạn.docx
 
Phương Trình Bậc Bốn Và Các Hệ Thức Hình Học Trong Tứ Giác Hai Tâm.docx
Phương Trình Bậc Bốn Và Các Hệ Thức Hình Học Trong Tứ Giác Hai Tâm.docxPhương Trình Bậc Bốn Và Các Hệ Thức Hình Học Trong Tứ Giác Hai Tâm.docx
Phương Trình Bậc Bốn Và Các Hệ Thức Hình Học Trong Tứ Giác Hai Tâm.docx
 
Giá Trị Trung Bình Với Hàm Tùy Ý Và Một Số Lớp Hàm Lồi Liên Quan.docx
Giá Trị Trung Bình Với Hàm Tùy Ý Và Một Số Lớp Hàm Lồi Liên Quan.docxGiá Trị Trung Bình Với Hàm Tùy Ý Và Một Số Lớp Hàm Lồi Liên Quan.docx
Giá Trị Trung Bình Với Hàm Tùy Ý Và Một Số Lớp Hàm Lồi Liên Quan.docx
 
Xap xỉ diophantine và phân so liên tục trong giải Phương trình pell.docx
Xap xỉ diophantine và phân so liên tục  trong giải Phương trình pell.docxXap xỉ diophantine và phân so liên tục  trong giải Phương trình pell.docx
Xap xỉ diophantine và phân so liên tục trong giải Phương trình pell.docx
 

More from DV Viết Luận văn luanvanmaster.com ZALO 0973287149

More from DV Viết Luận văn luanvanmaster.com ZALO 0973287149 (20)

Ảnh Hưởng Của Marketing Quan Hệ Đến Lòng Trung Thành Của Khách Hàng.Tình Huốn...
Ảnh Hưởng Của Marketing Quan Hệ Đến Lòng Trung Thành Của Khách Hàng.Tình Huốn...Ảnh Hưởng Của Marketing Quan Hệ Đến Lòng Trung Thành Của Khách Hàng.Tình Huốn...
Ảnh Hưởng Của Marketing Quan Hệ Đến Lòng Trung Thành Của Khách Hàng.Tình Huốn...
 
Phát triển nguồn nhân lực tại Uỷ ban nhân dân huyện Trà Bồng, tỉnh Quảng Ngãi...
Phát triển nguồn nhân lực tại Uỷ ban nhân dân huyện Trà Bồng, tỉnh Quảng Ngãi...Phát triển nguồn nhân lực tại Uỷ ban nhân dân huyện Trà Bồng, tỉnh Quảng Ngãi...
Phát triển nguồn nhân lực tại Uỷ ban nhân dân huyện Trà Bồng, tỉnh Quảng Ngãi...
 
Báo cáo tốt Nghiệp tài chính hợp nhất tại tổng công ty Indochina gol...
Báo cáo tốt Nghiệp  tài chính hợp nhất tại tổng công ty Indochina gol...Báo cáo tốt Nghiệp  tài chính hợp nhất tại tổng công ty Indochina gol...
Báo cáo tốt Nghiệp tài chính hợp nhất tại tổng công ty Indochina gol...
 
Tạo động lực thúc đẩy nhân viên làm việc tại ngân hàng TMCP Ngoại Thương Việt...
Tạo động lực thúc đẩy nhân viên làm việc tại ngân hàng TMCP Ngoại Thương Việt...Tạo động lực thúc đẩy nhân viên làm việc tại ngân hàng TMCP Ngoại Thương Việt...
Tạo động lực thúc đẩy nhân viên làm việc tại ngân hàng TMCP Ngoại Thương Việt...
 
Phát triển công nghiệp trên địa bàn Thành phố Tam Kỳ, Tỉnh Quảng Na...
Phát triển công nghiệp trên địa bàn Thành phố Tam Kỳ, Tỉnh Quảng Na...Phát triển công nghiệp trên địa bàn Thành phố Tam Kỳ, Tỉnh Quảng Na...
Phát triển công nghiệp trên địa bàn Thành phố Tam Kỳ, Tỉnh Quảng Na...
 
Giải pháp phát triển cho vay xuất nhập khẩu tại ngân hàng NN&PTNN ch...
Giải pháp phát triển cho vay xuất nhập khẩu tại ngân hàng NN&PTNN ch...Giải pháp phát triển cho vay xuất nhập khẩu tại ngân hàng NN&PTNN ch...
Giải pháp phát triển cho vay xuất nhập khẩu tại ngân hàng NN&PTNN ch...
 
Hoàn thiện công tác lập báo cáo tài chính hợp nhất tại tổng công ...
Hoàn thiện công tác lập báo cáo tài chính hợp nhất tại tổng công ...Hoàn thiện công tác lập báo cáo tài chính hợp nhất tại tổng công ...
Hoàn thiện công tác lập báo cáo tài chính hợp nhất tại tổng công ...
 
Luận Văn Thạc Sĩ Quản trị thành tích nhân viên tại Cục Hải quan TP Đà Nẵng.doc
Luận Văn Thạc Sĩ  Quản trị thành tích nhân viên tại Cục Hải quan TP Đà Nẵng.docLuận Văn Thạc Sĩ  Quản trị thành tích nhân viên tại Cục Hải quan TP Đà Nẵng.doc
Luận Văn Thạc Sĩ Quản trị thành tích nhân viên tại Cục Hải quan TP Đà Nẵng.doc
 
Hoàn thiện công tác quản lý thuế thu nhập cá nhân tại cục thuế Tỉ...
Hoàn thiện công tác quản lý thuế thu nhập cá nhân tại cục thuế Tỉ...Hoàn thiện công tác quản lý thuế thu nhập cá nhân tại cục thuế Tỉ...
Hoàn thiện công tác quản lý thuế thu nhập cá nhân tại cục thuế Tỉ...
 
Đề Tài Phát triển bền vững nông nghiệp Huyện Ba Tơ, Tỉnh Quảng Ngãi....
Đề Tài Phát triển bền vững nông nghiệp Huyện Ba Tơ, Tỉnh Quảng Ngãi....Đề Tài Phát triển bền vững nông nghiệp Huyện Ba Tơ, Tỉnh Quảng Ngãi....
Đề Tài Phát triển bền vững nông nghiệp Huyện Ba Tơ, Tỉnh Quảng Ngãi....
 
Hoàn thiện công tác bảo trợ xã hội trên địa bàn huyện Phong Điền, tỉnh Thừa T...
Hoàn thiện công tác bảo trợ xã hội trên địa bàn huyện Phong Điền, tỉnh Thừa T...Hoàn thiện công tác bảo trợ xã hội trên địa bàn huyện Phong Điền, tỉnh Thừa T...
Hoàn thiện công tác bảo trợ xã hội trên địa bàn huyện Phong Điền, tỉnh Thừa T...
 
Đề Tài Luận VănPhát triển sản phẩm du lịch tại thành phố Đà Nẵng.doc
Đề Tài Luận VănPhát triển sản phẩm du lịch tại thành phố Đà Nẵng.docĐề Tài Luận VănPhát triển sản phẩm du lịch tại thành phố Đà Nẵng.doc
Đề Tài Luận VănPhát triển sản phẩm du lịch tại thành phố Đà Nẵng.doc
 
Đào tạo nghề cho lao động thuộc diện thu hồi đất trên địa bàn Thàn...
Đào tạo nghề cho lao động thuộc diện thu hồi đất trên địa bàn Thàn...Đào tạo nghề cho lao động thuộc diện thu hồi đất trên địa bàn Thàn...
Đào tạo nghề cho lao động thuộc diện thu hồi đất trên địa bàn Thàn...
 
Tóm Tắt Luận Văn Thạc Sĩ Quản Trị Kinh Doanh Xây dựng chính sách Marketing tạ...
Tóm Tắt Luận Văn Thạc Sĩ Quản Trị Kinh Doanh Xây dựng chính sách Marketing tạ...Tóm Tắt Luận Văn Thạc Sĩ Quản Trị Kinh Doanh Xây dựng chính sách Marketing tạ...
Tóm Tắt Luận Văn Thạc Sĩ Quản Trị Kinh Doanh Xây dựng chính sách Marketing tạ...
 
Đề Tài Nghiên cứu rủi ro cảm nhận đối với mua hàng thời trang trực tuyến.docx
Đề Tài Nghiên cứu rủi ro cảm nhận đối với mua hàng thời trang trực tuyến.docxĐề Tài Nghiên cứu rủi ro cảm nhận đối với mua hàng thời trang trực tuyến.docx
Đề Tài Nghiên cứu rủi ro cảm nhận đối với mua hàng thời trang trực tuyến.docx
 
Giải pháp nâng cao động lực thúc đẩy người lao động tại công ty khai...
Giải pháp nâng cao động lực thúc đẩy người lao động tại công ty khai...Giải pháp nâng cao động lực thúc đẩy người lao động tại công ty khai...
Giải pháp nâng cao động lực thúc đẩy người lao động tại công ty khai...
 
Giải pháp phát triển dịch vụ ngân hàng điện tử tại ngân hàng đầu ...
Giải pháp phát triển dịch vụ ngân hàng điện tử tại ngân hàng đầu ...Giải pháp phát triển dịch vụ ngân hàng điện tử tại ngân hàng đầu ...
Giải pháp phát triển dịch vụ ngân hàng điện tử tại ngân hàng đầu ...
 
Giải pháp phát triển dịch vụ ngân hàng điện tử tại ngân hàng đầu ...
Giải pháp phát triển dịch vụ ngân hàng điện tử tại ngân hàng đầu ...Giải pháp phát triển dịch vụ ngân hàng điện tử tại ngân hàng đầu ...
Giải pháp phát triển dịch vụ ngân hàng điện tử tại ngân hàng đầu ...
 
Quản trị quan hệ khách hàng tại Chi nhánh Viettel Đà Nẵng – Tập đoàn Viễn thô...
Quản trị quan hệ khách hàng tại Chi nhánh Viettel Đà Nẵng – Tập đoàn Viễn thô...Quản trị quan hệ khách hàng tại Chi nhánh Viettel Đà Nẵng – Tập đoàn Viễn thô...
Quản trị quan hệ khách hàng tại Chi nhánh Viettel Đà Nẵng – Tập đoàn Viễn thô...
 
Đề Tài Đánh giá thành tích đội ngũ giảng viên trường Đại Học Phạm ...
Đề Tài Đánh giá thành tích đội ngũ giảng viên trường Đại Học Phạm ...Đề Tài Đánh giá thành tích đội ngũ giảng viên trường Đại Học Phạm ...
Đề Tài Đánh giá thành tích đội ngũ giảng viên trường Đại Học Phạm ...
 

Recently uploaded

Everybody Up 1 - Unit 5 - worksheet grade 1
Everybody Up 1 - Unit 5 - worksheet grade 1Everybody Up 1 - Unit 5 - worksheet grade 1
Everybody Up 1 - Unit 5 - worksheet grade 1
mskellyworkmail
 

Recently uploaded (20)

Nhân tố ảnh hưởng tới động lực làm việc của kiểm toán viên tại Chi nhánh Công...
Nhân tố ảnh hưởng tới động lực làm việc của kiểm toán viên tại Chi nhánh Công...Nhân tố ảnh hưởng tới động lực làm việc của kiểm toán viên tại Chi nhánh Công...
Nhân tố ảnh hưởng tới động lực làm việc của kiểm toán viên tại Chi nhánh Công...
 
Everybody Up 1 - Unit 5 - worksheet grade 1
Everybody Up 1 - Unit 5 - worksheet grade 1Everybody Up 1 - Unit 5 - worksheet grade 1
Everybody Up 1 - Unit 5 - worksheet grade 1
 
Báo cáo tốt nghiệp Đánh giá rủi ro quá trình xử lí nước cấp tại Chi nhánh Cấp...
Báo cáo tốt nghiệp Đánh giá rủi ro quá trình xử lí nước cấp tại Chi nhánh Cấp...Báo cáo tốt nghiệp Đánh giá rủi ro quá trình xử lí nước cấp tại Chi nhánh Cấp...
Báo cáo tốt nghiệp Đánh giá rủi ro quá trình xử lí nước cấp tại Chi nhánh Cấp...
 
Báo cáo tốt nghiệp Hoàn thiện công tác đào tạo và phát triển nguồn nhân lực c...
Báo cáo tốt nghiệp Hoàn thiện công tác đào tạo và phát triển nguồn nhân lực c...Báo cáo tốt nghiệp Hoàn thiện công tác đào tạo và phát triển nguồn nhân lực c...
Báo cáo tốt nghiệp Hoàn thiện công tác đào tạo và phát triển nguồn nhân lực c...
 
Báo cáo tốt nghiệp Phát triển sản phẩm thẻ tại Ngân hàng thương mại cổ phần K...
Báo cáo tốt nghiệp Phát triển sản phẩm thẻ tại Ngân hàng thương mại cổ phần K...Báo cáo tốt nghiệp Phát triển sản phẩm thẻ tại Ngân hàng thương mại cổ phần K...
Báo cáo tốt nghiệp Phát triển sản phẩm thẻ tại Ngân hàng thương mại cổ phần K...
 
Luận Văn: HOÀNG TỬ BÉ TỪ GÓC NHÌN CẢI BIÊN HỌC
Luận Văn: HOÀNG TỬ BÉ TỪ GÓC NHÌN CẢI BIÊN HỌCLuận Văn: HOÀNG TỬ BÉ TỪ GÓC NHÌN CẢI BIÊN HỌC
Luận Văn: HOÀNG TỬ BÉ TỪ GÓC NHÌN CẢI BIÊN HỌC
 
Báo cáo thực tập tốt nghiệp Phân tích thực trạng hoạt động bán hàng tại Công ...
Báo cáo thực tập tốt nghiệp Phân tích thực trạng hoạt động bán hàng tại Công ...Báo cáo thực tập tốt nghiệp Phân tích thực trạng hoạt động bán hàng tại Công ...
Báo cáo thực tập tốt nghiệp Phân tích thực trạng hoạt động bán hàng tại Công ...
 
Thực trạng ứng dụng công nghệ trong lĩnh vực giống cây trồng: Nghiên cứu điển...
Thực trạng ứng dụng công nghệ trong lĩnh vực giống cây trồng: Nghiên cứu điển...Thực trạng ứng dụng công nghệ trong lĩnh vực giống cây trồng: Nghiên cứu điển...
Thực trạng ứng dụng công nghệ trong lĩnh vực giống cây trồng: Nghiên cứu điển...
 
NHKTS SLIDE B2 KHAI NIEM FINTECH VA YEU TO CUNG CAU DOI MOI TRONG CN_GV HANG ...
NHKTS SLIDE B2 KHAI NIEM FINTECH VA YEU TO CUNG CAU DOI MOI TRONG CN_GV HANG ...NHKTS SLIDE B2 KHAI NIEM FINTECH VA YEU TO CUNG CAU DOI MOI TRONG CN_GV HANG ...
NHKTS SLIDE B2 KHAI NIEM FINTECH VA YEU TO CUNG CAU DOI MOI TRONG CN_GV HANG ...
 
Báo cáo tốt nghiệp Đánh giá thực trạng an toàn vệ sinh lao động và rủi ro lao...
Báo cáo tốt nghiệp Đánh giá thực trạng an toàn vệ sinh lao động và rủi ro lao...Báo cáo tốt nghiệp Đánh giá thực trạng an toàn vệ sinh lao động và rủi ro lao...
Báo cáo tốt nghiệp Đánh giá thực trạng an toàn vệ sinh lao động và rủi ro lao...
 
TỔNG HỢP HƠN 100 ĐỀ THI THỬ TỐT NGHIỆP THPT VẬT LÝ 2024 - TỪ CÁC TRƯỜNG, TRƯ...
TỔNG HỢP HƠN 100 ĐỀ THI THỬ TỐT NGHIỆP THPT VẬT LÝ 2024 - TỪ CÁC TRƯỜNG, TRƯ...TỔNG HỢP HƠN 100 ĐỀ THI THỬ TỐT NGHIỆP THPT VẬT LÝ 2024 - TỪ CÁC TRƯỜNG, TRƯ...
TỔNG HỢP HƠN 100 ĐỀ THI THỬ TỐT NGHIỆP THPT VẬT LÝ 2024 - TỪ CÁC TRƯỜNG, TRƯ...
 
Báo cáo tốt nghiệp Đánh giá rủi ro môi trường ô nhiễm hữu cơ trong nước thải ...
Báo cáo tốt nghiệp Đánh giá rủi ro môi trường ô nhiễm hữu cơ trong nước thải ...Báo cáo tốt nghiệp Đánh giá rủi ro môi trường ô nhiễm hữu cơ trong nước thải ...
Báo cáo tốt nghiệp Đánh giá rủi ro môi trường ô nhiễm hữu cơ trong nước thải ...
 
Báo cáo tốt nghiệp Đánh giá rủi ro môi trường từ ô nhiễm hữu cơ nước thải các...
Báo cáo tốt nghiệp Đánh giá rủi ro môi trường từ ô nhiễm hữu cơ nước thải các...Báo cáo tốt nghiệp Đánh giá rủi ro môi trường từ ô nhiễm hữu cơ nước thải các...
Báo cáo tốt nghiệp Đánh giá rủi ro môi trường từ ô nhiễm hữu cơ nước thải các...
 
Tiểu luận triết học_Nguyễn Gia Nghi_QHCCCLC_11230120.pdf
Tiểu luận triết học_Nguyễn Gia Nghi_QHCCCLC_11230120.pdfTiểu luận triết học_Nguyễn Gia Nghi_QHCCCLC_11230120.pdf
Tiểu luận triết học_Nguyễn Gia Nghi_QHCCCLC_11230120.pdf
 
CHỦ ĐỀ VÀ TÍNH LIÊN KẾT TRONG DIỄN NGÔN CHÍNH LUẬN TIẾNG VIỆT
CHỦ ĐỀ VÀ TÍNH LIÊN KẾT TRONG DIỄN NGÔN CHÍNH LUẬN TIẾNG VIỆTCHỦ ĐỀ VÀ TÍNH LIÊN KẾT TRONG DIỄN NGÔN CHÍNH LUẬN TIẾNG VIỆT
CHỦ ĐỀ VÀ TÍNH LIÊN KẾT TRONG DIỄN NGÔN CHÍNH LUẬN TIẾNG VIỆT
 
TỔNG HỢP HƠN 100 ĐỀ THI THỬ TỐT NGHIỆP THPT TOÁN 2024 - TỪ CÁC TRƯỜNG, TRƯỜNG...
TỔNG HỢP HƠN 100 ĐỀ THI THỬ TỐT NGHIỆP THPT TOÁN 2024 - TỪ CÁC TRƯỜNG, TRƯỜNG...TỔNG HỢP HƠN 100 ĐỀ THI THỬ TỐT NGHIỆP THPT TOÁN 2024 - TỪ CÁC TRƯỜNG, TRƯỜNG...
TỔNG HỢP HƠN 100 ĐỀ THI THỬ TỐT NGHIỆP THPT TOÁN 2024 - TỪ CÁC TRƯỜNG, TRƯỜNG...
 
Hệ thống ca dao than thân người Việt từ góc nhìn thi pháp
Hệ thống ca dao than thân người Việt từ góc nhìn thi phápHệ thống ca dao than thân người Việt từ góc nhìn thi pháp
Hệ thống ca dao than thân người Việt từ góc nhìn thi pháp
 
TỔNG HỢP HƠN 100 ĐỀ THI THỬ TỐT NGHIỆP THPT HÓA HỌC 2024 - TỪ CÁC TRƯỜNG, TRƯ...
TỔNG HỢP HƠN 100 ĐỀ THI THỬ TỐT NGHIỆP THPT HÓA HỌC 2024 - TỪ CÁC TRƯỜNG, TRƯ...TỔNG HỢP HƠN 100 ĐỀ THI THỬ TỐT NGHIỆP THPT HÓA HỌC 2024 - TỪ CÁC TRƯỜNG, TRƯ...
TỔNG HỢP HƠN 100 ĐỀ THI THỬ TỐT NGHIỆP THPT HÓA HỌC 2024 - TỪ CÁC TRƯỜNG, TRƯ...
 
Tiểu luận tổng quan về Mối quan hệ giữa chu kỳ kinh tế và đầu tư trong nền ki...
Tiểu luận tổng quan về Mối quan hệ giữa chu kỳ kinh tế và đầu tư trong nền ki...Tiểu luận tổng quan về Mối quan hệ giữa chu kỳ kinh tế và đầu tư trong nền ki...
Tiểu luận tổng quan về Mối quan hệ giữa chu kỳ kinh tế và đầu tư trong nền ki...
 
Quản lý dạy học phân hóa môn Toán tại các trường trung học cơ sở huyện Tam D...
Quản lý dạy học phân hóa môn Toán tại các trường trung học cơ sở huyện Tam D...Quản lý dạy học phân hóa môn Toán tại các trường trung học cơ sở huyện Tam D...
Quản lý dạy học phân hóa môn Toán tại các trường trung học cơ sở huyện Tam D...
 

M t so dạng toán Liên quan đen xác suat r i rạc và ứng dụng.docx

  • 1. ĐẠI HOC THÁI NGUYÊN TRƯ NG ĐẠI HOC KHOA HOC Tải tài liệu tại sividoc.com Viết đề tài giá sinh viên – ZALO:0973.287.149-TEAMLUANVAN.COM VŨ DUY ĐẠT M T SO DẠNG TOÁN LIÊN QUAN ĐEN XÁC SUAT R I RẠC VÀ ỨNG DỤNG Chuyên ngành: PHƯƠNG PHÁP TOÁN SƠ CAP Mã so: 8 46 01 13 LU N VĂN THẠC SĨ TOÁN HOC Người hướng dan khoa hoc: GS.TSKH. Nguyen Văn M u THÁI NGUYÊN - 2019
  • 2. ĐẠI HOC THÁI NGUYÊN TRƯ NG ĐẠI HOC KHOA HOC Tải tài liệu tại sividoc.com Viết đề tài giá sinh viên – ZALO:0973.287.149-TEAMLUANVAN.COM VŨ DUY ĐẠT M T SO DẠNG TOÁN LIÊN QUAN ĐEN XÁC SUAT R I RẠC VÀ ỨNG DỤNG Chuyên ngành: PHƯƠNG PHÁP TOÁN SƠ CAP Mã so: 8 46 01 13 LU N VĂN THẠC SĨ TOÁN HOC Người hướng dan khoa hoc: GS.TSKH. Nguyen Văn M u THÁI NGUYÊN - 2019
  • 3. i Viết đề tài giá sinh viên – ZALO:0973.287.149-TEAMLUANVAN.COM Mnc lnc L I CẢM ƠN ii M ĐAU 1 Chương 1. M t so dạng toán liên quan đen xác suat r i rạc 2 1.1 Phép thả và bien co....................................................................................2 1.2 Xác suat của bien co...................................................................................3 1.2.1 Định nghĩa cő đien của xác suat....................................................3 1.2.2 Định nghĩa thong kê ve xác suat....................................................6 1.3 Định lý c®ng xác suat ............................................................................. 6 1.4 Định lý nhân xác suat............................................................................. 9 1.5 M®t so mở r®ng của định lý c®ng và định lý nhân xác suat ..................13 1.6 Bien ngau nhiên và kì vong ......................................................................20 1.6.1 Định nghĩa.....................................................................................20 1.6.2 Tính tuyen tính của kì vong.........................................................22 Chương 2. Ứng dnng phương pháp xác suat trong giải toán trung hoc pho thông 24 2.1 Áp dụng xác suat và kì vong vào m®t so bài toán thi hoc sinh giỏi . 24 2.2 M®t so dạng toán thi Olympic liên quan...................................................34 KET LU N 40 TÀI LI U THAM KHẢO 41
  • 4. ii Viết đề tài giá sinh viên – ZALO:0973.287.149-TEAMLUANVAN.COM L i cảm ơn Lu n văn này được hoàn thành với sự hướng dan t n tình của GS.TSKH Nguyen Văn M u - Đại hoc Khoa hoc tự nhiên - Đại hoc Quoc gia Hà N®i. Tự đáy lòng mình, em xin tỏ lòng biet ơn sâu sac tới Thay đoi với sự quan tâm, chỉ bảo t n tình của Thay. Em xin chân thành cảm ơn các thay cô trong Trường Đạị hoc Khoa hoc - Đại hoc Thái Nguyên, đã giúp đơ em trong suot quá trình theo hoc. Tôi cũng xin chân thành cảm ơn Ban Giám hi u, các đong nghi p Trường THPT Đông Thành - Quảng Ninh và gia đình đã tạo đieu ki n cho tôi hoàn thành ke hoạch hoc t p. M c dù có nhieu co gang nhưng lu n văn khó tránh khỏi nhǎng thieu sót và hạn che. Tác giả mong nh n được nhǎng ý kien đóng góp của các thay cô và các bạn đoc đe lu n văn được hoàn thi n hơn. Xin chân thành cảm ơn! Thái Nguyên, tháng 5 năm 2019 Tác giả Vũ Duy Đạt
  • 5. 1 Viết đề tài giá sinh viên – ZALO:0973.287.149-TEAMLUANVAN.COM M ĐAU Lu n văn nham cung cap các dạng toán xác suat rời rạc và các bài toán áng dụng phương pháp xác suat trong giải toán trung hoc phő thông. Chuyên đe nam trong chương trình phục vụ cải cách giáo dục và boi dương hoc sinh giỏi ở các lớp chuyên toán phục vụ kỳ thi hoc sinh giỏi quoc gia, Olympic khu vực và quoc te. Trong chương trình cải cách giáo dục hi n nay, các van đe liên quan đen xác suat và thong kê đã được phê duy t đe giảng dạy chính thác trong các trường trung hoc phő thông. nhieu nước trên the giới, trong các kì thi hoc sinh giỏi toán các cap, Olympic Toán khu vực và quoc te có nhieu đe toán liên quan tới lý thuyet và phương pháp xác suat, thong kê. Nhǎng dạng toán này thường được xem là thu®c loại khó vì phan kien thác nâng cao ve chuyên đe này hi n nay không nam trong chương trình chính thong của sách giáo khoa hi n hành b c trung hoc phő thông. Đe đáp áng nhu cau boi dương giáo viên và boi dương hoc sinh giỏi ve chuyên đe xác suat và phương pháp xác suat, tôi chon đe tài lu n văn ”M®t so dạng toán liên quan đen xác suat rời rạc và áng dụng”. Trong đó, khảo sát m®t so lớp bài toán tà các đe thi hoc sinh giỏi Quoc gia và Olympic các nước nhǎng năm gan đây ve chuyên đe này. Cau trúc lu n văn gom 2 chương: Chương 1. M®t so dạng toán liên quan đen xác suat rời rạc. Chương 2. Úng dụng phương pháp xác suat trong giải toán trung hoc phő thông. Thái Nguyên, tháng 5 năm 2019 Tác giả
  • 6. 2 Viết đề tài giá sinh viên – ZALO:0973.287.149-TEAMLUANVAN.COM Chương 1. M t so dạng toán liên quan đen xác suat r i rạc Trong chương này trình bày cơ sở lý thuyet cùng các bài toán áp dụng nâng cao ve phép tính xác suat trong đại so, áp dụng nhǎng quy tac tőng quát như quy tac c®ng, quy tac nhân, công thác Bernoulli cho phép thả l p, công thác xác suat đay đủ, công thác Bayes vào các ví dụ thực te. Nghiên cáu tính toán đại lượng đ c trưng bien ngau nhiên rời rạc như kỳ vong, phương sai... trong thong kê. Tài li u tham khảo chính trong phan này là các cuon [4], [5], [6], [7]. 1.1 Phép thfi và bien co Định nghĩa 1.1. Vi c thực hi n m®t nhóm các đieu ki n cơ bản đe quan sát m®t hi n tượng nào đó có xảy ra hay không được goi là thực hi n m®t phép thả. Ví dn 1.1. Thực hi n tung m®t đong xu bon lan là m®t phép thả nham quan sát trong bon lan tung đong xu ta nh n được m t ngảa hay m t sap. Khi đó tat cả các ket quả có the xảy ra sau 4 lan tung là 24 = 16. Khả năng xảy ra m®t trong các ket quả là 1 . 16 Định nghĩa 1.2 (xem [4]). Không gian mau Ω của phép thả T là t p hợp tat cả các ket quả của m®t phép thả T sao cho các ket quả có khả năng xảy ra như nhau. T p A ⊂ Ω thì A được goi là bien co ngau nhiên. Khi A = Ω thì A được goi là bien co chac chan (chac chan xảy ra). Khi A = ∅ thì A được goi là bien co không (không xảy ra).
  • 7. 3 Viết đề tài giá sinh viên – ZALO:0973.287.149-TEAMLUANVAN.COM 1.2 Xác suat của bien co Trở lại với ví dụ ở trên ta xét bien co A là "ket quả bon lan tung có ít nhat ba lan xuat hi n m t ngảa". Ket quả của Ω làm A xuat hi n là: NNNN, NSNN, NNSN, NNNS, SNNN. Khi đó năm ket quả này được goi là ket quả có lợi cho bien co A, khả năng xảy ra bien co A là 5 . 16 Định nghĩa 1.3 (xem [4]). Xác suat của m®t bien co là m®t so đ c trưng cho khả năng xảy ra bien co đó khi thực hi n m®t phép thả. 1.2.1 Định nghĩa co đien của xác suat Định nghĩa 1.4. Xét phép thả T với không gian mau Ω là hǎu hạn. Bien co A ⊂ Ω, khi đó tỉ so P(A) = |A| |Ω| được goi là xác suat của bien co A. Nói m®t cách khác P là m®t hàm so xác định trên t p tat cả các t p con của Ω, mà t p giá trị của P là [0,1] vì |A| ≤ |Ω| với moi A ⊂ Ω. Ta có m®t so tính chat của xác suat như sau: 1) 0 ≤ P (A) ≤ 1, ∀ A⊂ Ω. 2) P(Ω) = 1. 3) P(∅) = 0. Bài toán 1.1. [xem [5]]. Gieo đong thời hai đong xu cân đoi đong chat tìm xác suat đe có bien co: A: " Xuat hi n hai m¾t sap". B: "M®t m¾t sap, m®t m¾t ngủa". C: "Ít nhat m®t m¾t sap". Lài giai. Phép thả T là tung hai đong xu cân đoi đong chat. Không gian mau: Ω = {SS, NN, NS, NN }, |Ω| = 4 A = {SS}, |A| = 1 B = {SN, NS}, |B| = 2
  • 8. 4 Viết đề tài giá sinh viên – ZALO:0973.287.149-TEAMLUANVAN.COM 6 C = {SS, SN, NS}, |C| = 3 nên P(A) = 1 = 0, 25; P(B) = 2 = 0, 5; P(C) = 3 = 0, 75. 4 4 4 Bài toán 1.2. [xem [7] trang 447]. Hi n nay có rat nhieu giải xő so trao giải thướng lớn cho nhũng người chon đúng m®t b® sáu so trong so n so nguyên dương đau tiên, trong đó n thường nam trong khoảng tù 30 đen 60. Tìm xác suat mà m®t người chon đúng sáu so trong so 40 so? Lài giai. Chỉ có m®t b® đạt giải thưởng lớn. Tőng so cách đe chon sáu so trong so 40 là 40! 40 = 34!6! = 3838380. Do đó, khả năng chien thang là 1 3838380 ≈ 0 . 00000026. Bài toán 1.3. M®t h®p có a quả cau trang, b quả cau đen, lay ngȁu nhiên lan lượt hai quả cau. Tìm xác suat đe bien co sau xảy ra a) A : "Quả cau thú nhat là trang". b) B : "Quả cau thú hai là trang biet quả cau thú nhat là trang". Lài giai. a) Ta có so cách lay lan lượt hai quả bóng là: (a + b)(a + b − 1) nên |Ω| = (a + b)(a + b − 1). So cách lay quả bóng đau tiên là trang, quả thá hai là tùy ý là a.(a + b − 1) nên |A| = a(a + b − 1). V y P(A) = a(a + b − 1) = a . (a + b)(a + b − 1) a + b b) Sau khi lan đau lay quả trang, so cách đe lan thá hai lay được quả trang là (a − 1) nên |B| = a − 1. So cách đe lay được m®t quả tà a + b − 1 quả là a + b − 1, tác |Ω| = a + b − 1 nên P (B) = a − 1 a + b − 1 Bài toán 1.4. Lay ngȁu nhiên ra 8 con bài tù b® tú lơ khơ 52 con. Tìm xác suat của bien co sau A : "Lay được 5 con màu đó". B : "Lay được m®t con cơ, hai con rô, ba con bích". C
  • 9. 5 Viết đề tài giá sinh viên – ZALO:0973.287.149-TEAMLUANVAN.COM 52 52 39 C : "Lay được m®t con át, hai con J, ba con 9, hai con 2". D : "Lay được ba con cùng m®t chat đã chon trước". Lài giai. Đe lay 8 con tà 52 con tú có C8 (cách) nên |Ω| = C8 Ta can lay 5 con đỏ, 3 con đen, nên 3 5 |A| = C26C26 = 171028000. Ta can lay 1 con cơ, 2 con rô, 3 con bích, 2 con tép, nên 5 3 3 2 |B| = C13.C26.C13.C13 = 22620312. Ta can lay 1 con át, hai con J, ba con 9, hai con 2, nên 1 2 3 2 |C| = C4 .C4 .C4 .C4 = 576. Ta can lay ba con cùng m®t chat và năm con thu®c ba chat khác nên Ta có: |D| = 13 .C5 = 286575757. P(A) = P (B) = 171028000 752538150 22620312 752538150 576 = 0, 227268; = 0, 03006; P (C) = P(D) = 752538150 286575757 752538150 = 0, 000007654; = 0, 2188148. Bài toán 1.5. Có n người khách ra khói nhà mà không lay mũ của mình. Chủ nhà không biet rõ chủ của các chiec mũ là ai nên gủi trả ho m®t cách ngȁu nhiên. Tìm xác suat đe a) Cả n người không nh¾n đúng mũ của mình. b) Cả n người được trả đúng mũ. c) Có k người (1 ≤ k ≤ n − 1) được trả đúng mũ. Lài giai. Ta có:|Ω| = n! a) Goi bien co A : "Cả n người không nh n đúng mũ" 3
  • 10. 6 Viết đề tài giá sinh viên – ZALO:0973.287.149-TEAMLUANVAN.COM | | Σ |C| = C D ; n−k n ≈ Khi đó:|A| = Dn , nên P (A) = Dn n! b) Goi bien co B : "Cả n người được trả đúng mũ" Khi đó: B = 1, nên P (B) = 1 . n! c) Goi bien co C : "Có k người được trả đúng mũ" Đe có k người được trả đúng mũ thì có đúng n − k người không được trả đúng mũ, nên n−1 k n k=1 n Σ −1 CkDn−k P (C) = k=1 . n! 1.2.2 Định nghĩa thong kê ve xác suat Định nghĩa 1.5. Tan suat xuat hi n bien co trong n phép thả là t so giǎa so phép thả trong đó bien co xuat hi n và tőng so phép thả được thực hi n. Ta ký hi u so phép thả là n, so lan xuat hi n bien co A là k. Tan suat xuat hi n bien co A là f(A) thì: k f(A) = n Định nghĩa 1.6. Xác suat xuat hi n bien co A trong m®t phép thả là m®t so p không đői mà tan suat f xuat hi n bien co đó trong n phép thả sě dao đ®ng xung quanh p, khi so phép thả của n tăng lên vô hạn thì P(A) ≈ f(A). Bài toán 1.6. Có the xem xác suat sinh con trai là bao nhiêu khi theo dõi 88200 tré sơ sinh ớ m®t vùng có 45000 con trai. Lài giai. Ta có P(A) f(A) = 4500 88200 = 0, 51. Tác xác suat sinh con trai xap xỉ 0,51. Hay tỉ l sinh con trai và con gái xap xỉ là 51 nam, 50 nǎ. 1.3 Định lý c ng xác suat Theo định nghĩa của bien co thì bien co là t p con của không gian mau Ω nên trên các bien co cũng có các phép toán t p hợp, trên cơ sở đó ta cũng xây dựng
  • 11. 7 Viết đề tài giá sinh viên – ZALO:0973.287.149-TEAMLUANVAN.COM m®t so công thác tính xác suat khác. Định nghĩa 1.7. Bien co C là tőng của hai bien co A và B, kí hi u là A + B, neu C chỉ xảy ra khi có ít nhat m®t trong hai bien co xảy ra. Nói cách khác, ta viet C = A + B ⇔ C = A ∪ B. Mở r®ng n C = A1 + A2 + ... + An ⇔ C = i= ∪ 1 Ai Định nghĩa 1.8. Hai bien co A và B goi là xung khac với nhau neu chúng không the đong thời xảy ra trong cùng m®t phép thả. Ve m t t p hợp A và B xung khac tương đương A ∩ B = ∅. Định nghĩa 1.9. Nhóm n bien co A1, A2, ..., An được goi là xung khac tàng đôi m®t neu bat kì hai bien co này cùng xung khac nhau. Tác là Ai ∩ Aj = ∅, ∀ i, j = 1, n. Định lý 1.1 (Đ nh lý c®ng xác suat). Cho A và B là hai bien co xung khac, khi đó P(A + B) = P(A) + P (B). ChGng minh. Ta có A + B = A ∪ B, giả sả |A| = m1, |B| = m2, tat cả các ket quả của không gian mau là n. Do |A ∪ B| = |A| + |B| − |A ∩ B|, mà |A ∩ B| = ∅ nên |A ∪ B| = |A| + |B| = m1 + m2. Suy ra: P(A ∪ B) = (m1 + m2)/n; P (A) + P (B) = m1/n + m2/n = (m1 + m2)/n. V y ta có P(A + B) = P(A) + P(B). H quả 1.1. Cho A1, A2, ..., An là nhóm các bien co đôi m®t xung khac, ta có: P(A1 + A2 + ... + An) = P(A1) + P(A2) + ... + P (An). Bài toán 1.7. Xác suat đe m®t xạ thủ ban trúng bia điem 10 là 0,1; trúng bia điem 9 là 0,2; trúng bia điem 8 là 0,25 và ít hơn điem 8 là 0,45. Xạ thủ ay ban
  • 12. 8 Viết đề tài giá sinh viên – ZALO:0973.287.149-TEAMLUANVAN.COM Σ n−m ≤ n−m − − k m®t viên đạn. Tìm xác suat đe xạ thủ được ít nhat 9 điem. Lài giai. Goi A1 là bien co "Xạ thủ ban trúng điem 10". Goi A2 là bien co "Xạ thủ ban trúng điem 9". Goi A là bien co "Xạ thủ ban được ít nhat 9 điem". V y A = A1 + A2. Vì A1, A2 xung khac nên ta có: P(A1 + A2) = P(A1) + P(A2) = 0, 1 + 0, 2 = 0, 3. Định nghĩa 1.10. Nhóm bien co A1, A2, ..., An được goi là m®t nhóm đay đủ các bien co neu A1 + A2 + ... + An = Ω và Ai ∩ Aj = ∅, ∀ i, j = 1, n. H quả 1.2. Neu A1, A2, ..., An tạo thành nhóm đay đủ các bien co thì n P i=1 (Ai) ! = i=1 P(Ai) = 1 Định nghĩa 1.11. Hai bien co A và B được goi là đoi l p neu chúng tạo thành m®t nhóm đay đủ các bien co, tác là A ∪ B = Ω và A ∩ B = ∅. Khi đó ta kí hi u bien co đoi (đoi l p) của bien co A là A. H quả 1.3. Neu A và A là hai bien co đoi l p, thì ta có P(A) = 1 − P(A). Bài toán 1.8. Trong hòm có n sản phȁm trong đó có m chính phȁm m ≤ n. Lay ngȁu nhiên k sản phȁm, tìm xác suat đe trong k sản phȁm có ít nhat m®t chính phȁm (k ≤ n). Lài giai. Goi A là bien co "k sản phȁm lay ít nhat m®t chính phȁm", thì bien co đoi của A là "k sản phȁm lay ra đeu là phe phȁm". Do đó P (A) = 1 − P (A). Mà |A| = Ck C k , k n m, nên P (A) = 1 n Bài toán 1.9. Trong hòm có 10 chi tiet trong đó có 2 chi tiet hóng. Tìm xác suat khi lay ra ngȁu nhiên 6 chi tiet thì không có quá m®t chi tiet hóng. Lài giai. Goi A là bien co "Trong 6 chi tiet lay ra không có chi tiet hỏng". n C Σ
  • 13. 9 Viết đề tài giá sinh viên – ZALO:0973.287.149-TEAMLUANVAN.COM C Mà P (A0) = 8 = C ; P(A1) = 2 8 i=1 Goi A1 là bien co "Trong 6 chi tiet lay ra có m®t chi tiet hỏng". Goi A0 là bien co "Trong 6 chi tiet lay ra không có quá m®t chi tiet hỏng". Khi đó A = A0 + A1, A0, A1 là hai bien co xung khac. P(A) = P(A0 + A1) = P(A0) + P(A1). C6 2 6 10 C 1C5 8 6 = 10 nên P (A) = 2/15 + 8/15 = 2/3. 1.4 Định lý nhân xác suat Bây giờ ta xét trường hợp m®t bien co C là giao của hai bien co A, B. Định nghĩa 1.12. Bien co C được goi là tích của hai bien co A và B, neu C xảy ra khi và chỉ khi A và B cùng xảy ra. Ve m t t p hợp: C = A.B ⇔ C = A ∩ B Định nghĩa 1.13. Bien co A được goi là tích của n bien co A1, A2, ..., An neu A Q n Định nghĩa 1.14. Hai bien co A, B được goi là đ®c l p với nhau neu vi c xảy ra của bien co này không làm thay đői xác suat xảy ra bien co kia và ngược lại. Hai bien co A, B không đ®c l p thì hai bien co đó goi là phụ thu®c nhau. Ví dn 1.2. Trong bình có 3 quả cau trang, 2 quả cau đen. Lay ngau nhiên m®t quả cau. Goi A là bien co "Lay được quả cau trang" thì P (A) = 3/5 . Quả cau được bỏ lại vào bình và tiep tục lay ra m®t quả cau. Goi B là bien co "Lan thá hai lay được quả cau trang" thì P (B) = 3/5, P (B) không phụ thu®c vào A nên A, B đ®c l p. Tuy nhiên neu ta lay quả cau trang và không bỏ lại vào bình thì ta có P(B) = 1/2. Khi đó A, B là phụ thu®c nhau. Chú ý: Neu A và B là hai bien co đ®c l p thì A và B, A và B , A và B cũng đ®c l p với nhau. Định nghĩa 1.15. Các bien co A1, A2, ..., An goi là đ®c l p tàng đôi neu Ai, Aj đ®c l p, i /= j, i, j = 1, n. Định nghĩa 1.16. Các bien co A1, A2, ..., An goi là đ®c l p toàn phan với nhau neu moi bien co đ®c l p với m®t tő hợp bat kì của các bien co còn lại. xảy ra khi và chỉ khi tat cả n bien co A1, A2, ..., An xảy ra. Kí hi u: A = Ai. 5 15
  • 14. 10 Viết đề tài giá sinh viên – ZALO:0973.287.149-TEAMLUANVAN.COM Q Q Định lý 1.2 (Đ nh lý nhân xác suat). Cho A, B là hai bien co đ®c l p thì P(A.B) = P(A).P (B). Nh n xét 1.1. P(A.B) = P(A).P(B) ⇔ A và B đ®c l p. H quả 1.4. Neu A, B đ®c l p thì P(A) = P(A.B)/P(B), P(B) > 0. H quả 1.5. Neu A1, A2, ..., An đ®c l p toàn phan thì P n Ai i=1 n = i=1 P(Ai). Bài toán 1.10. Trong m®t cu®c thi đau có A và B tham gia. Khả năng lot vào chung ket của A là 90%, của B là 70%. A, B không cùng m®t bảng đau. Tìm xác suat của các bien co D: "Cả hai lot vào chung ket". E: "Có ít nhat m®t người lot vào chung ket". F: "Chí có A lot vào chung ket". Lài giai. Goi A là bien co "Người A lot vào chung ket". Goi B là bien co "Người B lot vào chung ket". Khi đó, de thay A, B là hai bien co đ®c l p, và D = A.B; E = A.B + A.B + AB; F = A.B. Theo bài P (A) = 0, 9, P (B) = 0.7 nên P(D) = P(A.B) = P(A)P(B) = 0, 9.0, 7 = 0, 63; P(E) = P (A.B) + P(A.B) + P (A.B) = 0, 7.0, 9 + 0, 7.0, 1 + 0, 9.0, 3 = 0, 97; P(F ) = P(A.B) = P(A).P(B) = 0, 9.0, 3 = 0, 27. Bây giờ ta xét trường hợp hai bien co A và B phụ thu®c nhau. Trước het ta xét khái ni m xác suat có đieu ki n. Định nghĩa 1.17. Xác suat của bien co A được tính với đieu ki n bien co B đã xảy ra, goi là xác suat có đieu ki n của A và kí hi u là P(A/B)
  • 15. 11 Viết đề tài giá sinh viên – ZALO:0973.287.149-TEAMLUANVAN.COM Định lý 1.3 (Xác suat có đieu ki n). Cho A, B là hai bien co phụ thu®c, khi đó: P(A.B) = P (A).P (A/B) = P (B).P (B/A). ChGng minh. Giả sả A, B là hai bien co của cùng không gian mau Ω và |Ω| = n, |A| = m1.|B| = m2, |A.B| = k. Khi đó P(A.B) = k/n, P(B/A) = k/m1, P(A/B) = k/m2. V y: P (A.B) = k/n = (k/m1).(m1/n) = P (A)P (B/A). P (A.B) = k/n = (k/m2).(m2/n) = P (B)P (A/B). H quả 1.6. Neu P(B) > 0 thì P(A/B) = P(AB)/P(B). H quả 1.7. Neu A1, A2, ..., An là n bien co phụ thu®c nhau thì P (A1A2...An) = P (A1)P (A2/A1)P (A3/A1A2)...P (An/A1A2...An−1). H quả 1.8. Hai bien co A và B đ®c l p khi và chỉ khi P(A/B) = P (A), P (B/A) = P (B). Bài toán 1.11. M®t cơ quan có ba xe ô tô, khả năng xảy ra sự co tương úng với mői xe là 5%, 20%, 10%.Tìm khả năng xảy ra các tình huong sau: - Cả ba ô tô b sự co. - Có ít nhat m®t xe hoạt đ®ng tot. - Có đúng m®t xe hoạt đ®ng tot. - Cả ba xe hoạt đ®ng tot. - Có không quá hai xe hoạt đ®ng tot. Lài giai. Goi Ai là bien co "Xe thá i bị sự co", i = 1, 2, 3. Ba bien co này không xung khac nhưng đ®c l p. A là bien co "Cả ba ô tô cùng bị sự co". B là bien co "Có ít nhat m®t xe hoạt đ®ng tot". C là bien co "Có đúng m®t xe hoạt đ®ng tot". D là bien co "Cả ba xe cùng hoạt đ®ng không tot". E là bien co "Có không quá hai xe hoạt đ®ng không tot". Khi đó theo giả thiet: P (A1) = 0.05, nên P (A1) = 0, 95; P (A2) = 0, 2, nên P (A2) = 0, 8;
  • 16. 12 Viết đề tài giá sinh viên – ZALO:0973.287.149-TEAMLUANVAN.COM P (A3) = 0, 1, nên P (A3) = 0, 9. Ta có the bieu dien A = A1A2A3 nên P (A) = P (A1A2A3) và P (A1A2A3) = P (A1)P (A2)P (A3) = 0, 05.0, 2.0, 1 = 0, 001. B = A1 + A2 + A3, xét B = A1A2A3 = A nên P(B) = 1 − P (B) = 1 − P(A) = 1 − 0, 001 = 0, 999. C = A1A2A3 + A1A2A3 + A1A2A3 nên P (C) = 0, 05.0, 8.0, 1 + 0, 05.0, 2.0, 9 + 0, 95.0, 2.0, 1 = 0, 032. D = A1A2A3 nên P(D) = P(A1A2A3) = P (A1).P (A2).P (A3) = 0, 95.0, 8.0, 9 = 0, 684. E: "Không quá hai xe bị sự co" là bien co "Có ít nhat m®t xe hoạt đ®ng tot", tác E = B, nên P (E) = 0, 999. Bài toán 1.12. Xét phép thủ tung m®t đong xu bon lan. Giả sủ ta không thay ket quả nhưng ta biet có ít nhat hai m¾t sap. Tính xác suat tung được cả bon m¾t sap. Lài giai. Goi A là bien co "Tung được bon m t sap". Goi B là bien co "Có ít nhat hai m t sap". Do A ∩ B = A nên P (A/B) = P(A)/P(B). Mà P (A) = 1 1 4 6 1 11 = ; P(B) = + + = . 24 16 16 16 16 16 Khi đó P (A) = 1 . 11 Bài toán 1.13. Cho P = P1P2...Pn là m®t hoán v ngȁu nhiên của n so tự nhiên đau tiên. Goi A là bien co "P1 > P2", B là bien co "P2 > P3". Hói A và B có đ®c l¾p không? Lài giai. Ta có với hai so P1, P2 cháa hai khả năng xảy ra P1 > P2 ho c P2 > P1 nên P(A) = 1/2, tương tự P(B) = 1/2 và A ∩ B là bien co "P1 > P2 > P3", P(A ∩ B) = 1/6.
  • 17. 13 Viết đề tài giá sinh viên – ZALO:0973.287.149-TEAMLUANVAN.COM Vì với ba so P1, P2, P3 có 6 hoán vị, chỉ có m®t hoán vị thỏa mãn P1 > P2 > P3. Khi đó: P (A/B) = P (A ∩ B)/P (B) = (1/6)/(1/2) = 1/3 V y A, B không đ®c l p. P (A). M®t cách đơn giản hơn ta có ngay P (A ∩ B) = 1/6 = / P (A)P (B), nên A, B không đ®c l p. Bài toán 1.14. M®t người săn thó trong rùng. Khả năng anh ban trúng thó trong mői lan ban tí l ngh ch với khoảng cách. Anh ta ban lan đau với khoảng cách 20m với xác suat ban trúng là 50 %. Neu trượt anh ta ban tiep viên thú 2 ớ khoảng cách 30 m, neu trượt anh ta ban tiep viên thú 3 ớ khoảng cách 50 m. Tìm xác suat đe người thợ săn ban được thó trong lan đi săn này. Lài giai. Goi Ti là bien co "Thợ săn ban trúng thỏ lan thá i”, i = 1, 2, 3, ba bien co này không đ®c l p. Theo bài ta có P (T1) = k/20 = 0, 5, Suy ra k = 10. Do đó: P(T2/T1) = 10/30 = 1/3; P (T 3/T2) = 10/50 = 1/5; Goi T là bien co thợ săn ban trúng thỏ trong cu®c đi săn này T = T 1 + T1.T 2 + T1.T2.T 3. Khi đó: P(T) = P(T1 + T1.T2 + T1.T2.T3) = P(T1) + P(T1.T2 ) + P(T1.T2.T3) = 0, 5 + P(T1).P(T2/T1) + P(T1).P(T2/T1).P(T3/T1T2) = 0, 5 + (1 - 0, 5)(1/3) + (1 - 0, 5)(1 -1/3).0, 2 =22/30= 0, 733. 1.5 M t so m r ng của định lý c ng và định lý nhân xác suat hai phan trước ta đã tìm hieu định lý c®ng xác suat với đieu ki n các bien co xung khac, định lý nhân xác suat với đieu ki n các bien co đ®c l p. phan này chúng ta sě xét m®t so công thác mở r®ng của hai định lý trên.
  • 18. 14 Viết đề tài giá sinh viên – ZALO:0973.287.149-TEAMLUANVAN.COM Σ n Σ Σ Q Σ Y Y Σ Q Σ Trước het trong định lý c®ng xác suat, neu A, B không xung khac ta có ket quả sau: Định lý 1.4. Cho A, B là hai bien co của cùng không gian mau Ω, khi đó P(A + B) = P (A) + P(B) − P(A.B). ChGng minh. Giả sả |Ω| = n, |A| = m1, |B| = m2, |A ∩ B| = k, k ≥ 0. Do A, B không có đieu ki n xung khac. Khi đó ta có: |A ∪ B| = |A| + |B| − |A ∩ B| = m1 + m2 − k. Nên P (A ∪ B) = P (A + B) = (m1 + m2 − k)/n = m1/n + m2/n − k/n = P(A) + P(B) − P(A.B). Tương tự, áp dụng công thác tính so phan tả t p hợp ta cháng minh được: H quả 1.9. Cho A1, A2, ..., An là các bien co của không gian mau Ω ta có n P i=1 Ai ! = i=1 P (Ai) − i< Σ j<1 P (Ai)P (Aj)+ + i<j<1 P (AiAjAk) + ... + (−1)n−1P (A1A2...An) Định lý 1.5. Cho A1, A2, ..., An là các bien co đ®c l p toàn phan với nhau, khi đó ChGng minh. n n P i=1 n Ai ! = 1 − n P(Ai). i=1 Goi A = i=1 Ai thì A = i=1 Ai do đó Σ n ! Y n ! Do các Ai đ®c l p toàn phan nên Ai cũng đ®c l p toàn phan, nên ta có: n P i=1 Ai ! = i=1 P(Ai). Suy ra P n Ai i=1 = 1 − n P (Ai) i=1 Neu P (A1) = P (A2) = ... = P (An) = p thì ta có n P i=1 Ai ! = 1 − (1 − p)n n i=1 Ai i=1 Ai n P(A) = P = 1 − P (A) = 1 − P Σ Y
  • 19. 15 Viết đề tài giá sinh viên – ZALO:0973.287.149-TEAMLUANVAN.COM — − Σ − − Σ − Σ Bài toán 1.15. Xác suat đe đ®ng cơ thú nhat b trúng đạn là 0,2, đ®ng cơ thú hai b trúng đạn là 0,3, xác suat phi công b trúng đạn là 0,1. Tìm xác suat đe máy bay rơi, biet rang máy bay rơi khi cả hai đ®ng cơ đeu b trúng đạn ho¾c phi công b trúng đạn. Lài giai. Goi Ai là bien co " đ®ng cơ thá i bị trúng đạn", i = 1, 2. A3 là bien co "phi công bị trúng đạn". A là bien co "máy bay rơi". Khi đó A = A1A2 + A3, suy ra P (A) = P (A1A2 + A3). Do A1, A2, A3 không xung khac nên P (A) = P (A1A2) + P (A3) − P (A1A2A3). M t khác A1, A2, A3 đ®c l p toàn phan nên ta có P(A) = P(A1A2) + P(A3) − P(A1)P(A2)P(A3)= 0, 154. Bài toán 1.16. M®t người viet n lá thư, bó ngȁu nhiên vào n phong bì có đe sȁn đ a chí. Tìm xác suat có ít nhat m®t lá thư đúng đ a chí. Lài giai. Goi A là bien co "có ít nhat m®t lá thư đúng địa chỉ". Khi đó, A là bien co "tat cả các lá thư không đúng địa chỉ". So cách đe bỏ n lá thư vào n phong bì là n!. So cách tat cả lá thư không đúng địa chỉ là: n Dn = (−1) i−1 n! , nên P i! A = Dn . Suy ra: n! i=1 P(A) = 1 P(A) = 1 Dn n! n = 1 ( 1)i−1 1 i! i=1 n = ( 1)i 1 i! i=1 Bài toán 1.17. Phải tung m®t con súc sac bao nhiêu lan đe với xác suat không nhó hơn 0,5 ta có hy vong rang có ít nhat m®t lan được m¾t sáu cham. Lài giai. Giả sả ta tung con súc sac n lan. Goi Ai là bien co "tung lan i được m t sáu cham", i = 1, n . Goi A là bien co "trong n lan có ít nhat m®t lan tung được m t sáu cham". V y
  • 20. 16 Viết đề tài giá sinh viên – ZALO:0973.287.149-TEAMLUANVAN.COM Σ Σ Σ Q n A = Ai , các bien co Ai là không xung khac và đ®c l p toàn phan với nhau nên i=1 n ta có: P(A) = 1 - Ai . i=1 Vì P(A1) = P(A2) = ... = P(An) = 1/6, nên V y ta có P(A) = 1 - P(A1) = P(A2) = ... = P(An) = 5/6 5 n . 6 Theo giả thiet P (A) ≤ 0, 5 ⇔ 1 − V y phải tung ít nhat bon lan. 5 n ≤ 0, 5 ⇔ n ≥ 3, 7, n ∈ N∗ . Định lý 1.6 (Công thúc xác suat đay đủ). Cho H1, H2, ..., Hn là nhóm các bien co đay đủ. A là m®t bien co của cùng m®t không gian mau Ω. Khi đó ta có n P(A) = P (Hi)P (A/Hi). i=1 ChGng minh. Do H1, H2, ..., Hn là m®t nhóm các bien co đay đủ, nên A = A ∩ Ω = A.H1 + A.H2 + ... + A.Hn. Vì Hi ∩ Hj = ∅, ∀ i /= j nên A.Hi ∩A.Hj = ∅, ∀i /= j, i, j = 1, n . Do v y n P(A) = P (Hi)P (A/Hi). i=1 Bài toán 1.18. Có ba h®p giong nhau, h®p thú nhat có 10 sản phȁm trong đó có 6 chính phȁm, h®p thú hai có 15 sản phȁm trong đó có 10 chính phȁm, h®p thú ba có 20 sản phȁm trong đó có 15 chính phȁm. Lay ngȁu nhiên m®t h®p, tù h®p đó lay ngȁu nhiên ra m®t sản phȁm. Tính xác suat đe lay được chính phȁm. Lài giai. Goi A là bien co "lay được chính phȁm". Hi là bien co "chính phȁm lay ra thu®c h®p i". Khi đó, A xảy ra đong thời với H1 ho c H2 ho c H3. Xác suat đe lay được các h®p H1, H2, H3 là: P(H1) = P(H2) = P(H3) = 1/3 6
  • 21. 17 Viết đề tài giá sinh viên – ZALO:0973.287.149-TEAMLUANVAN.COM Σ Σ và P (A/H1) = 6/10, P (A/H2) = 10/15, P (A/H3) = 15/20. Do đó: P(A) = P (H1)P (A/H1) + P (H2)P (A/H2) + P (H3)P(A/H3) 1 6 . + 3 10 1 10 . + 3 15 1 15 . = 3 20 124 31 = . 180 45 Định lý 1.7. (Công thúc Bayes). Cho H1, H2, ..., Hn là m®t nhóm đay đủ các bien co. A là m®t bien co của cùng không gian mau Ω. Khi đó P(H /A) = P (Hi) P (A/Hi) . i n P (Hi) P (A/Hi) i=1 ChGng minh. Theo định nghĩa xác suat có đieu ki n và công thác xác suat đay đủ, ta có: P (A.Hi) = P (A).P (Hi/A) = P (Hi).P (A/Hi), i = 1, n . Tà đó: P(H /A) = P (Hi) P (A/Hi) = P (Hi) P (A/Hi) i P(A) n P (Hi) P (A/Hi) i=1 Bài toán 1.19. Dây chuyen lap ráp nh¾n được các chi tiet do hai máy sản xuat. Trung bình máy thú nhat cung cap 60% chi tiet, máy thú hai cung cap 40% chi tiet. Khoảng 90% chi tiet do máy m®t sản xuat là đạt chuȁn, 85% chi tiet do máy thú hai sản xuat là đạt chuȁn. Lay ngȁu nhiên tù dây chuyen m®t sản phȁm, thay sản phȁm đạt chuȁn. Tìm xác suat đe sản phȁm đó do máy thú nhat sản xuat. Lài giai. Goi A là bien co "chi tiet lay tà dây chuyen đạt chuȁn". Hi là bien co "chi tiet do máy thá i sản xuat", i = 1, 2... A xảy ra đong thời với H1, H2. Do đó xác suat can tìm là: P(H1/A) = P (H1) .P (A/H1) P (H1) .P (A/H1) + P (H2) .P (A/H2) Theo bài P(H1) = 0, 6; P (H2) = 0, 4; P (A/H1) = 0, 9; P (A/H2) = 0, 85 nên 0, 6.0, 9 P(H1/A) = 0, 6.0, 9 + 0, 4.0, 85 = 0, 614. Bài toán 1.20. Trước khi đưa sản phȁm ra th trường người ta phóng van ngȁu nhiên 200 khách hàng ve sản phȁm. Có 34 người trả lời "sẽ mua", 96 người trả lời
  • 22. 18 Viết đề tài giá sinh viên – ZALO:0973.287.149-TEAMLUANVAN.COM n n "có the sẽ mua", 70 người trả lời "không mua". Kinh nghi m cho thay ty l khách hàng mua hàng tương úng với cách trả lời trên là 40%, 20%, 10%. a) Tìm xác suat khách hàng mua sản phȁm. b) Trong so khách hàng thực sự mua sản phȁm có bao nhiêu phan trăm trả lời sẽ mua. Lài giai. Goi A là bien co "khách hàng mua sản phȁm". H1 là bien co "người đó trả lời là sě mua". H2 là bien co "người đó trả lời là có the sě mua". H3 là bien co "người đó trả lời là không mua". a) Theo công thác xác suat đay đủ P(A) = P (H1)P (A/H1) + P (H2)P (A/H2) + P (H3)P(A/H3) = (34/200).0, 4 + (96/200).0, 2 + (70/200).0, 01 = 0, 1675. V y so người mua sản phȁm là 16,75 b) Theo công thác Bayes P(H /A) = P (H1) P (A/H1) = 0, 17.0, 4 = 0, 40579. 1 P (A) 0, 1675 V y trong so khách hàng mua sản phȁm có xap xỉ 40% trả lời sě mua. Trong hoạt đ®ng thực te, có rat nhieu trường hợp m®t phép thả được l p đi l p lại. M®t dãy các phép thả được goi là đ®c l p với nhau neu xác suat xảy ra m®t bien co nào đó trong tàng phép thả không phụ thu®c vào m®t bien co đó có xảy ra ở các phép thả khác hay không. Chȁng hạn trong vi c sản xuat sản phȁm hay tung nhieu lan các đong xu, con súc sac tạo nên m®t dãy các phép thả đ®c l p. M®t dãy các phép thả đ®c l p thỏa mãn đieu ki n ket quả xảy ra chỉ có hai trường hợp ho c bien co A xảy ra với xác suat p; ho c bien co A không xảy ra với xác suat 1 − p = q, được goi là dãy phép thả Bernoulli. Định lý 1.8. Trong m®t dãy n phép thả Bernoulli liên quan đen bien co A. Xác suat đe bien co A xuat hi n đúng m lan là Pn(m) và Pn(m) = Cmpmqn−m = Cmpm(1 − p)n−m
  • 23. 19 Viết đề tài giá sinh viên – ZALO:0973.287.149-TEAMLUANVAN.COM n Σ C 100 10 n n m ChGng minh. Goi Ai là bien co "xảy ra bien co A ở lan thá i", i = 1, n. Khi đó Ai là bien co "không xảy ra bien co A ở lan thá i". B là bien co "trong n phép thả bien co A xảy ra đúng m lan". Khi đó bien co A xảy ra m lan , A xảy ra n − m lan. B = A1A2...AmAm+1...An + ... + A1 A2...An−mAn−m+1...An. So cách chon m lan trong n lan xảy ra bien co A là Cm . Với moi cách chon m lan đe bien co A xảy ra có n − m lan A xảy ra. Nên Pn(m) = P (B) = pmqn−m + ... + pmqn−m = Cmpmqn−m = Cmpm(1 − p)n−m Bài toán 1.21. M®t bài thi trac nghi m có 100 câu hói. Mői câu có bon phương án trả lời, trong đó chí có m®t phương án đúng. Mői câu đúng được m®t điem còn trả lời sai thì không được điem. M®t hoc sinh lười hoc đã chon ngȁu nhiên các phương án trả lời. Tìm xác suat đe hoc sinh này a) Đạt tù 50 điem trớ lên. b) Đạt tù 49 điem trớ xuong. Lài giai. Goi A là bien co "chon câu trả lời đúng", thì ở moi lan chon P(A) = 1/4, nên P (A) = 3/4. a) Khi đó P (m ≥ 50) = P100(50 ≤ m ≤ 100) 100 = 100(0, 25)m(0, 75)100−m. m=50 b) P (m < 50) = P100(0 ≤ m ≤ 49) = Σ 49 Cm (0, 25)m (0,75) 100−m . m=0 Bài toán 1.22. Theo ket quả đieu tra b nh lao, ty l người mac b nh lao ớ vùng X là 0,001. Tìm xác suat đe khi khám cho 10 người. a) Có hai người b lao. b) Có ít nhat m®t người b lao. Lài giai. Goi A là bien co "g p người bị lao" thì P (A) = 0, 001, P (A) = 0, 999 P10(2) = C2 (0, 0001)2(0, 999)8
  • 24. 20 Viết đề tài giá sinh viên – ZALO:0973.287.149-TEAMLUANVAN.COM Σ P10(m ≥ 1) = 1 − P10(0) = 1 − (0, 999)10. 1.6 Bien ngȁu nhiên và kì vong 1.6.1 Định nghĩa Định nghĩa 1.18. Cho Ω là t p hợp ket quả các phép thả nào đó. Bien ngau nhiên X là hàm so được định nghĩa trên Ω. Ví dn 1.3. Cho Ω là t¾p tat cả các đo th n đính, ta đ nh nghĩa bien ngȁu nhiên X cho tương úng bới mői G ∈ Ω với X(G) là so cạnh của G. Ho¾c ta có the đ nh nghĩa bien ngȁu nhiên Y cho tương úng với mői G ∈ Ω với Y (G) là thành phan liên thông của G. Định nghĩa 1.19. Cho X, Y là hai bien ngau nhiên trên Ω. Khi đó ta định nghĩa tőng hai bien ngau nhiên X + Y cho tương áng với moi u ∈ Ω với (X + Y )(u) = X(u) + Y (u) và tích hai bien ngau nhiên X.Y cho tương áng moi u ∈ Ω với X.Y (u) = X(u)Y (u). Thông so đ c trưng quan trong nhat của bien ngau nhiên đó là kì vong của bien ngau nhiên. Định nghĩa 1.20. Cho X : Ω → R là m®t bien ngau nhiên sao cho t p giá trị của X là S = {X(u), u ∈ Ω} hǎu hạn. Tác là X nh n hǎu hạn giá trị. Khi đó so E(X) = i.p(X = i) i∈S được goi là giá trị kỳ vong hay kì vong của X trên Ω. Trong đó P (X = i) là xác suat của bien co X(u) = i. P(X = i) = |{u ∈ Ω, X (u) = i}| |Ω| Chú ý: Ta dùng kí hi u EΩ(X) đe kí hi u cho kì vong của bien ngau nhiên X được định nghĩa trên không gian Ω. Ta có the viet là E(X) neu ta không can nhan mạnh Ω. Kì vong chính là giá trị trung bình của các giá trị i.
  • 25. 21 Viết đề tài giá sinh viên – ZALO:0973.287.149-TEAMLUANVAN.COM Σ dq i=1 i=1 Bài toán 1.23. Gieo đong thời hai con súc sac cân đoi đong chat. Goi X là tőng so cham xuat hi n trên hai con súc sac. Tính E(X). Lài giai. Ta có Ω = {(i, j)}, 1 ≤ i, j ≤ 6. Bien ngau nhiên X : Ω → S cho tương áng u = (i, j), với X(u) = i + j, nên S = {2..., 11, 12}. Ta l p được bảng X 2 3 4 5 6 7 8 9 10 11 12 P(X) 1 36 2 36 3 36 4 36 5 36 1 36 5 36 4 36 3 36 2 36 1 36 Khi đó E(X) = Σ 12 i.P(X = i) i=2 = 2.1/36+ 3.2/36+ 4.3/36+ 5.4/36+ 6.5/36+ ... + 11.2/36+ 12.1/36= 7. Bài toán 1.24. Xác suat đe m®t máy sản suat ra phe phȁm bang p. Máy sẽ được sủa chũa ngay khi làm ra phe phȁm. Tìm so sản phȁm trung bình được sản suat giũa hai lan sủa chũa. Lài giai. Goi X là so sản phȁm được sản xuat ra giǎa hai lan sảa chǎa. Khi đó X nh n giá trị thu®c t p N∗ do xác suat sản xuat ra phe phȁm là p nên xác suat sản xuat ra chính phȁm là q = 1 − p. Ta có the l p bảng X 1 2 3 ... n ... P p q.p q2.p ... qn−1.p ... So sản phȁm trung bình được sản xuat ra chính là E(X). Ta có ∞ ∞ E(X) = Σ n.qn−1p = p Σ n.qn−1. Do 0 < q < 1 nên theo công thác của cap so nhân lùi vô hạn E(X) = p Σ ∞ n.qn−1p = p. d 1 d = p. 1 = p 1 = p 1 = 1 dq 1 − q (1 − q)2 p2 p Nh n xét 1.2. Ta có the mở r®ng t p S hǎu hạn thành t p S đem được neu tőng ∞ i.p(X = i) có giới hạn hǎu hạn. i=1 1 − q i=1
  • 26. 22 Viết đề tài giá sinh viên – ZALO:0973.287.149-TEAMLUANVAN.COM i i 3 i=2 1.6.2 Tính tuyen tính của kì vong Định nghĩa 1.21. Cho c là m®t hang so ta định nghĩa bien ngau nhiên cX cho tương áng với moi x với so cX(u) = c(X(u)) với moi u ∈ Ω. Với định nghĩa tőng, tích của hai bien ngau nhiên, tích của m®t so với bien ngau nhiên chúng ta cháng minh được. Định lý 1.9. Cho c là m®t so thực, X, Y là hai bien ngau nhiên trên không gian Ω. Khi đó E(X + Y ) = E(X) + E(Y ); E(c.X) = cE(X). Nói m®t cách khác kì vong có tính chat tuyen tính. Bài toán 1.25. Cho p = p1p2...pn là m®t hoán v của t¾p {1, 2, ..., n}, n ≥ 2. Khi đó trung bình mői hoán v của 1, 2, ..., n sẽ có (n − 2)/3 so pi thóa mãn pi > pi − 1 và pi > pi+1 với i = 2, n − 1 . Lài giai. Goi Ω là t p tat cả các hoán vị của t p 1, 2, 3, ..., n. Trên Ω ta định nghĩa n − 2 bien ngau nhiên Yi, i =2, n − 1 cho tương áng với p ∈ Ω. Yi(p) = 1 neu p(i) thỏa mãn pi > pi − 1 và pi > pi + 1; Y (p) = 0 neu pi không thỏa mãn đieu ki n trên. Ta có với moi b® ba so pi−1, pi, pi+1 xác suat đe pi nhỏ nhat là 1/3, xác suat đe so pi không nhỏ nhat là 2/3. Do đó E(Yi) = 1/3.1 + 23.0 = 1/3 n Σ −1 so các pi thỏa mãn pi < pi−1 và pi > pi+1. Ta có n−1 n−1 E(Y ) = E( Σ Y ) = Σ E(Y ) = (n − 2)E(Y ) = n − 2 . Bài toán 1.26. Cho p = p1p2...pn là m®t hoán v của t¾p {1, 2, ..., n}, i được goi là điem co đ nh neu pi = i. Hói trung bình mői hoán v có bao nhiêu điem co đ nh. Lài giai. Goi Ω là t p tat cả các hoán vị của 1, 2, ..., n. Xét n bien ngau nhiên Xi trên Ω, cho tương áng moi p = p1, p2, ..., pn ∈ Ω với Xi(p) = 1 neu pi = i, Xi(p) = 0 neu pi /= i. i=2 i=2 Xét bien ngau nhiên Y = Yi . Khi đó với moi p ∈ Ω cho tương áng với Y (p) là 1
  • 27. 23 Viết đề tài giá sinh viên – ZALO:0973.287.149-TEAMLUANVAN.COM Σ Khi đó xác suat đe pi = i là 1/n, xác suat đe pi /= i là (n − 1)/n. Nên E(X ) = 1/n + n − 1 .0 = 1/n Xét bien ngau nhiên X = co định của p. Khi đó i n i=1 n n Xi, cho tương áng p ∈ Ω với X(p) là so các điem E(X) = Σ E(Xi) = nE(X1 1 ) = n. n = 1. i=1 V y trung bình moi hoán vị có m®t điem co định.
  • 28. 24 Viết đề tài giá sinh viên – ZALO:0973.287.149-TEAMLUANVAN.COM Chương 2. Ứng dnng phương pháp xác suat trong giải toán trung hoc pho thông Trong chương này trình bày vi c áp dụng tính chat đ c trưng xác suat, kỳ vong đe giải toán đại so như cháng minh sự ton tại, cháng minh đȁng thác, bat đȁng thác, tính toán tő hợp...Tài li u tham khảo chính trong phan này là các cuon [1],[3],[9],[10]. 2.1 Áp dnng xác suat và kì vong vào m t so bài toán thi hoc sinh giỏi Bài toán 2.1 (IMO 1970). Trên m¾t phȁng cho 100 điem, trong đó không có 3 điem nào thȁng hàng. Xét tat cả các tam giác có đính tại các điem đã cho. Chúng minh rang không quá 70%các tam giác này là tam giác nhon. Lài giai. Trước het ta cháng minh:"với 5 điem bat kì trên m t phȁng" trong đó không có ba điem nào thȁng hàng thì có ít nhat ba tam giác với đỉnh lay tà năm đỉnh trên không phải là tam giác nhon. Th t v y ta xét 3 trường hợp Trường hợp 1: Cho tam giác ABC và hai điem D, E nam trong ∆ABC, khi đó có ít nhat hai trong ba tam giác ∆ADB, ∆ADC, ∆DBC tù tại D, ít nhat hai trong ba tam giác ∆AEB, ∆AEC, ∆EBC tù tại E, do đó có ít nhat bon tam giác không nhon trong tőng so các tam giác được tạo nên tà 5 điem A, B, C, D, E. Trường hợp 2: Cho tá giác ABCD loi. Điem E nam trong tá giác ABCD. Trong
  • 29. 25 Viết đề tài giá sinh viên – ZALO:0973.287.149-TEAMLUANVAN.COM 100 97 100 97 n 100 97 100 Σ Σ tá giác ABCD có ít nhat m®t đỉnh trong bon đỉnh A, B, C, D không nhon. Giả sả là góc A thì có m®t tam giác không nhon là ABC. E nam trong tam giác ABC ho c ∆BCD, khi đó nó sě tạo ra hai tam giác tù. Trường hợp 3: Với ngũ giác loi ABCDE có ít nhat hai góc không nhon, giả sả là hai góc A, B. Khi đó ∆ABC, ∆ABE là hai tam giác không nhon và tá giác BCDE có ít nhat m®t góc không nhon, ta lại có thêm m®t tam giác không nhon. Bây giờ ta thay so cách chon 5 điem trong 100 điem là C5 . Với moi cách lay 5 điem đó có ít nhat 3 tam giác không nhon. Nhưng moi đỉnh của tam giác được đem C2 lan, do đó so tam giác không nhon ít nhat là 3.C5 C2 (với các đỉnh được lay tà 100 điem trên). Khi đó xác suat đe có m®t tam giác nhon nhieu nhat là 1 − 3.C5 /(C2 C3 ) = 0, 7. Tác ta có không quá 70% tam giác nhon. Bài toán 2.2 (MOP test 2007). Trong bảng n×n mői m®t trong so các so 1, 2, ..., n xuat hi n đúng n lan. Chúng minh rang ton tại ít nhat m®t hàng ho¾c m®t c®t với ít nhat √ n so phân bi t. Lài giai. Ta sě đi cháng minh với m®t hàng hay m®t c®t bat kì, trung bình so phân tả khác nhau là √ n. Th t v y so cách chon m®t hàng ho c m®t c®t trong bảng là 2n. Goi X là so phan tả khác nhau trên m®t hàng ho c c®t đã chon Xi ∈ {0, 1}, Xi = 1 neu i có m t trong hàng ho c c®t đã chon. Xi = 0 neu i không có m t trong hàng ho c c®t đã chon. n n Khi đó X = Xi mà E(Xi) = Xi.p(Xi = 1) = p(Xi = 1). i=1 i=1 M t khác so i xuat hi n trên ít hàng hay ít c®t nhat khi so i xuat hi n trong m®t ô vuông nhỏ kích thước √ n. √ n nên: 2 √ n 1 E(Xi = 1) ≥ Do tính tuyen tính của kì vong: 2n = √ n E(X) = Σ i=1 1 E(Xi) ≥ n.E(Xi) = n.√ n = √ n.
  • 30. 26 Viết đề tài giá sinh viên – ZALO:0973.287.149-TEAMLUANVAN.COM 1 2 n 1 2 n Mà E(X) là giá trị trung bình của so các phan tả khác nhau trên m®t hàng ho c m®t c®t nên ton tại hàng ho c c®t sao cho so phan tả khác nhau lớn hơn √ n, tác ít nhat có √ n phan tả khác nhau. Bài toán 2.3 (Bulgaria MO 1984). Cho xi, yi (i = 1, 2, . . . , n) là 2n so thực dương sao cho xi + yi = 1. Chúng minh rang (1 − x1x2 . . . xn)m + (1 − ym)(1 − ym) . . . (1 − ym) ≥ 1 với moi so nguyên dương m và n. Nh n xét 2.3. Có nhieu lời giải cho bài toán này, trong đó có cách giải thuan túy đại so. Tuy nhiên, các cách giải đó đeu khá phác tạp. Lời giải dưới đây của Pierre Bornsztein đăng trên website mathlinks.ro th t đep đě và thanh thoát. Lài giai. Xét thí nghi m xác suat sau đây: Xét c(1), c(2), . . . , c(n) là các đong xu sao cho với moi i, xác suat đe c(i) ra m t ngảa là xi. Ta tung các xu này m®t cách đ®c l p m lan. Khi đó (1 − x1 . . . xn)m là xác suat p(A) của bien co: "với moi m®t trong m lan tung, có ít nhat m®t đong xu ra m t ngảa ". Chú ý rang A = B ∪ C, trong đó B là "ton tại m®t đong xu ra m t ngảa ở moi m®t trong m lan tung ", và C là "có ít nhat m®t đong xu ra m t ngảa ở moi lan tung, nhưng đong xu này không giong nhau qua moi lan tung". Hơn nǎa B ∩ C = ∅, do đó p(A) = p(B) + p(C). M t khác, ta có (1 − ym)(1 − ym) . . . (1 − ym) là xác suat của bien co áng với moi m®t đong xu ít nhat m®t lan không ra m t ngảa trong m lan tung bang p(B̄), trong đó B̄ là bien co đoi l p với bien co B. Như v y ve trái của bat đȁng thác đã cho là p(A) + p(B̄) = p(B) + p(B̄) + p(C) = 1 + p(C) ≥ 1. Chú ý rang đȁng thác xảy ra khi và chỉ khi n = 1. Trong bài toán trên, bieu thác ve trái và đ c bi t là đieu ki n xi + yi = 1 gợi ý ta đen ý tưởng sả dụng xác suat. Xét bài toán sau đây tuy t nhiên không có ”dau hi u” của xác suat:
  • 31. 27 Viết đề tài giá sinh viên – ZALO:0973.287.149-TEAMLUANVAN.COM Bài toán 2.4 (Putnam 2000). Cho aj, bj, cj, 1 ≤ j ≤ N là các so nguyên. Giả sủ rang với mői j, trong ba so aj, bj, cj có ít nhat m®t so lé. Chúng minh rang ton tại các so nguyên r, s, t sao cho raj của j, 1 ≤ j ≤ N. + sbj + tcj là lé với ít nhat 4N 7 giá tr Lài giai. Cách 1. (Manjul Bhargava, Kiran Kedlaya và Lenny). Xét 7 b® ba (a, b, c) với a, b, c ∈ {0, 1}, không phải tat cả đeu bang 0. Vì rang rj, sj, tj không phải tat cả đeu chȁn, nên 4 trong các tőng arj + bsj + ctj với a, b, c ∈ {0, 1} là chȁn và 4 là lẻ. Tat nhiên là tőng với a = b = c = 0 là chȁn, do đó ít nhat 4 trong 7 tőng với a, b, c không đong thời bang 0 có tőng lẻ. Nói cách khác, có ít nhat 4N trong các b® (a, b, c, j) cho tőng lẻ. Theo nguyên lý Dirichlet, ton tại m®t b® (a, b, c) với ít nhat 4N 7 tőng là lẻ. Cách 2. Bài toán trên đây được sả dụng lại tại kỳ thi Olympic Toán Singapore năm 2012 (SMO 2012) và trên Mathlinks giới thi u lời giải xác suat thú vị sau. Ta xét tat cả theo modul 2, vì trong bài ta chỉ quan tâm đen tính chȁn lẻ. Ta có 7 cách chon cho b® (r, s, t) với r, s, t không đong thời bang 0; với moi b® (a, b, c), có đúng 4 trong 7 b® sao cho ra + sb + tc ≡ 1. Suy ra, với (ai, bi, ci) đã cho neu ta chon ngau nhiên (r, s, t) /= (0, 0, 0) thì giá trị kỳ vong của so các bieu thác lẻ là 4N . Nhưng neu đây là so trung bình thì phải 7 có ít nhat m®t b® (r, s, t) có so này lớn hơn hay bang 4N . 7 Nh n xét 2.4. Lời giải trên đây đã sả dụng m®t nguyên lý rat đơn giản (giả thiet Karramata): Neu trung bình của m®t so so là A thì sě có ít nhat m®t trong các so đó ≥ A và ít nhat m®t trong các so đó ≤ A. Neu áp dụng vào xác suat thì giá trị trung bình sě tương áng với giá trị kỳ vong. Đây là m®t nguyên lý rat hǎu hi u mà ta sě còn nhac tới ở nhǎng phan sau. Bài toán 2.5 (APMO 1998). Cho F là t¾p hợp tat cả các b® (A1, A2, . . . , An) trong đó mői Ai, i = 1, 2, . . . , n là t¾p con của {1, 2, . . . , 1998}. Giả sủ |A| ký hi u so phan tủ của t¾p hợp A, hãy tìm (A1,A2 Σ ,...,An)∈F |A1 ∪ A2 ∪ · · · ∪ An|. Lài giai. Chú ý rang t p hợp {1, 2, . . . , 1998} có 21998 t p con vì ta có the chon
  • 32. 28 Viết đề tài giá sinh viên – ZALO:0973.287.149-TEAMLUANVAN.COM Σ C v C 799 399 x hay không chon m®t phan tả vào t p con. Vì the có tat cả 21998n so hạng trong tőng trên. Bây giờ ta tính giá trị trung bình của moi so hạng. Với moi i = 1, 2, . . . , 1998. i là phan tả của A1 ∪ A2 ∪ · · · ∪ An neu và chỉ neu nó là phan tả của ít nhat m®t trong các A1, A2, . . . ,An. Xác suat của bien co này là 1 − 2−n. Do đó, giá trị trung bình của moi so hạng trong tőng là 1998(1 − 2−n), và như the đáp so là 21998n.1998(1 − 2−n). Bő đe sau đây là chìa khóa cho nhieu bài toán giải bang phương pháp xác suat. Bo đe 2.1. Cho X là bien ngau nhiên. Khi đó ton tại điem nào đó của không gian xác suat mà X ≥ E[X], và ton tại điem bào đó của không gian xác suat mà X ≤ E[X]. Bài toán 2.6 (Iran TST 2008). Giả sủ rang 799 đ®i bóng chuyen tham gia vào m®t giải đau mà trong đó hai đ®i bat kỳ đau với nhau đúng m®t lan. Chúng minh rang ton tại hai nhóm A và B rời nhau, mői nhóm có 7 đ®i sao cho mői đ®i bóng của nhóm A đeu thua các đ®ng bóng của nhóm B. Lài giai. Xét giải đau như m®t đo thị có hướng đay đủ. Ta xét A là m®t t p ngau nhiên có 7 phan tả. Goi X là so đ®i thang tat cả các đ®i của A. Goi d(v−) là b c vào của v, ta có E(X) = 7 d(v−) 7 . 799 Nhưng Σ v d(v−) = C2 , nghĩa là b c trong trung bình của m®t đỉnh đúng bang 399. Theo tính loi của hàm C7, ta có E(X) ≥ 799C7 7 799 ≈ 800. 1 7 ≈ 6.25, nhưng đieu này là đủ vì X là so nguyên. Chon 7 đ®i bóng của B tà nhóm đ®i thang tat cả các đ®i của A ta có đieu can tìm. Bài toán 2.7 (Russia MO 1996). Trong vi n Duma quoc gia có 1600 đại bieu, l¾p thành 1600 tieu ban, mői tieu ban có 80 người. Chúng minh rang ta có the tìm được hai tieu ban có ít nhat 4 thành viên chung. 2 C
  • 33. 29 Viết đề tài giá sinh viên – ZALO:0973.287.149-TEAMLUANVAN.COM ni . Σ E(X) ≥ n C = 800 Lài giai. Chon ngau nhiên m®t c p tieu ban (tác là lay m®t cách ngau nhiên m®t c p trong 2 16000 c p). Goi X là so người có trong cả hai tieu ban được chon. Chú ý rang X = X1 + · · · + X1600, trong đó moi Xi là bien ngau nhiên {0, 1} chỉ ra rang người thá i có m t trong cả hai tieu ban hay không. Theo tính tuyen tính của kỳ vong, ta có E[X] = E[X1] + · · · + E[X1600]. Ta thay các E[Xi] có the tính de dàng. Goi ni là so tieu ban mà người thá i thu®c vào tieu ban đó. Khi đó E[Xi] = P[người thá i được chon vào cả hai tieu ban] = C2 2 16000 Thông tin mà ta biet ve {ni} là tőng của chúng i ni = 16000.80. Đieu này gợi cho ta sả dụng tính loi đe đánh giá E[X] thông qua giá trị trung bình của {ni}, được ký hi u là n và bang n = (16000.80)/1600 = 800. Ta có 1600C2 2 16000 1600C2 2 16000 = 1600 800.799 16000.15999 = 3.995. Theo Bő đe 2.1, ta biet rang sě có m®t ket quả nào đó cho ta X ≥ 3.995. Vì X luôn là so nguyên, ket quả này thực sự phải có X ≥ 4. Nói riêng, ta ket lu n rang có m®t c p hai tieu ban có ≥ 4 thành viên chung. Bài toán 2.8 (MOP Test 2008). Giả sủ a, b, c là các so thực dương sao cho với moi n nguyên [an] + [bn] = [cn]. Chúng minh rang ít nhat m®t trong ba so a, b, c nguyên. Nh n xét 2.5. Bạn có the sả dụng ket quả lý thuyet so quen thu®c sau đây: neu x là so vô t thì phan phân của các b®i so của x phân bo đeu trên đoạn [0, 1]. Nói riêng, neu ta chon n m®t cách ngau nhiên trong {1, 2, . . . , N} thì E[xn] → 1/2 khi N → ∞. Lài giai. Giả sả rang không có so nào trong a, b, c là so nguyên. Chia hai ve cho n và cho n dan đen vô cùng, ta được a + b = c. C C C
  • 34. 30 Viết đề tài giá sinh viên – ZALO:0973.287.149-TEAMLUANVAN.COM { } 2 2100 2100 2q 2 2q 4 2 Tà đó, suy ra {an} + {bn} = {cn}. (1) Neu x vô t thì {xn} phân bo đeu trên đoạn [0, 1]. Nói riêng, neu ta chon n m®t cách ngau nhiên trong {1, 2, . . . , N} thì E[{xn}] → 1/2 khi N → ∞. M t khác, neu x là so hǎu t có dạng toi giản là p/q thì xn có kỳ vong tien đen q − 1 = 1 − 1 . Như v y nó nam trong khoảng h1 , 1 . Ket lu n: với so không nguyên x, E[{xn}] → t, trong đó t ∈ h1 , 1 i . Lay kỳ vong hai ve của (1), và cho n dan đen vô cùng, ta thay rang cách duy nhat đe có đȁng thác là E[{an}] và E[{bn}] phải tien đen 1 , và 4 1 E[{cn}] → 2 . Nhưng cách duy nhat đe có kỳ vong 1 4 là khi a, b hǎu t , còn cách duy nhat đe có kỳ vong 1 là c vô t . Nhưng do a + b = c nên ta không the có hai so hǎu t c®ng 2 lại ra so vô t , mâu thuan. Phương pháp xác suat có áng dụng hi u quả trong vi c cháng minh sự ton tại của m®t cau trúc. Ta biet rang trong 6 người bat kỳ ton tại 3 người đôi m®t quen nhau ho c ba người đôi m®t không quen nhau. Khi 6 được thay bở 5 thì đieu này không còn đúng nǎa và ta có the cháng tỏ đieu này bang cách chỉ ra phản ví dụ. Khi các con so là lớn, vi c xây dựng phản ví dụ trở nên khó khăn. Trong nhǎng trường hợp như the phương pháp xác suat tỏ ra hǎu dụng. Bài toán 2.9. Chúng minh rang giũa 2100 người, không nhat thiet phải có 200 người đôi m®t quen nhau ho¾c 200 người đôi m®t không quen nhau. Lài giai. Ta sě cho m®t c p 2 người bat kỳ quen nhau ho c không quen nhau bang cách tung m®t đong xu đoi xáng. Trong m®t nhóm 200 người, xác suat đe ho đôi m®t quen nhau ho c đôi m®t không quen nhau là 2 × 2−C200 = 2−19899. Vì có C200 cách chon ra 200 người, xác suat ton tại 200 người đôi m®t quen nhau ho c đôi m®t không quen nhau nhieu nhat bang (2100) 200 2101 C200 × 2−19899 < 200! × 2−19899 = 200! < 1. Tà đây suy ra xác suat không ton tại 200 người đôi m®t quen nhau ho c đôi m®t không quen nhau lớn hơn 0, tà đó suy ra đieu phải cháng minh. 4 2
  • 35. 31 Viết đề tài giá sinh viên – ZALO:0973.287.149-TEAMLUANVAN.COM 100 99 Ta thay ở đây m®t phương pháp tőng quát đe xây dựng ví dụ ngau nhiên: neu xác suat của ton tại ví dụ ta can là dương thì ton tại ví dụ đó. Bài toán 2.10. Trong mői ô của bảng 100×100, ta viet m®t trong các so nguyên 1, 2, . . . , 5000. Hơn nũa, mői m®t so nguyên xuat hi n trong bảng đúng 2 lan. Chúng minh rang ta có the chon được 100 ô của bảng thóa mãn 3 đieu ki n sau: (1) Mői m®t hàng được chon đúng m®t ô. (2) Mői m®t c®t được chon đúng m®t ô. (3) Các so trong các ô được chon đôi m®t khác nhau. Lài giai. Chon hoán vị ngau nhiên (a1, a2, . . . , a100) của {1, 2, . . . , 100} và chon ô thá ai trong hàng thá i. Cách chon như v y thỏa mãn (1) và (2). Với moi j = 1, . . . , 5000, xác suat đe chon 2 ô có cùng so j là 0 neu hai ô này cùng hàng ho c cùng c®t và là 1 × 1 trong trường hợp ngược lại. Do đó xác suat đe cách chon này thỏa mãn (3) ít nhat là 1 1 − 5000 × 100 × 999 > 0 và ta có đieu phải cháng minh. Nh n xét 2.6. Ta có the de dàng chuyên hai lời giải xác suat nêu ở hai bài toán trên sang lời giải chỉ sả dụng thuan túy phép đem (bang cách tính so các ket quả thu n lợi thay vì tính xác suat), mà thực chat sě hoàn toàn giong. Nhưng lời giải xác suat ngan gon hơn và tự nhiên hơn. Nh n xét 2.7. M®t tính chat mang tính đ c trưng của xác suat là đȁng thác P (A1) + P (A2) + · · · + P (An) = 1, neu {A1, A2, . . . , An} là m®t phân hoạch của không gian xác suat Ω. Tính chat này có the dùng đe cháng minh nhieu đȁng thác tő hợp bang phương pháp xác suat. Bài toán 2.11. Cho p, q là các so thực dương sao cho p + q = 1. Chúng minh rang p + pq + pq2 + pq3 + · · · = 1. (∗)
  • 36. 32 Viết đề tài giá sinh viên – ZALO:0973.287.149-TEAMLUANVAN.COM P Lài giai. Xét thí nghi m tung đong xu với xác suat ra m t ngảa là p và m t xap là q. Ta thực hi n thí nghi m cho đen khi ra được m t ngảa. Goi X là so lan tung, khi đó P(X = n) = pqn−1. Ve trái của đȁng thác (*) bang P(X = 1) + P(X = 2) + · · · + P(X = n) + . . . và dĩ nhiên là bang 1. Bài toán 2.12 (IMO Shortlist 2006). Cho S là t¾p hũu hạn các điem trên m¾t phȁng sao cho không có ba điem nào thȁng hàng. Với mői m®t đa giác loi P với các điem thu®c S, goi a(P ) là so các điem của P và b(P ) là so các điem của S nam ngoài P. Chúng minh rang với moi so thực x, ta có Σ xa(P )(1 − x)b(P ) = 1, trong đó tőng được tính theo tat cả các đa giác loi có đính thu®c S. Chú ý quan trong: đoạn thȁng, m®t điem và t p rong được coi là đa giác loi với 2, 1 và 0 đỉnh tương áng. Lài giai. Ta tô màu m®t cách ngau nhiên các điem bang màu đen và màu trang, trong đó các điem được tô màu đen với xác suat x. Với moi đa giác loi P, goi EP là bien co tat cả các đỉnh nam trên chu vi của P có màu đen và tat cả các đỉnh nam ngoài P có màu trang. Các bien co này đôi m®t xung khac nhau, như the ve trái là xác suat của sự ki n có m®t EP nào đó đúng. Nhưng đây là sự ki n chac chan xảy ra: ta chỉ can xét bao loi của tat cả các điem màu đen! Đe tính xác suat của m®t bien co theo định nghĩa cő đien ta thường phải giải quyet hai bài toán tő hợp: tính so các ket quả thu n lợi và tính so các ket quả có the. Thông thường, bài toán sau đơn giản hơn bài toán trước. Chính đieu này tạo ra m®t tình huong áng dụng thú vị của xác suat: Neu ta tính được so ket quả có the và tính được xác suat thì sě tính được so ket quả thu n lợi. Bài toán 2.13. Trong so cách chon ra 3 đính tù 8 đính của hình l¾p phương đơn v , có bao nhiêu cách chon thóa mãn đieu ki n: 3 đính được chon là đính của m®t tam giác đeu. Bài toán này không phải là khó, nhưng cũng khá roi. Ta giải bài này bang cách tính xác xuat ba đỉnh được chon ngau nhiên tạo thành 3 đỉnh của 1 tam giác đeu.
  • 37. 33 Viết đề tài giá sinh viên – ZALO:0973.287.149-TEAMLUANVAN.COM 8 Ta lan lượt chon các đỉnh. Đỉnh đau tiên có the là 1 đỉnh bat kỳ. Với đỉnh thá hai, khi đỉnh thá nhat đã được chon thì ta chỉ có the chon 1 trong 3 đỉnh có khoảng cách √ 2 đen đỉnh đau. Xác suat thành công là 3/7. lượt cuoi, xác suat thành công là 2/6. Như v y xác suat đe 3 đỉnh được chon là 3 đỉnh của 1 tam giác đeu sě là 1/7. Vì so cách chon 3 đỉnh tà 8 đỉnh là C3 nên so cách chon thỏa mãn đieu ki n 3 đỉnh được chon là đỉnh của m®t tam giác đeu sě bang 1 C3 = 8.7.6 = 8. 7 8 7.1.2.3 Các ví dụ trên đây cho thay phương pháp xác suat đôi khi mạnh hơn các phương pháp truyen thong. Ta ket thúc phan này bang ví dụ sau, sả dụng m®t tính chat mang tính hien nhiên của xác suat, đó là xác suat của m®t bien co luôn nam giǎa 0 và 1. Bài toán 2.14. Trong m®t kỳ thi có n môn thi, trong đó có đe tieng Pháp và đe tieng Anh. Thí sinh có the thi bao nhiêu môn tùy ý, nhưng thí sinh chí có the chon m®t trong hai ngôn ngũ cho mői môn thi. Với hai môn thi bat kỳ, ton tại m®t thí sinh thi hai môn này bang các ngôn ngũ khác nhau. Neu mői m®t môn có nhieu nhat 10 thí sinh dự thi, hãy tìm giá tr lớn nhat có the của n. Lài giai. Đáp so là 1024. Ví dụ sau đây cho thay n = 1024 là có the. Giả sả có 10 thí sinh (đánh so tà 1 đen 10) tham dự tat cả 1024 môn thi (đánh so tà 0 đen 1023). Với thí sinh i, môn thi thá j sě được thi bang tieng Pháp neu chǎ so thá i tính tà bên phải sang trong bieu dien nhị phân của j là 0 và sě thi bang tieng Anh trong trường hợp ngược lại. Bang cách này de dàng kiem tra được đieu ki n được thỏa mãn. (Ket quả cũng như ví dụ có the thu được không may khó khăn neu ta thay 10 bang 1 so nhỏ hơn và quan sát quy lu t.) Đe cháng minh rang 1024 là so lớn nhat, ta gán ngau nhiên cho các thí sinh là ”người Pháp” ho c ”người Anh). Goi Ej là bien co ”moi thí sinh thi môn j đeu thi bang đe đúng với quoc tịch mình được gán”. Vì ‘với hai môn thi bat kỳ, ton tại m®t thí sinh thi hai môn này bang hai ngôn ngǎ khác nhau’, nên không có hai Ej nào có the xảy ra đong thời. Tà đây suy ra: P(ít nhat m®t trong các Ej xảy ra) = P(E1) + · · · + P(En) ≥ n/1024.
  • 38. 34 Viết đề tài giá sinh viên – ZALO:0973.287.149-TEAMLUANVAN.COM ≥ n n i n Σ Σ 2 + C2 − ≥ Nhưng vì xác suat của m®t bien co bat kỳ không vượt quá 1 nên tà đây ta suy ra n ≤ 1024 , đó là đieu phải cháng minh. 2.2 M t so dạng toán thi Olympic liên quan Bài 2.1 (IMO 1998). Trong m®t cu®c thi, có m thí sinh và n giám khảo, trong đó n ≥ 3 là so nguyên lé. Mői m®t giám khảo sẽ đánh giá thí sinh đ¾u ho¾c rớt. Giả sủ k là so sao cho với mői c¾p hai giám khảo đánh giá của ho trùng nhau ớ nhieu nhat k thí sinh. Chúng minh rang: k n − 1 m 2n Lài giai. Do n lẻ, đ t n = 2r + 1, tà n giám khảo có C2 c p giám khảo, mà 2 giám khảo có ý kien trùng nhau nhieu nhat ở k thí sinh nên tőng so ý kien trùng nhau của các giám khảo toi đa là C2.k. M t khác thí sinh thá i được xi giám khảo đong ý, n−xi giám khảo không đong ý. Khi đó so ý kien trùng nhau của giám khảo là: xi n−xi = x2 + (n − xi)2 − n 2 r2 + (n − r)2 − n 2 (n 1)2 . 4 Nên tőng so ý kien trùng nhau ít nhat là m(n − 1)2 4 , đieu đó dan đen: k.C2 ≥ m(n − 1)2 4 nên k n − 1 . m 2n Bài 2.2 (IMO 1987). Goi pn(k) là so các hoán v của t¾p {1, 2, ..., n} , n ≥ 1, có đúng k điem bat đ®ng. Chúng minh rang: n k.pn(k) = n! k=0 Lài giai. Đȁng thác can cháng minh tương đương với n k.pn(k) = 1 n! k=0 Goi Ω là t p tat cả các hoán vị của {1, 2, ..., n}, tác |Ω| = n! P = p1p2...pn là m®t hoán vị của 1, 2, ..., n, X là so điem bat đ®ng của p ∈ Ω. C ≥ =
  • 39. 35 Viết đề tài giá sinh viên – ZALO:0973.287.149-TEAMLUANVAN.COM n Σ k=0 Ta có: E(X) = 1. M t khác ta có xác suat đe P có k điem bat đ®ng là: Ck.Dn−k n! Khi đó E(X) theo định nghĩa được tính: n = pn(k) . n! E(X) = k. pn(k) = 1 n! k=0 Bài 2.3 (IMO Shortlist, 1999). Cho A là m®t t p gom N th ng dư modulo N 2. Cháng minh rang ton tại t p B gom N th ng dư modulo N2 sao cho ít nhat m®t nảa th ng dư modulo N2 có the viet dưới dạng a + b, a ∈ A, b ∈ B. Bài 2.4 (IMO 1998). Trong m®t cu®c thi, có m thí sinh và n giám khảo, trong đó n ≥ 3 là so nguyên lẻ. Moi m®t giám khảo sě đánh giá thí sinh đ u ho c rớt. Giả sả k là so sao cho với moi c p hai giám khảo đánh giá của ho trùng nhau ở nhieu nhat k thí sinh. Cháng minh rang k m n − 1 2n Bài 2.5 (IMO 1971). Cháng minh rang với moi so nguyên dương m, ton tại t p hǎu hạn S các điem trên m t phȁng với tính chat sau: Với moi điem A trong S, có đúng m điem của S có khoảng cách 1 đen A. Bài 2.6 (China MO 1986). Cho z1, z2, . . . , zn là các so phác. Cháng minh rang ton tại t p con S ⊆ {1, . . . , n} sao cho .Σ . 1 Σ .j∈S zj . ≥ π j=1 |zj|. Bài 2.7 (IMO Shortlist 1987). Cháng minh rang ta có the tô màu các phan tả của t p hợp {1, 2, . . . , 1987} bởi 4 màu sao cho moi cap so c®ng 10 phan tả của t p hợp này đeu không đơn sac. Bài 2.8 (Zarankiewicz). Cháng minh rang ton tại m®t cách chia t p hợp các so nguyên dương thành hai t p con sao cho moi t p con đeu không cháa cap so c®ng với vô so phan tả và không cháa ba so nguyên liên tiep. Bài 2.9 (IMO 1987). Goi pn(k) là so các hoán vị của t p {1, . . . , n}, n ≥ 1, có đúng Σ n ≥ n k điem bat đ®ng. Cháng minh rang kpn(k) = n!
  • 40. 36 Viết đề tài giá sinh viên – ZALO:0973.287.149-TEAMLUANVAN.COM P Bài 2.10 (Russia MO 1996). Trong vi n Duma quoc gia có 1600 đại bieu, l p thành 16000 tieu ban, moi tieu ban có 80 người. Cháng minh rang ta có the tìm được hai tieu ban có ít nhat 4 thành viên chung. Bài 2.11 (IMO 1998). Trong m®t cu®c thi, có a thí sinh và b giám khảo, trong đó b ≥ 3 là so nguyên lẻ. Moi m®t giám khảo sě đánh giá thí sinh ”đ u” ho c ”rớt”. Giả sả k là so sao cho với moi c p hai giám khảo, đánh giá của ho trùng ở nhieu nhat k thí sinh. Cháng minh rang k/a ≥ (b − 1)/(2b). Bài 2.12 (APMO 1998). Cho F là t p hợp tat cả các b® (A1, A2, . . . , An) trong đó moi Ai, i = 1, 2, . . . , n là t p con của {1, 2, . . . , 1998}. Giả sả |A| ký hi u so phan tả của t p hợp A, hãy tìm (A1,A2 Σ ,...,An)∈F |A1 ∪ A2 ∪ · · · ∪ An|. Bài 2.13 (USA TST 2001). Với t p hợp S, ký hi u | S | là so phan tả của S. Cho A là t p hợp các so nguyên dương với |A| = 2001. Cháng minh rang ton tại t p B sao cho (i) B ⊆ A; (ii) |B| ≥ 668; (iii) với moi u, v ∈ B (không nhat thiet phân bi t), u + v ∈ / B. Bài 2.14 (Bay Area Math Olympiad 2004). Cho n so thực không đong thời bang 0 có tőng bang 0. Cháng minh rang ton tại m®t cách đánh so các so này là a1, a2, . . . , an sao cho a1a2 + a2a3 + · · · + an−1an + ana1 < 0. Bài 2.15 (IMO Shortlist 2006). Cho S là t p hǎu hạn các điem trên m t phȁng sao cho không có ba điem nào thȁng hàng. Với moi m®t đa giác loi P với các điem thu®c S, goi a(P ) là so các điem của P và b(P ) là so các điem của S nam ngoài P. Cháng minh rang với moi so thực x, ta có Σ xa(P )(1 − x)b(P ) = 1, trong đó tőng được tính theo tat cả các đa giác loi có đỉnh thu®c S. Bài 2.16 (MOP Test 2007). Trong bảng n×n moi m®t trong các so 1, 2, . . . , n xuat hi n đúng n lan. Cháng minh rang ton tại ít nhat m®t hàng ho c m®t c®t với ít nhat so phân bi t.
  • 41. 37 Viết đề tài giá sinh viên – ZALO:0973.287.149-TEAMLUANVAN.COM Bài 2.17 (Iran TST 2008). Giả sả rang 799 đ®i bóng chuyen tham gia vào m®t giải đau mà trong đó hai đ®i bat kỳ đau với nhau đúng m®t lan. Cháng minh rang ton tại hai nhóm A và B rời nhau, moi nhóm có 7 đ®i sao cho moi đ®i bóng của nhóm A đeu thua các đ®ng bóng của nhóm B. Bài 2.18 (MOP Test 2008). Giả sả a, b, c là các so thực dương sao cho với moi n nguyên [an] + [bn] = [cn]. Cháng minh rang ít nhat m®t trong ba so a, b, c nguyên. Bài 2.19. Cho X là bien ngau nhiên. Cháng minh rang ton tại điem của không gian xác suat mà X ≥ E[X], và ton tại điem nào đó của không gian xác suat mà X ≤ E[X]. Bài 2.20. Trong moi ô của bảng 100×100, ta viet m®t trong các so nguyên 1, 2, . . . , 5000. Hơn nǎa, moi m®t so nguyên xuat hi n trong bảng đúng 2 lan. Cháng minh rang ta có the chon được 100 ô của bảng thỏa mãn 3 đieu ki n sau: (1) Moi m®t hàng được chon đúng m®t ô. (2) Moi m®t c®t được chon đúng m®t ô. (3) Các so trong các ô được chon đôi m®t khác nhau. Bài 2.21. Trong so cách chon ra 3 đỉnh tà 8 đỉnh của hình l p phương đơn vị, có bao nhiêu cách chon thỏa mãn đieu ki n: 3 đỉnh được chon là đỉnh của m®t tam giác đeu. Bài 2.22. Trong m®t kỳ thi có n môn thi, trong đó có đe tieng Pháp và đe tieng Anh. Thí sinh có the thi bao nhiêu môn tùy ý, nhưng thí sinh chỉ có the chon m®t trong hai ngôn ngǎ cho moi môn thi. Với hai môn thi bat kỳ, ton tại m®t thí sinh thi hai môn này bang các ngôn ngǎ khác nhau. Neu moi m®t môn có nhieu nhat 10 thí sinh dự thi, hãy tìm giá trị lớn nhat có the của n. Bài 2.23. Trong m®t giải cờ vua có 40 kỳ thủ. Có tőng c®ng 80 ván đau, và hai kỳ thủ bat kỳ đau với nhau nhieu nhat m®t lan. Với m®t so nguyên n, cháng minh rang ton tại n kỳ thủ chưa he đau với nhau (Tat nhiên là so n càng lớn càng tot.) Bài 2.24. Cho p và q là các so không âm có tőng bang 1. Cháng minh rang (1 − pm)n + (1 − qn)m ≥ 1.
  • 42. 38 Viết đề tài giá sinh viên – ZALO:0973.287.149-TEAMLUANVAN.COM Σ n | 3 Σ C ≡ 1 (mod p ). p 2 Σ Σ x C ≡ 0 (mod p ).k Σ kC 2 Σ (−1) C k p p+j k k k j=0 Bài 2.25 (Hungarian MO 2001). Cho m và n là các so nguyên. Cháng minh rang m là m®t ước của n m−1 k=0 (−1) Ck. Bài 2.26 (Chinese MO 1998). Xác định tat cả các so nguyên dương n ≥ 3 sao cho 22000 chia het cho 1 + C1 + C2 + C3. n n n Bài 2.27 (G p gơ Toán hoc 2014). Cho p ≥ 5 là so nguyên to và m, k ∈ Z+ . Cháng minh rang pk+2 Cp−1 mpk−1 — 1. Bài 2.28 (Putnam 1996). Cho so nguyên to p ≥ 5 và k = ,2p , . Cháng minh rang k i 2 p i=0 Bài 2.29 (Putnam 1991). Cho p là so nguyên to lẻ. Cháng minh rang p Σ CjCj ≡ 2 + 1 (mod p ). Bài 2.30 (Thi chon đ®i tuyen VMO 2012, tỉnh Ngh An). Cho p > 3 là so nguyên to và t p hợp M = {1; 2; . . . ; p}. Với moi so nguyên k thỏa mãn 1 ≤ k ≤ p ta đ t Ek = {A ⊂ M : |A| = k}, và xk = A∈Ek (min A + max A). Cháng minh rang p−1 k 3 p k=1 Bài 2.31 (VMO 2017). Cháng minh rang 1008 a) 2017 ≡ 0 (mod 2017 ). k=1 504 b) 2017 ≡ 3 22016 − 1 (mod 20172). k=1 Bài 2.32. Cho ā là b® (a1, a2, . . . , an) các so thực dương. Đ t P0 = 1, Pk = Pk(ā), Er = Er(ā). Cháng minh rang Pk−1Pk+1 ≤ P2 (k = 1, 2, . . . , n − 1) . (Neu các ai đeu dương và không đong thời bang nhau thì ta có dau bat đȁng thác thực sự). k
  • 43. 39 Viết đề tài giá sinh viên – ZALO:0973.287.149-TEAMLUANVAN.COM n 2 2 3 n Bài 2.33. Cho các so ai > 0 (i ∈ {1, 2, 3, . . . , n}) và không đong thời bang nhau. Cháng minh bat đȁng thác 1 1 1 P1 > P 2 P 3 > · · · > P 3 . Bài 2.34. Cháng minh rang neu n và k là hai so tự nhiên thỏa mãn 0 ≤ k ≤ n thì Bài 2.35. Cháng minh rang n 2n+k .Cn 2n−k ≤ (C2n) . C2 + 2C3 + · · · + (n − 1)Cn > (n − 2)2n−1. n n n C1 + 2C2 + 3C3 + · · · + nCn Bài 2.36. Cháng minh rang với n ∈ N, n > 3. n n n n n < n!, C
  • 44. 40 Viết đề tài giá sinh viên – ZALO:0973.287.149-TEAMLUANVAN.COM KET LU N Lu n văn đã trình bày được nhǎng van đe sau: 1. Trình bày các bài toán cơ bản ve xác suat, khái ni m xác suat, tính chat, quy tac tính, xác suat có đieu ki n, công thác Bernoulli, công thác Bayes ve xác suat đay đủ. Các đại lượng đ c trưng của xác suat như kỳ vong, phương sai, bien ngau nhiên rời rạc. 2. Trình bày các dạng toán thường g p ve xác suat và các áp dụng liên quan. 3. Trình bày được các bài toán đại so có the áng dụng xác suat đe giải. 4. Trình bày được các bài toán thi olympic có n®i dung xác suat. Do thời gian thực hi n không nhieu và khả năng còn hạn che nên lu n văn mới chỉ đưa ra được m®t so dạng toán ve xác suat rời rạc và áng dụng của nó trong các bài toán thi hoc sinh giỏi. Em xin chân thành cảm ơn!
  • 45. 41 Viết đề tài giá sinh viên – ZALO:0973.287.149-TEAMLUANVAN.COM Tài li u tham khảo A Tieng Vi t [1] Nguyen Văn M u (2017), Tő hợp và các dạng toán liên quan, NXB Đại hoc Quoc gia Hà N®i. [2] Tạp chí TH&TT (2007), Các bài thi Olympic Toán trung hoc phő thông Vi t Nam (1990-2006), NXB Giáo dục. [3] Tran Nam Dũng, ”Phương pháp xác suat ”, bài viet đăng trên trang Thông tin toán hoc, H®i Toán Hoc Vi t Nam tháng 12 năm 2012 t p 16 so 4, tháng 3 năm 2013 t p 17 so 1, tháng 6 năm 2013 t p 17 so 2. [4] Đào Hǎu Ho (1996), ”Xác suat thong kê”, NXB Đại hoc Quoc gia Hà N®i, trang 3 – 49. [5] Đào Hǎu Ho (2011), ”Hướng dȁn giải bài toán xác suat thong kê”, NXB Đại hoc Quoc gia Hà N®i, trang 3 – 52. [6] Nguyen Văn Cao, Tran Thái Ninh (2006), "Bài t¾p xác suat và thong kê toán", Đại hoc Kinh te Quoc dân, trang 5 – 74. B Tieng Anh [7] Kenneth H. Rosen (2012), Discrete Mathematics and its Applications, Seventh Edition, Mc Graw Hill, pages 447, 449, 450. [8] Paulo Ney de Sausa, Jorge- Nume Silva (1998), Berkeley Problems in Mathematics, Springer. [9] Titu Andreescu, Zuming Feng (2002), 102 combinatorial problems from the training of the USA IMO team.
  • 46. 42 Viết đề tài giá sinh viên – ZALO:0973.287.149-TEAMLUANVAN.COM [10] Dusan Djukic, Vladimir Jankovíc, Ivan Matíc, Nikola Petrovíc, ”The IMO compendium”, Spinger, pages 338, 661, 667.